Ευκλειδης Β 45

84

Upload: demi-de

Post on 16-Mar-2016

242 views

Category:

Documents


0 download

DESCRIPTION

 

TRANSCRIPT

Page 1: Ευκλειδης Β 45
Page 2: Ευκλειδης Β 45

άλγεr>ρα dτ4« Α 'fόξ,1 f\ooiov �.ω

,... ,_

Γ:.(/c� -.-" c;r.;-r::, αξιολόγηση-ς

σrο

ΕΚΔΟΣΕΙΣ ΠΑΤΑΚΗ www.patakis.qr

Από τp Παιδαγωγικό Ινστιτούτο του Υπουργείου Παιδείας εγκρίθηκαν

53 βιβλία και σύνολα εκπαιδευτικού υλικού των ΕΚΔΟΣΕΩΝ ΠΑΤΑΚΗ ως ΕΠΙΣΗΜΑ ΣΧΟΛΙΚΑ ΒΙΒΛΙΑ για τα σχολεία όλης της Ελλάδας.

Page 3: Ευκλειδης Β 45

Υπεύ8υνοι Έκδοση� Ευσταθίου Ευάγγελος, Μπαραλής Γεώργιος, Σφαrής Ιωά.ννης

Συνιακηκή Οιιάδα: Ανδρουλακάκης Νίκος ΒακαλcΊπουλος Κώστας Βλάχου Αγγελική Γιαννοοπύρος Σωτήρης Γράψας Κωνσταντίνος Δούναβης Αντώνης Ευθύμοyλου Πέτρος Καλίκας Σταμάτης Καρακατσάνης Βασίλης Καρκάνης Βασίλης Καισούλης Γιώργος Κeρασαρίδης Γιάννης ΚηπουρcΊς Χρήοως ΚcΊνψας Νίκος Κυριακόπουλος Ανιώνης Κυριακόπουλος Θανάσης ΚυριακοηούλουοΚυβερνήωυ Χρυσι. Λαζαρίδης Χρi]οιος Λουρίδας Σωτήρης Μαλαφέκας Θανάσης Μώκος Χρήστος Νιζιώρας Ηλίας Ρήγας ΘccΊδωρος ΣΟ:ίιη Εύα ΣακελλιΊρης Βαοίλης Τασοόπουλος Γιώργος Τσικαλuυδάκης Γιώρ,·ος Τσιού].Lας Θανάσι1ς Χαραλαμπίδης Γιώργος Χαραλα�mοπούλου Λίνα Χαραλάμπους Θάνος ΧριστcΊηουλος Παναγιώτης

Συvcρyάιcς: Φeλούρης Αργύρης Φίλη Χριστίνα Καρβέλης Γιάννης Μαντάς Ιωάννης Χριστόπουλος Θανά.σης Σκοτίδας Σ.

ΕΚΔΟΣΗ ΤΗΣ ΕΛΛΗΝΙΚΙΙΣ ΜΑΘΗΜΑτΙΚΗΣ ΕΤΑΙΡΕΙΑΣ ΙΊΑl'<ΈΠΙΣΤΗΜΙΟΥ 34 - 106 79 ΑΘΙ-ΙΝΑ Τηλ36 17 78-±- 36 16 532 Fωο 36 41 025 Εκδόιης: Αλεξανδρής Νικόλαος ΔιευDυvιής: ΊΌρλής Ιωάννης

ISSN: 1105 - 7998 Επιμέλεια Έκδοση�:

Μαραγκάκrις Στέλιος ΣΥΝΔΙ>ΟΜΕΣ:

ΕΛΛΗΝΙΚΗ ΜΑΘΗΜΑΤΙΚΗ ETAIPEIA Τεύχοι; 45 ο Ιούλιοι; ο Αύyουστοι; ο Σεπτέμβριοι; 2002 ο Ευρώ: 2,50

eomail: info@hmsogr wwwohmsogr

I ΜΑΘΗΜΑ ΊlΚΟ ΠΕΡΙΟΔΙΚΟ ΠΑ ΤΟ Λ ΥΚΕΙΟ

ΠΕΡΙΕΧΟΜΕΝΑ 2 Η ΔcύΙΙψΙ1 Μαllημαrική Σχολή rωv Αllηvών

4 Το Βήμα rου Ευκλcίδη

4 Δρασrηριόιηια OIΙJV Τάξη

5 Αριllμός Υπο1ογισrής 'Ρ 11' (Πασκαλιvll)

8 Οι φcίς ηλικίες

12 Ο ΈυιU,είδης' προιείνει.οοΕυκλείδη και Διόφαvrσ

22 ΗΟΜΟ MATHEMAΊICUS 24 Μαllημαιικοί Διαγωνισμοί - Μαllηιιαηκές Ολυιιπιάδες

I ΜαDημαιικά yια ιηv Α' Τάξη ιου Λυκείου I

33 Υπάρχουν και Ανισόrηιες

35 Γcωμεφία π1ς Α' rάζης rου Λυκείου (Βασικά Γεωιιcφικά σχήμαrα)

40 Σχέσεις ιιεrαζύ cυllυγράμιιων φημάrωv

41 θέμαrα cζάσκησης

I ΜαDηιιαιικά yια ιηv Β' Τάξη ιου Λυκείου I {:_\ fU is V<il- 4'ϊ (ψ) -{) :2 � Θvι ()(;ο Λiu61<D 43 Δια,'1Jσμαια

49 Μcφικές σχέσεις

57 Τριγωνσμcφία Β' Λυκείου 232. ι -Ό- '3'8 2.22

I ΜαDημαηκά yια ιηv Γ' Τάξη ιου Λυκείου I

63 Γεωμεφικοί rόποι σrσ Μιγαδικό Επίπεδο

69 Μαllηιιαιικά Γενικής παιδείας llcιιαrα rσυ 1ου Κεφαλαίου

78 Πρσ6λήιιαrα απ' ΙΙJ" καllημcριvή Ζωή

Tcιixo� 2,50 cυρώ 79 Η σrήλη rψ;; Αλληλογραφίας Ειήσια συνδρnJDΊ 10,00 ε Ηρώ (Σ.χολ.)

Σuνδρο�η1ιέ� lΟ,αΉ-2,00 ευρώ (Ταχ.) 1---------------------------------1 Οργανισμοί: 18,00 ευρώ Σιοιχcιοllεοία - Σελιδοrιοίηοη Ταχο Ειιιιαyέ� Τ. Γραφεlο ΑΙJήvα 54, Τοθο 30044 Ελληvικι'ι ΜαΟιηωrικiι Εrαιρεία

ΕκιύπωοrιΙΝΤΕΡΠt•ΕΣ ΛοΕο , lερά ο&\� 81 - 83 YncuO. Τuπnγραφεί.οu η Τρικεριώιης-Ειλ;3-i7465'1

Page 4: Ευκλειδης Β 45

Η Δεύτερη Μαθηματική Σχολή των Αθηνών

Διdνές unορικό: Οι ξένοι μαθηματιιωί πσυ αvαφέροvrω εδώ υπήρ­

ξαν φυσιογvωμiες. Οι Poincare και Hilbert θεωρούvrω οι μεyσλύτεροι γεvιιωί μαθηματιιωί του 2(fυ αιώνα. Οι Monge, Lie, Darboux, Ε. Cartan, οι μεγαλύτεροι δια­φορογεωμέτρες των αντιστοιΧων εποχών. Οι Ε. Borel, Hαdamard, Painleve, Picard ιωρυφαίοι δημισυργοί οτην Ανάλnση (και όχι μόνο). Ο Goursat, αναθεμΕλιω­τής της Θεωρίας Συναρτήσεων μιας μιγαδικής μεταβλη­τής, διεθνής αυθεντία στις μερικi:ς διαφορικi:ς εξισώσεις και σοφός συγγραφέας οτην ανδλυση. Ο F. Klein, ανα­νεωτής της Γεωμετρίας. Ο Chevalley, μi:ΝJς των Bourbaki, ιωρυφαίος αλγεβριστής.

Στο πρώτο μισό του 20ου αιώνα, ο ελληνικός λαός εξορμά· εθνικά, κοινωνικά, πνευματικά, καλλιτεχνικά. Εδώ αναφερόμαστε σε πανεπιστημιακούς Καθηγητές που κεντρικά συμβάλλανε στην μαθηματική παιδεία των γενεών του ΟΧΙ και της Εθνικής Ανriστασης. Από τις οποίες, άλλωστε, βyήκε ο Χρίστος Παπακυρισχό­πσυλος (1903-1976), από τους διασημότερους μαθη­ματικούς της εποχής μας.

Η αυγή του 20ου αιώνα υπήρξε ευοiωνη για τη νε­οελληνική μαθηματική έρευνα. Τον σεβασμό που εί­χαν εξασφαλίσει, γι' αυτήν, διεθνώς, οι εργασίες των Νικολαίδη, Κ. Στεφάνου και I. Χατζηδάκη ανανέωσαν οι τρεις νέοι δρόμοι που άνοιξαν, αντίστοιχα στον Λο­-γισμό των Μεταβολών ο Κωνσταντίνος Καραθεοδω­ρής (ήδη γράψαμε για τον μεγάλο μαθηματικό του Ελ­ληνισμού), στην θεωρία Συναρτήσεων μιας μιγαδικής μεταβλητής ο Αθηναίος Γεώργιος Ρεμούνδος (1878-1928) και λίγο αργότερα (1905), στις Μερικές Διαφο­ρικές Εξισώσεις ο Κεφαλλονίτης μαθηματικός Πανα­γιώτης Ζερβός (1878-1952). Οι Ζερβός και Ρεμούνδος, συμφοιτητές - βιοπαλαιστές στην Αθήνα, συνεχίζουν τις σπουδές τους στο Παρίσι Ο Ρεμούνδος γενίκευσε στις αλγεβροειδείς συναρτήσεις το θαυμαστό θεώρημα του Picard για τις ακέραιες. Δοσμένος στην έρευνα, θα είναι παραγωγικότατος στην σύντομη ζωή του. Υ πάρ­χουν αναφορές των εργασιών του από τους πρωτοπό­ρους Ρ. Montel, Ο. Blιunenthal (βλ και την εyκuκλο­παίδεια των Μαθηματικών από τον Painleνe). Γίνεται καθηγητής της Ανώτερης Μαθηματικής Ανάλυσης στο Πανεπιστήμω Αθηνών (1912) και το 1916 στο Ε.Μ.Π.. Τύπος λειτουργικού μαθηματικού, f).ηε στους φοιτητές του «είμαι προϊόν των ασκήσεων». Έ-

της Χριστίνας Π. Φίλη<*>

yινε Ακαδημαϊκός από την ίδρυση της Ακαδημίας (1926). Τα βιβλία του Θεωρία Διαφορικών Εξισώσεων (τ. Α' 1912, τ. Β' 1914), Αρχαί της Θεωρίας των Αναλυτικών Συναρτή­σεων (1914) κ.ά. συνοπτικά, περιειcrικά και ωrΜ. συμβάλ­λουν στην απόκτηση στέρεων μαθηματικών γνώσεων. Ως αντιπρόσωπος της Ελλάδας στη Κοινωνία των Εθνών (τμή­μα: πνευματική επικοινωνία) ταξιδεύει συχνά, καταβάλλο­ντας την κλονισμένη υγεία του. Ο Ζερβός πρώτος διέκρινε ότι η αποτυχία μεγάλων μαθηματικών να αποδείξουν την περιώνυμη εικασία του Monge (1784) αpείλεται στο ότι αυ­τή ήταν εσφαλμένη. Οι καθηγητές Borel και Picard του Ρε­μσύνδου και Darboux Goursat, Hadamard Painleνe του Ζερβού αντιλαμβάνονται τη σημασία των νέων κατευθύν­σεων. Π.χ. την έρευνα του Ζερβού συνεχίζουν και οι Goursat, Hilbert και Ε. Cartiιn. Αυτός, την θεμελιώδη εργα­σία του 1914, την αρχίζει έτσι: «Σε μια πρόσφατη ανακοί­νωση (1913) ο κ. Ζερβός γενικεύει ένα θεώρημα του κ. Hilbert (1912) ... )). Καθώς οι σχετικές έρευνες των Ζερβού και Ρεμούνδου συνεχίστηκαν (για όλη τους τη ζωή), τους ανατέθηκε η συγγραφή των αντιστοίχων μονογραφιών στην προπολεμική περιώνυμη γαλλική σειρά Memoήal dec Sci. Math. Σχετικά, στο σαpό βιβλίο του Linear Analysis and Representations Theoιy (Spήnger 1970) ο S.A. Gaal γράφει: «πριν μια γενιά οι ομάδες Lie ονομάζονταν συνεχείς ομάδες. Οι κύριες πηγές στο θέμα αuτό είναι τα βιβλία των Lie και Elie Cartan, ένας τόμος από τον Kowaleνski και ένα βιβλίο από τον Π. Ζερβόν για το πρόβλημα του Monge. Οι διάφο­ροι τόμοι και εκδόσεις ενός βιβλίου του Cheνalley είναι πω πρόσφατοι)). Εξάλλου, η βιβλωyραφία του σημαντικού βι­βλίου V alue Distribution Theoιy των L. Sario και Κ. Noshiro (νan Nostrand 1966) περιλαμβάνει εργασίες των Ρεμούνδου και του ερευνητικού του μαθητή - συνεχιστή του, Θεόδωρου Βαρόπουλου (1894-1957 Καθηγητού στο Παν/μω Θεσσαλονίκης).

Με μοναδικό ταλέντο στις ξένες γλώσσες, «αυτόματη)) ακριβολογία και πλατειά παιδεία μαθητής των Darboux, Fuchs, Hilbert, F. Κlein, Η Schwarz, ο ΝLΧατζιδάχης (Κα­θηγητής στο Πανεπιστήμω Αθηνών από το 1904) (1872-1942) θα συνεχίσει επάξια την πατρώα ερευνητική παράδο­ση, αγκαλιάζοντας τη νέα, τότε, Κινητική Απειροστική Γε­ωμετρία, των Γάλλων. Σ' αυτήν είναι αφιερωμένο και το ομώνυμο βιβλίο του. Ακόμα πω προχωρημένη ερευνητικά, η μονογραφία του «Σμήνη και συμπλέγματα καμπυλών και επιφανειών)). Η γενίκευση των Θεωριών του Darboux σε ν­διάστατο χώρο (1900) εντυπωσιάζει Μάλιστα σ' αυτήν τη

(*)Επ. Καθ. Ε.Μ.Π., Docteur d' Etat, Αντεπιστέλλον Μέλος, της Διεθνούς Ακαδημίας, της Ιστορίας των Εmστημών

ΕΥΚΛΕΙΔΗΣ Β' λστ' τ.l/2

Page 5: Ευκλειδης Β 45

Αρχαία Ελληνικά Μαθηματικά

μελέτη του εισάyει στην Ελλάδα την έννοαι της πολ­λαπλότητας, έναν χρόνο μεrά την παρουσίασή της (1899) από τον Poincare.

Στην ομ1λi.α του (1910) στο σύ/J..iyyo των φοιτη­τών του Μαθηματικού, παρουσιάζει τις βασικές κα­τευθύνσεις για τη δημιουργία <<Μαθηματικών Σχο­λι:ίJV», σημειώνοντας πως το δύσκολο είναι να αφυπνι­σθεί το ενδιαφέρον των φοιτητών, γιατί τα Μαθηματι­κά δεν είναι άμεσα προσιτά και με την πρώτη ματιά ελκυστικά, δεν υπάρχει κέρδος σλλ1:ι. ούτε και θαυμα­σμός, γιατί τα επιτεύγματα παραμένουν γνωστά σ' έ­ναν πολύ περωρισμένο κύκλο. Όμως η έρευνα προ­σφέρει μεγάλη πνευματική ικανοποίηση γι' αυτό ό­ποως ζητά για τον εαυτό του <<υψηλότερες απολαύσεις του πνεύματος αυτός ας σπουδάσει προπάντων Μαθη­ματικά>).

Καθηγητές στο Παν/μω Αθηνών οι Π. Ζερβός, Γ. Ρεμούνδος, N.l. Χατζιδάκης, φίλοι μεrαξύ τους και φι­λικοί προς τους φοιτητές (για πρώτη φορά πολυάριθ­μους), με την άνεση του διεθνούς κύρους, της διδακτι­κής πείρας (οι Ρεμούνδος και Ζερβός είχαν διδάξει και στην Μ.Ε.) και της αφοσίωσης στην αποστολή τους, δημιουpyήσανε υψηλά πρότυπα (όπως προηγουμένως ο Κ Στέφανος και ο I. Χατζιδάκης). Ένας ολόκληρος κόσμος αποφοίτων με ξεκάθαρες, λιτές και στερεές γνώσεις θα μορφώσει την Ελλάδα, θα συμβάλει απο­φασιστικά στο έπος του '40 και θα πρωταγωνιστήσει στην ολόπλευρη Εθνική Αντίσταση, προσφέροντας και έαν κορυφαίο Μάρτυρά της, τον Γιώργο Κότσηρα (το νέο με την έντονη παρουσία στη φωτογραφία εκ­δρομών Καθηγητών-φοιτητών, συνηθισμένες τότε).

Οι τρεις Καθηγητές καλύψανε, αθροιστικά, με συγγράμματα όλn. τα τότε διδασκόμενα μαθήματα. Ενδεικτικά: Τα <<Στοιχεία Ανωτέρας Άλγεβ�) του N.l. Χατζηδάκη. Η «Θεωρία των Διαφορικών Εξισώ­σεωV)) του Γ. Ρεμούνδου (από τις παραδόσεις του το, γράψαν οι Κωνσταντίνος Παπαϊωάννου - αpγότερα Καθηγητής της Μηχανικής στο ΠΑ και στο Ε.Μ.Π. και Ακαδημαϊκός, και ο Κώστας Χαράκης, εξαίρεtος Κύπρως μαθηματικός). <<Απειροστικός Λογισ�) του Π. Ζερβού (έντυπο, 1929). Με αυτό, η από το 1918 ει­σαγωγή της Θεωρίας Συνόλων και του αριθμητικά θε­μελιωμένου Απειροστικού Λογισμού, από τον Π. Ζειr βό, αποτυπώθηκε γραπτά. Είναι η μεγαλύτερη γενική προσφορά του στην νεοελληνική μαθηματική παιδεία. Θαυμαστής του βιβλίου, ο Χ. Παπακυραικόπουλος (προηγουμένως, μαθητής του Νικολάου Κριτικού στο Ε.Μ.Π.).

Οι τρεις Καθηγητές εισήγαγαν τον θεσμό των <<Ανωτέρων Φροντιστηρίων)) (Σεμιναρίων) για την συστηματική καλλιέργεια έρευνας υψηλού επιπέδου (συχνά ισότιμο με εκείνο στο τότε Παρίσι). Παράλλη­λα, ο εραστής της θεωρίας των Αριθμών Π. Ζερβός προσανατόλισε τον μόνιμο κάτοικο Κρήτης Γεώρyω Ξηρουδάκη στην Δtαρανtική Ανάλυση, στην οποία θα

αναδειχθεί συν τω χρόνω σε σπουδαίο διεθνή έλληνα ερευ­νητή - αυθεντία. Το έργο του συνεχίστηκε απ' τον εξαφετι­κό μαθηματικό και δραματικά άrοχο Ιάσωνα Κατζουράιcη.

Μαθητής και θαυμαστής του Poincare, ο Π. Ζερβός, θα μεrαφράσει (1912) το περιώνυμο βιβλίο του «Επιστήμη και Υπόθεση)). Εισάyει έτm στην Ελλάδα, τη φιλοσαρία των θετικών επιστημών, αντικείμενο ισόβιας αφοσίωσής του. Ιδρυτής και πρόεδρος της Ελληνικής Φιλοσαριιcής Εταφεί­ας (1933), η οποία θα ατονήσει όταν η αντιπνευματική δι­κτατορία συλλάβει μέλη της, όπως τον Παναγιώτη Κανελ­λόπουλο. Ως Ακαδημαϊκός, θα επισημάνει πρώτος, την ταύ­τιση της σκέψης του ΙD..άτωνα (στην παραβολή του Σπη­λn.ίου) και του Poincare στο παραπάνω βιβλίο (1948-49).

Οι Π. Ζερβός και N.l. Χατζιδάκης θα εγκαινιάσουν την Δαιβαλκανιιcή Συνεργασία, μεrαφέροντας σ' αυτήν το πολι­τικό όραμα του Αλεξάνδρου Παπαναστασίου. Το πρώτο συνέδρω είχε μεγάλη επιτυχία και συμμεrείχαν μεγάλοι Βαλκάνωι μαθηματικοί Οργανωτικός φορέας του ήταν ο δημοκρατικός Σύνδεσμος Μαθηματικών Μέσης Παιδείας, ο οποίος θα γίνει ο αιμοδότης στην Εθνιιcή Αντίσταση.

Ο Π. Ζερβός θα ιδρύσει και θα εκδίδει το περωδucό Revue Math6natique de 1' Union Interbalcanique (1936-41 ).

Γαι το επίπεδο του περωδικού αρκεί να αναφέρουμε ότι πε­ριέχει και άρθρα των Κ Καραθεοδωρή και Αλεξάνδρου Δί­γκα (μεrαπολεμικά Καθηγητή στο Βερολίνο και μ.αθητού του Ε. Schmitdt).

Οι αρχικές εργασίες του Π. Ζερβού, ανήκουν στην Θεωρία Συναρτήσεων και στις Αλγεβρικές Εξισώσεις. Από το Θεώρημά του σ' αυτές, το οποίο θα εκθέσει στο τελευ­ταίο μάθημά του (1949) θα δρομολογηθεί μαι πορεία με μα­κροπρόθεσμες ερευνητικές συνέπειες. Τέλος, στα 1971, ο καθηγητής της Σορβόνης Μ. Janet θα δημοσιεύσει μακρό άρθρο με τίτλο «0 Π. Ζερβός και το Πρόβλημα του Monge)) (Bull. Sci. Math.).

Σημειώσεις: 1 . Τα Σεμινάραι που αναφέραμε ήταν ανοικτά σε όλους

τους ενδιαφερομένους. 2. Η ατμόσφαιρα στο ΠΑ δεν ήταν μόνο τεχνοκρατική. 3. Ο Ν.Ι. Χατζιδάκης ήταν και δόκιμος ποιητής (ιδαιίτερα

σε μεrαφράσεις, καθώς μιλούσε 13 γλώσσες), με το όνομα <<Ζέφυρος Βρα.δυνός)). Ακόμα ήταν υποστηρι­κτής της δημοτικής γλώσσας. Το 1896 εγκαταλείπει τις σπουδές του στο Παρίσι και μαζί με τον θείο του Γε­ώρyω Χατζηδάκη (εμπνευσμένο γ"λιοοσολόγο) κατε­βαίνει στην Κρήτη και συντάσσεται με τους επαναστά­τες

ΕΥΚΛΕΙΔΗΣ Β' λστ' τ.l/3

Page 6: Ευκλειδης Β 45

ο I

ΔΡΑΣΤΗΡΙΟΤΗΤΑ ΣΤΗΝ ΤΑΞΗ

Η επίλύση ασκήσεων και προβλημάτων σιην τάξη στα πλαίσtα της νέας δtδαιαιιcής των Μαθηματικών, είναι άμεσα συνδεδεμένη με την καθημερινή διδακτική πράξη. Οι μεν ασκήσεις στηρίζσύν τη νέα μαθηματική έννοια σιην κατανόησή της, βοηθούν σιην αφοι.ωiωσή της' Τα δε προβλήματα είναι πιο σύνθετα, απαιτούν γενικότερες -γνώ­σεις Μαθηματικών και οργανώνσύν τη σκέψη των μαθητών' Τούς βοηθούν να αναπrύξσύν πρωτοβουλίες και την κριτική τους ικανότητα. Μπορούν ακόμη να ανταλ­λάσσΟIΝ απόψεις μεrαξύ τους, να ανακαλύπτσύν τη σ(Ι}­στή πορεία πού θα τους οδηγήσει στο σωστό αποτέλεσμα. Οι μέθοδοι και οι θεωρiες πού f:ι..OIN αναπτυχθεί σχετικά με τη δtδαιαιιcή των Μαθηματικών και ακόμα πιο συyκε­κρψένα, δηλαδή πως το κέντρο βάρσύς της διδασκαλίας μεrατοπίζεται από τον καθηγητή στο μαθητή, είναι αντι­κείμενο έρεύνας πo').,)..fjJV ετών και πo').,)..fjJV ερευνητών" Αναμφtσβήτητα ωrοτελσύν σταθμό στη δημωυργiα νέων μεθόδων και Θεωριών και ταυτόχρονα πραγματικό βοηθό για κάθε καθηγητή Μαθηματικών.

Ο Ευκλείδης Γ στα τεύχη τον παρουσιάζει μία πολύ συyιφοτημένη δουλειά σχετικά με τα παραπάνω. Στα τεύ­χη 38 και 56 αναφέρεται στα προβλήματα των Μαθηματι­κών.

Το πρόβλημα σιην τάξη με τη ι.ωρφή ομαδικής εργασίας και νέας μεθόδού διδασκαλίας στο Ελληνιιcό σχολείο είναι σημαντική και πρωτοποριακή ενέργεια. Ό­μως δεν είναι απλή υπόθεση η νέα αυτή ι.ωρφή διδασκαλί­ας των

Μαθηματικών, για παράδειγμα ελάχιστα είναι τα προβλήματα στα βιβλία των Μαθηματικών του Γυμνασίου Το ζήτημα είναι πως θα εξασφαλiσούμε οι.ωωyένεια στις τάξεις των μαθητών μας, πως θα επιτύχσύμε πρωτογενές αποτέλεσμα

σιην εργασία πού τους αναθέτουμε, όταν μάλιστα αυτή απαιτεί μια, προεργασία στο σπίτι και δεν γίνεται κατ' ευθείαν σιην τάξη. Πραγματικά απαιτείται πολύ δουλειά και χρόνος ώστε να εφαρμόσσύμε τις νέες μεθόδούς πού θέλούν το μαθητή στο κέντρο της εκπαιδειmκής διδασκα­λίας και ταυ εκπmδεuτικό σιΝtονιστή και βοηθό.

Οι συνάδελφοι Γιώργος Κόσυβας και Παύλος Μά­στακας από το l0Πεφαματικό Αγ. Αναργύρων στα πλαί­σια της δtδαιαιιcής των Μαθηματικών, έδωσαν το εξής

της Βλάχου Αyyελική

πρόβλημα στους μαθητές τους σιην Α' Λυκείου ((Δύο ευθεiες ε1, ε2 τέμνονται fl,ω αΠ τη σελiδα τον

τετραδίου σας. Να γράψετε τη διχοτόι.ω της γωνίας πού σχηματίζσύν

οι παραπάνω ευθείες, χωρiς να βγείτε fl,ω ωt' τη σελiδας.>> Οι μαθητές χωρίστηκαν στις παραιcάτω ομάδες όπου

και ανέπτυξαν δημωυργική συνεργασία: Οι ομάδες εργασίας το πρώτο δίωρο εργάστηκαν χ(Ι}­

ριστά και στο επόμενο δίωρο παρουσiασαν σ' όλη τη τάξη τα αποτελέσματα των λύσεων που είχαν βρει Μαζί με τους καθηγητές τους Γ. Κόσυβα και Π. Μάστακα χώρι­σαν σε τέσσερις κατηγορiες τις λύσεις που είναι συνολικά 14.

Ο Ευκλείδης Β' χαίρεται για την πρωτοβουλία και τη νέα πραγματικότητα που εισάγεται στο σχολείο και συγ­χαίρει τους μαθητές για το ενδιαφέρον που έδειξαν σ)J.iJ. και για το αποτέλεσμα της εργασίας τους.

� Ξ:::-Ι ΣΕ ΙΈΝΙΙΑ θΕΝΛΤΑ

ΜΑΘΗΜΑΠΚΟΝ ΓΕΝΙΚΑ ΘΕΜΑΤΑ r ΛΥΚΕWΥ τ Ω θEJ1Kiit-lf�OI'ΊΚHL

Κι\ll.ΎθVΝΙιι.ι

θιωρηnκlsοοι.ιιiσεοc; Γ ΛΥΚΕΙΟΥ ·-- (Θετικής· Τεχνολογικής . ..... _......,.. · --- ·--·-::=:=. κατεύθυνσης) EWJ�IWoXOl

Συγγραφική • Β. Βλάχοι; • Π. Κουτσούκοc;

ομά3α • Π. Ξηρόκωσταc; Μαθηματικών • Χ. Πλατήc;

Ένα βιβλίο σταθμόc; στη μέση εκπαί3ευση με α. Αναφορά στα λεmά σημεία των Μαθηματικών

και στα σημεία που γίνονται λάθη β. Εκατοντά3εc; θεωρητικά θέματα όλα λυμένα ν' Για τουc; μαθητέc; που στοχεύουν στο 30 και 40

θέμα. ν' Για τουc; Μαθηματικού«; που 3ι3άσκουν στο

Λύκειο, στο φροντιστήριο, στο ι3ιαίτερο.

Κοστίζει περίπου όσο μια ώρα ιδιαίτερο. Εξασφαλίζει εκατοντάδες ώρες διδασκαλίας.

Εκδόσειc;: Β. ΒΛΑΧΟΣ (Μαθηματικόc; και Φυσικόc;)

ΠΑΠΑΦΛΕΣΣΑ 3 13451 ΖΕΦΥΡΙ • ΑΘΗΝΑ Τηλ.: 010 2384834 Fax: 010 2320954

Page 7: Ευκλειδης Β 45

Το βήμα του Ευκλείδη

ΑΡΙΘΜΗΤΙΚΟΣ ΥΠΟΛΟΓΙΣΊΉΣ «Ρ 11» (ΠΑΣΚΑΛΙΝ 11)

ΗΑΦΗΓΗΣΗ Στο Εθνικό Μουσείο Τεχνών και Επαγγελμά­

των, εκείνο το καλοκαιριάτικο απόγευμα του 1995, επικρατούσε μια ασυνήθιστη αναστάτωση. Οι εm­σκέπτες που αναμένονταν δεν ήταν κάποιο συνηθι­σμένο τουριστικό γκρουπ, αλλά μια ομάδα Γιαπω­νέζων μαθηματικών. Συμμετείχαν στις εργασίες του συνεδρίου που γινόταν εκείνες τις μέρες στο Παρίσι, για το έργο του μεγάλου και πρόωρα αδικοχαμένου συμπατριώτη τους Γιουτάκα Τανιγιάμα.

Το μικρό λεωφορείο σταμάτησε στην οδό Σαιν Μαρτέν και οι επισκέπτες πέρασαν την είσοδο του παλιού μοναστηριού. Στους χώρους του στεγαζόταν το Μουσείο από το 1799, με απόφαση του Διευθυ­ντηρίου. Στην Αυλή της τιμής τους υποδέχτηκε ο Πρόεδρος του Διοικητικού Συμβουλίου του ΕΜΤΕ, γεγονός που υπογράμμιζε την ιδιαίτερη σημασία της επίσκεψης. Θα τους ξεναγούσε ο ίδιος.

Πρώτα κατευθύνθηκαν προς την εκκλησία του Σαιν Μαρτέν ντε Σαν, περνώντας από την περίφημη «Αίθουσα της Ηχούφ, όπου βρισκόταν σχεδόν ολό­κληρος ο εξοπλισμός του εργαστηρίου του Λαβουα­ζιέ. Στάθηκαν στο Χορό της παλιάς εκκλησίας και ο Πρόεδρος τους έδειξε το Εκκρεμές, το οποίο έφτια­ξε ο Φουκώ πριν από 140 χρόνια για να αποδείξει την περιστροφή της Γης. Μετά έμειναν κάμποση ώρα στο χώρο με τα εκθέματα που σχετίζονταν με την εξέλιξη των θερμικών μηχανών και της αεροπο­ρίας. Κοίταξαν με απορία τη μικρογραφία του α­γάλματος της Ελευθερίας, του λιμανιού της Νέας Υόρκης.

-Είναι το πρώτο μοντέλο που έφτιαξε ο Μπαρ­τολντί, τους εξήγησε ο Πρόεδρος.

Φωτογραφήθηκαν γελώντας δίπλα του, δεν έκαναν όμως το ίδιο και με το άγαλμα που βρισκό­ταν στην άλλη πλευρά. Η μορφή του Πασκάλ τους ενέπνεε σεβασμό.

Βγήκαν από την εκκλησία, περιδιάβηκαν τις αίθουσες του πρώτου ορόφου με τον τεράστιο πλού­το των εκθεμάτων, σχετικών με τις εφαρμογές των εmστημών, και συνέχισαν στο δεύτερο όροφο. Τε­λευταίος σταθμός της περιήγησης οι αίθουσες 57 και 58, αφιερωμένες στα μαθηματικά. Εδώ βρίσκο­νταν και οι αριθμομηχανές, από τα αρχαία κινέζικα aριθμητήρια ως τις σύγχρονες.

Η ομάδα των μαθηματικών περιεργαζόταν με θαυμασμό την "Πασκαλίν", τον πρώτο Αριθμητικό Υπολογιστή του Πασκάλ. Το παράξενο κουτί με

Του Γιάννη Καρβέλη

τους τροχίσκους δε φάνταζε και πολύ εντυπωσιακό, αυτοί όμως καταλάβαιναν πολύ καλά τη μεγάλη του αξία, ιστορική και μαθηματική . . .

Ο Πρόεδρος παρακάλεσε να έχει την προσοχή τους.

- Κύριοι, σας έχουμε υποσχεθεί μια ενδιαφέ­ρουσα έκπληξη. Ακολουθείστε με παρακαλώ στη βιβλιοθήκη.

Ξαναγύρισαν στην Αυλή της τιμής και πέρα­σαν στην παλιά ψηλοτάβανη αίθουσα που φτιάχτηκε το 1300, για τραπεζαρία του μοναστηριού. Με δέος προσπαθούσαν να εκτιμήσουν το μέγεθος του πλού­του που φυλασσόταν εκεί. Περίπου 100.000 τόμοι με εmστημονικές και τεχνολογικές εφαρμογές!

Ο Πρόεδρος, έβγαλε από το χαρτοφύλακ� του δυο φύλλα χαρτιού, που φαίνονταν σαν φωτοτυΠίες παλιού ξεθωριασμένου χειρόγραφου. Καθάρισε το λαιμό του με ένα μικρό ξερόβηχα και άρχισε να μι­λάει με σοβαρό ύφος.

- Πριν από λίγες μέρες λάβαμε μια περίεργη ε­πιστολή. Ο ανώνυμος αποστολέας ισχυριζόταν ότι είχε στα χέρια του ένα γράμμα, που είχε στείλει ο Πασκάλ στην ανιψιά του το Μάϊο του 165 ... Η ημε­ρομηνία ήταν λίγο ξεθωριασμένη, αλλά αυτό δεν έχει μεγάλη σημασία. Μας έστειλε και ένα αντίγρα­φο του γράμματος, αυτήν τη φωτοτυπία. Ισχυριζό­ταν ότι έφτασε στα χέρια του από έναν καλόγερο, που το ξετρύπωσε στη βιβλιοθήκη κάποιου μονα­στηριού στη Νορμανδία και μας ζητούσε μια οικο­νομική προσφορά για το πρωτότυπο. Μας βεβαίωνε ότι το γράμμα είναι εξακριβωμένα γνήσιο και τε­λείωνε με τη δήλωση "θα έλθω πάλι σε επαφή μαζί σας σε ένα μήνα".

Δε δώσαμε και μεγάλη σημασία, παίρνουμε αρκετές τέτοιες εmστολές. Αναθέσαμε την εξέταση του γράμματος σε έναν ειδικό συνεργάτη του Μου­σείου, που κατά σύμπτωση είναι και μαθηματικός. Την άλλη μέρα μας περίμενε πρωί-πρωί για να μας ανακοινώσει ότι δεν μπορούσε να αποφανθεί ακόμα για τη γνησιότητα του γράμματος, τον προβλημάτιζε το ύφος και η δομή του. Οπωσδήποτε όμως εύρισκε το περιεχόμενό του πολύ ενδιαφέρον. Συμφωνήσαμε και μεις, όταν διαβάσαμε το κείμενο, αλλά αποφα­σίσαμε ότι δε θα το δημοσιοποιήσουμε πριν από την επόμενη επαφή με τον αποστολέα. Θέλαμε να βε­βαιωθούμε ότι δεν πρόκειται για απάτη ή φάρσα, αρκετά έξυπνη πάντως, όπως θα διαπιστώσετε.

Σταμάτησε, έβγαλε και καθάρισε τα γυαλιά του

ΕΥΚΛΕΙΔΗΣ Β' λστ' τ.l/5

Page 8: Ευκλειδης Β 45

Το βήμα του Ευκλείδη

και τα άφησε στο τραπέζι που ήταν πλάι του. Κοίτα- εμπλοκή ο μηχανισμός του εκθέτη και δε μπορούμε ξε για λίγο τους aνυπόμονους ακροατές και συνέχι- να κάνουμε επιλογές. Δεν είναι τίποτα σπουδαίο, αλ­σε. λά είμαι πολύ κουρασμένος για να το φτιάξω σήμερα.

- Αποφασίσαμε όμως, εξαιτίας μιας πολύ ευτυ- Θα δοκιμάσετε λοιπόν αναγκαστικά με τον εκθέτη χούς συγκυρίας -θα καταλάβετε μόνοι σας σε λίγο που έχει ήδη διαλεχτεί. " τι εννοώ-, να γνωστοποιήσουμε σε σας το περιεχό- Τότε ο Ζακ ρώτησε: " Ποιόν εκθέτη έχει τώρα μενο του γράμματος, κατά τη σημερινή σας επίσκε- θείε ; " ψη. Ανεξάρτητα από το αν είναι γνήσιο, οπότε και Και εγώ τους απάντησα: ''Δε θυμάμαι παιδιά θα έχουμε μια σημαντική ιστορική πληροφορία για μου που έχει μπλοκάρει, αλλά θα τον βρείτε μόνοι την εξέλιξη της aριθμομηχανής του Πασκάλ , πα- σας, είναι πολύ εύκολο ! Πάω τώρα να κοιμηθώ λίγο, ρουσιάζει σήμερα και άλλου είδους ενδιαφέρον. δουλεύω συνέχεια από το πρωί. Θα γυρίσω σύντομα

Φόρεσε πάλι τα γυαλιά του και ετοιμάστηκε να κοντά σας. " αρχίσει την ανάγνωση, μέσα σε μια ατμόσφαιρα Όταν γύρισα, μου περίγραψαν με ακρίβεια τι έ-έντονης περιέργειας. ·

γινε όσο έλειπα. Στα διηγούμαι με κάθε λεπτομέρεια. - Θα παραλείψω τα τυmκά για την "καλή σου Περιεργάστηκαν για πολύ ώρα και με μεγάλη

υγεία" και διάφορα άλλα οικογενειακά. Κατ' ευθεί- προσοχή την «Πασκαλίν 11», όπως την ονόμασαν. αν στο θέμα! Ετοιμάστηκαν για τις δοκιμές. Ο Ζακ αναρωτήθηκε

... Θα θυμάσαι που σου έλεγα, πριν από πολλά φωναχτά με ποιόν αριθμό να αρχίσουν. Εκείνη τη χpόνια, ότι ο πολλαπλασιασμός ισοδυναμεί με μια στιγμή ο κούκος του μεγάλου ρολογιού του εργαστη­σειρά προσθέσεων με ίδιους προσθετέους και η ύψω- ρίου σήμανε την ώρα. ση σε δύναμη με μια σειρά πολλαπλασιασμών με ί- " Την ώρα να βάλουμε ", πετάχτηκε ο μικρός διους παράγοντες. Είχα από τότε την πρόθεση να Μπλαιζ. τελειοποιήσω την Πασκαλίν, ώστε να μπορεί να κάνει Ο Ζακ, που αγαπά πολύ τη Φυσική, τον κοίταξε και αυτές τις πράξεις. Σου ανακοινώνω λοιπόν σήμε- επιτιμητικά. Τι νόημα είχε η δύναμη ώρας ! Του έκα­ρα, με μεγάλη χαρά, ότι τα κατάφερα! Η Πασκαλίν ναν όμως το χατίρι . Έτσι σύντομα βρήκαν το απατέ­υπολογίζει πλέον και δυνάμεις ακεραίων! Όπως βέ- λεσμα αλλά και τον εκθέτη. Τότε ο Πιερ, που είναι ο βαια ξέρεις δεν ενδιαφέρομαι πια για τα εγκόσμια και "ταμίας" της τριάδας, παρατήρησε : δε νομίζω ότι θα αξιοποιήσω με κανέναν τρόπο αυτή "Προσέξατε ότι το αποτέλεσμα είναι το άθροι­μου την εφεύρεση. Συνέβη όμως ένα πολύ ενδιαφέρον σμα των οικονομιών μας; Προτείνω να κάνουμε μια περιστατικό που νομίζω ότι αξίζει να στο διηγηθώ, ακόμα δοκιμή , με βάση τα σκούδα μας. " είμαι σίγουρος πως θα σε ευχαριστήσει πολύ. Ο Μπλαιζ ρώτησε. 'Ό καθένας τα δικά του;"

Το απόγευμα που τελείωσα την τροποποίηση Αφού πρώτα του επισήμαναν ότι θα τους "συνέ-της Πασκαλίν, κάλεσα στο εργαστήριο τους τρεις εξα- φερε" να είχαν τη δύναμη του συνολικού ποσού και δέλφους σου και τους ανακοίνωσα το συγκλονιστικό μετά να τα μοίραζαν(!), συμφώνησαν και έκαναν νέο. Έμεινα αρκετά μαζί τους και μετά τους είπα πως από μια δοκιμή ο καθένας, με βάση το μερίδιό του. θα aποσυρθώ στο δωμάτιό μου. Τον τελευταίο καιρό Όταν τελείωσαν, ο Μπλαιζ αναστέναξε: η υγεία μου έχει επιδεινωθεί πάλι και χpειάζομαι αρ- «τι ωραία να είχαμε αυτά τα αποτελέσματα σε κετή ανάπαυση. Πριν φύγω τους εξήγησα πως δου- χpυσές λίβρες, κι 'ας είχα τις λιγότερες! » λεύει και τους προειδοποίησα για ένα μικρό τεχνικό Τότε ο δαιμόνιος Πιερ, αφού σκέφτηκε λίγο, του πρόβλημα. Τους είπα : είπε χαμογελώντας :

"Λοιπόν πρώτα γυρίζουμε αυτό το μικρό στρό- «Μη στενοχωριέσαι αδερφούλη! Θα σας μοίρα-φαλο και επιλέγουμε τον εκθέτη που, όταν ολοκλη- ζα τις δικές μου, αντιστρόφως ανάλογα με τις δικές ρώσω μια τεχνική ρύθμιση, θα εμφανίζεται στο πάνω σας και έτσι θα είχες ακριβώς τις ίδιες με το Ζακ!» μικρό πλαίσιο. Μετά επιλέγουμε τη βάση χpησιμο- Και συνέχισε. «Τι λέτε, σκαρώνουμε ένα προ­ποιώντας τα κατάλληλα βλήτρα και τη βλέπουμε στο βληματάκι για το θείο Μπλαιζ; Αν του πούμε για τη κεντρικό πλαίσιο. Σύρουμε προς τα έξω αυτόν το μο- μοιρασιά που θα έκανα, σίγουρα θα βρει μια σχέση χλό και σε λιγάκι, φεύγει η βάση από το κεντρικό για τις λίβρες μας, αλλά αν δεν ξέρει τον εκθέτη και πλαίσιο και στη θέση της εμφανίζεται το αποτέλεσμα! την ώρα, θα καταφέρει να βρει τα μερίδιά μας;» Δεν είναι θαυμάσιο;" Συμφώνησαν και επεξεργάστηκαν το ερώτημα.

Με παρακολουθούσαν με έξαψη. Ο μικρός, που Όταν επέστρεψα κοντά τους, μου περίγραψαν τι α­είναι η αδυναμία μου -αφού έχουμε και το ίδιο όνο- κριβώς έκαναν και τι είπαν, χωρίς όμως να μου απο-μα- είπε αμέσως : καλύψουν κανένα αριθμητικό στοιχείο, και μου έθε-

, Θείε μπορούμε να δοκιμάσουμε και μεις ; " σαν το εξής ερώτημα: "Ασφαλώς καλέ μου " απάντησα, " αλλά θα πα- "Ποιο ήταν το μεplδιο του ιtαθεvός τους, πριν

ραβλέψετε μια μικρή προσωρινή ατέλεια. Έχει πάθει και μετά την ύψω(Jη (J'tOV ά-yνωtJτο εκθέτή ; ,,

ΕΥΚΛΕΙΔΙΙΣ Β' λστ' τ.l/6

Page 9: Ευκλειδης Β 45

Το βήμα του Ευκλείδη

Δεν ήξερα την ώρα που σήμανε ο κούκος -δεν ήμουν εκεί εκείνη τη στιγμή- και πράγματι δε θυμό­μουν τον εκθέτη, ξέρεις πως δε λέω ποτέ ψέματα!

Αφού σκέφτηκα λίγο, έγραψα κάμποσες γραμμές σε ένα χαρτί και συμπλήρωσα έναν πίνακα, τους είπα, με σκόπιμα πολύ σοβαρό ύφος:

''Μπορώ να σας δώσω την απάντηση, ορίστε οι αριθμοί που ζητάτε, δεν μπορώ όμως να πω ότι έλυ­σα το πρόβλημα!"

Παραδέχτηκαν έκπληκτοι πως οι απαντήσεις μου ήταν σωστές, απορούσαν όμως για αυτό που εί­πα. Αλλά όταν τους εξήγησα τι σημαίνει για μένα λύση και απόδειξη, τι είναι αξίωμα, θεώρημα και

. εικασία, τότε κατάλαβαν γιατί η μαθηματική μου συνείδηση δε μου επέτρεπε να ισχυριστώ ότι έλυσα το πρόβλημα!

Κάναμε μεγάλη συζήτηση πάνω στο θέμα αυτό και δεν ξέρω αν τους έπεισα τελείως για την άποψή μου.

Λίγο αργότερα, πλησίαζε 8 η ώρα και είχε ήδη σκοτεινιάσει, πήραμε μαζί το δείπνο, με καληνύχτι­σαν και έφυγαν, έπρεπε να γυρίσουν στο σπίτι τους. Όμως την άλλη μέρα έμαθα ότι κοιμήθηκαν τα με­σάνυχτα, συζητούσαν ως αργά αυτά που τους είχα πει . . .

Περιμένω το επόμενο γράμμα σου και τα σχόλιά σου για το περιστατικό αυτό. Αν θέλεις προσπάθησε να βρεις και συ τους αριθμούς, πιστεύω ότι θα τα καταφέρεις.

Σε aσπάζομαι με αγάπη θείος Μπλαιζ. Οι �ο ανυπόμονοι από τους ακροατές είχαν

ήδη βγάλει τα σημειωματάρια και τα μολύβια τους. Σε λίγο η αίθουσα αντιβούιζε από εmφωvήματα και χαμογελαστά σχόλια. Όλοι αστειευόμενοι "καμάρω­ναν" γιατί είχαν λύσει το πρόβλημα που δεν "έλυσε" ο Πασκάλ! Και βέβαια κατάλαβαν τον υπαινιγμό του Προέδρου για την ευτυχή συγκυρία . . .

Ασυναίσθητα έγινε μια μικρή σιωπή. Στο μυα­λό τους ήρθε και πάλι το όνομα του Τανιγιάμα που συνδέθηκε, όπως και του άλλου μεγάλου συμπα­τριώτη τους, του Γκόρο Σιμούρα, με μια από τις mo συγκλονιστικές στιγμές της σύγχρονης ιστορίας των Μαθηματικών .. .

Σε λίγο, με πολύ ευχάριστη διάθεση, ξεκίνησαν μαζί με τον Πρόεδρο για το γραφικό ρεστωράν της Σιτέ, που θα έπαιρναν το δείπνο τους. Είχαν πολύ ωραίο θέμα για συζήτηση, στην όχθη του Σηκουά­να .. .

Β Ί'.ΞΒΠΙΣΒ Έστω Μ, Ζ και Π τα σκούδα των Μπλαιζ, Ζακ

και Πιέρ αντίστοιχα, πριν από την ύψωση στον ά­γνωστο εκθέτη ε. Έστω ακόμα ω (ω < 8) η ώρα που σήμανε ο κούκος.

Όλοι αυτοί οι αριθμοί είναι ακέραιοι (ο Πα­σκάλ μιλάει για δυνάμεις ακεραίων).

1. Ο Πιερ είχε πει στον Μπλαιζ: «Θα σας μοί­ραζα τις δικές μου, αντιστρόφως ανάλογα με τις δικές σας και έτσι θα είχες ακριβώς τις ίδιες με το Ζακ!»

Έστω ότι ο Πιερ θα δώσει Μπ στον Μπλαιζ και Ζπ στον Ζακ. Η "διανομή" δίνει:

Μπ I Ζπ= Ζ ε/ Μ ε => Μπ I Ζ ε = Ζπ I Με = λ => Ζπ = λ Με και Μπ =λ Ζ ε=> Ζπ - Μπ = λ(Με- zε).

Αλλά Ζπ+Ζε = Μπ+Με => Ζπ-Μπ = Με-zε . Άρα λ=1=> Μπ=Ζ \ Ζπ=Με και Με+z

ε=Ζπ+Μπ=Πε. Σύμφωνα όμως με το Τελευταίο θεώρημα

του Φερμά, αφού οι αριθμοί αυτοί είναι ακέραιοι, θα είναι ε = 2 . Είναι λοιπόν Μ2 + Ζ2 = Π2 και Π+Ζ+Μ = ω2•

2. Οι Μ, Ζ, Π αποτελούν Πυθαγόρεια τριάδα, άρα το άθροισμά τους είναι άρτιο.

Πράγματι, αφού Μ2 + Ζ2 = Π2 , αν ο Π (άρα και ο Π2) είναι περιττός, ο ένας (και μόνον) από τους Μ, Ζ θα είναι περιττός. Α ν δε ο Π είναι άρτιος, οι Μ, Ζ θα είναι ή και οι δύο άρτιοι ή και οι δύο περιττοί. Άρα είναι Μ+Ζ+Π = ω2 πάντα άρτιος, άρα και ο ω

είναι άρτιος. 3. Αφού είναι Μ, Ζ, Π ακέραιοι, Μ ο μικρότε­

ρος και Μ2+ Ζ2 = Π2, θα είναι Π>Ζ>Μ . Είναι Μ2=Π2- Ζ2 = (Π+Ζ)(Π-Ζ). Αν Π-Ζ = 1 ,

θα ήταν Μ2 = Π+Ζ και Μ2+Μ=Π+Ζ+Μ=ω2 => Μ(Μ+1) == ω2• ΑJ..λά ο ω δε' μπορεί να είναι ο γεω­μετρικός μέσος των Μ, Μ+ 1 , γιατί τότε θα ήταν Μ< ω< Μ+1 , πράγμα αδύνατο για ακεραίους Μ και ω. Άρα Π-Ζ >1 .

Αν ο Μ ήταν πρώτος, τότε: ή Μ2=1χΜ2 , ή Μ2=ΜχΜ. ΑJ..λά Π-Ζ '# 1 και (Π+Ζ) '* (Π-Ζ). Άρα η σχέση Μ2=(Π+Ζ)(Π-Ζ), θα ήταν αδύνατη, κατά συνέπεια ο Μ δεν είναι πρώτος, Ομοίως, επειδή εί­ναι Π-Μ> 1 και Ζ2= (Π+Μ)(Π-Μ), ούτε ο Ζ είναι πρώτος. Κατά συνέπεια οι ελάχιστες τιμές των Μ, Ζ, Π είναι: minM =4, minZ=6, και αφού Π-Ζ > 1 , minΠ=8.

Άρα minω2 = 18 => ω>4. Αφού δε ο ω είναι άρτιος και 4< ω< 8 , θα είναι ω = 6.

4. Αφού ω2=36 οπότε maxM < 36 I 3 = 12 και Μ όχι πρώτος, θα είναι 4::;; Μ :S:lO.

Καταρτίζουμε λοιπόν τον ακόλουθο πίνακα τι­μών για τη διαφορά Π-Ζ :

ΕΥΚΛΕΙΔΗΣ Β' λστ' τ.l/7

Page 10: Ευκλειδης Β 45

Το βήμα του Ευκλεiδη

6 36 10 100 26 100/26 Π - Ζ όχι ακέραιος 9 81 27 81 I 27 Π-Ζ=3,οπότε Π= 15,Ζ= 12 8 64 28 64/28 Π -Ζ όχι ακέραιος 7 49 29 49/29 Π -Ζ όχι ακέραως 6 36 30 36/30 Π -Ζ όχι ακέραως

Άρα είναι Μ = 9, Ζ = 12, Π = 15, Με= 81, zε = 144 και W = 225 . Σημείωση: Το Τελευταίο Θεώρημα του Φερμά αποδείχτηκε τελικά από τον Α. Γουάϊλς το 1995, γιαυτό και ο υπαινιγμός για την "ευτυχή σuyκυρία". Ο Γουάϊλς βασίστηκε στην ει­

κασία των Σιμούρα - Τανιγιάμα, που αποδείχτηκε ενδιάμεσα. Ο Πασκάλ γνώριζε τη διατύπωση της πρότασης από επιστολή του ίδιου του Φερμά, αλλά ως Εικα­

σία. Γιαυτό και δήλωσε (στην αφήγηση) ότι δεν μπορούσε να αποκαλέσει την εύρεση των αριθμών «μαθη­ματική λύση».

Υστερόγραφο: Ευχαριστώ θερμά τα μέλη της Συντακτικής Επιτροπής του "Ευκλείδη" κ.κ. Νίκο Αν­δρουλακάκη και Γιάννη Στρατή για την πολύ ουσιαστική συμβολή τους στην τελική διαμόρφωση της εξή­γησης.

Οι τρεις ηλικίες

Σ' αυτό το άρθρο θα σχολιάσω τρεις εργασίες που έστειλαν στο περιοδικό τρεις αναγνώστες που οι ηλικίες τους διαφέρουν πολύ, μεταξύ τους.

Η πρώτη γράφτηκε από μία μαθήτρια της ΒΆυκείου ενός ενιαίου Λυκείου της Αmκής , που έκρινα σκόπιμο να μην αναφέρω το όνομά της αλλά μόνο τα αρχικά, που είναι Α.Κ.

Η δεύτερη από τη νέα συνάδελφο μαθηματικό , Παναγιώτα Βάθη που ανήκει στη μαχόμενη παιδεία και η τρίτη από τον παλαίμαχο μαθηματικό ηλικίας 89 ετών αλλά πολύ νέο στις σκέψεις και στην κρί­ση,τον κ. Γ. Ιωαννίδη.

I. Η πρώτη επιστολή περιέχει μερικές σκέψεις και προβληματισμούς που έχει η αναγνώστριά μας σχετικά με ιδιότητες και ορισμούς των τρίγωνων και τετράγωνων αριθμών. Από τον τρόπο, όμως, που διατυπώνονται διαφαίνεται ότι πολύ λίγα πράγματα γνωρίζει γι· αυτούς τους αριθμούς.,

Ακόμη πρέπει να γνωρίζουν οι αναγνώστες μας, ότι τα γραπτά τους πρέπει να είναι καθαρο­γραμμένα και μη αμφισβητήσιμα, ως προς τον συμβολισμό τους και το νόημά τους. Μόνο έτσι θα μπορέσουμε να τους βοηθήσουμε, όπως έχουμε υ­ποσχεθεί.

Ομολογώ ότι με συγκίνησαν οι ανησυχίες της αναγνώστριάς μας και θέλω να την βοηθήσω, όχι λέyοντάς της κολακευτικά λόγια , αλλά σοβαρές και αυστηρές υποδείξεις, τις οποίες αν ακολουθήσει, θα υπερνικήσει τα εμπόδια που θα συναντήσει. κατά τη μελέτη των πολύγωνων αριθμών.

Υπάρχει ένα αξίωμα που έχει διατυπώσει ο Α­ριστοτέλης που ισχύει διαχpονικάκαι λέει:

((Πάσα διδασκαλία και πάσα μάθησις διανοητική,

Του Χρήστου Κηπουρού εκ προϋπαρχούσης γίνεται γνώσεως��.

Οι δυσκολίες που γράφεις ότι συναντάς, οφείλονται στην απουσία προϋπαρχουσών γνώσεων . Και εδώ θα σε βοηθήσουμε. Επειδή, όμως, δυνατόν να εν­διαφερθούν και άλλοι αναγνώστες, για τέτοιου εί­δους θέματα, θα δώσω την απαιτούμενη βιβλιογρα­φία:

Από το περιοδικό «Ευκλείδης Β')), τεύχος 4° του έτους 1990.σελ. 43, στο άρθρο «Ο γνώμων από την αρχαιότητα μέχρι σήμερα». Σ' αυτό το άρθρο περιγράφονται με απλό τρόπο όλοι οι πολύγωνοι αριθμοί και οι γνώμονές τους και ορίζονται, για πρώτη φορά σε παγκόσμια βιβλιογραφία, οι δίγω­νοι αριθμοί και οι γνώμονές τους. Ακόμη περιέχο­νται οι πίνακες Α και Β που περιέχουν αντίστοιχα, τους πολύγωνους αριθμούς και τους γνώμονές τους, συμπληρωμένοι για πρώτη φορά και με τους δίγω­νους αριθμούς. Κάθε πολύγωνος αριθμός με <<V>) πλευρές που βρίσκεται στην «D)) τάξη στον πίνακα Α, συμβολίζεται με Π� ενώ ο αντίστοιχος γνώμο-

νας, με Γ� . Υπάρχουν ακόμη και 5 «αλ γεβρικοb)

τύποι από τους οποίους, ο. πρώτος που συνδέει τον πολύγωνο αριθμό με το γνώμονά του, είναι μία ταυ­τότης που ονομάστηκε «Γνωμονικός τύπος του Ήρωνος)).

Πώς όμως θα αποκτήσετε αυτό το άρθρο; 1. Αν κάποιος καθηγητής σας ή φροντιστής

σας, που είναι μέλος της ΕΜΕ, μπορέσει να σας το εξασφαλίσει.

2. Μπορείτε να το βρείτε στο CD της ΕΜΕ που περιέχει τα πρακτικά του 1 r Πανελλήνιου Συνε­δρίου μαθηματικής παιδείας που έγινε το Νοέμβριο

ΕΥΚΛΕΙΔΙΙΣ Β' λστ' τ.Ι/8

Page 11: Ευκλειδης Β 45

Το μημα του Ευκλείδη

2000. ή και το βιβλίο των Πρακτικών 3. Επίσης, μπορείτε να αναζητήσετε τον 4° τό­

μο των απάντων του 'Ήρωνος που εκδόθηκε από την ΕΜΕ το έτος 1995 με τον τίτλο «Ονόματα γεωμε­

τρικών όρων- Γεωμετρικά>>. Εκεί περιέχεται βελ­τιωμένο,[ στο σχόλιο της σελίδας 45,] στο τέλος του έργου. Επειδή είναι εξαντλημένο και έχει σταλεί στα Γυμνάσια από το Υ π. Παιδείας, ας ψάξουν οι μαθη­ματικοί σας στη βιβλιοθήκη του Σχολείου σας. Ε­κτός των παραπάνω, μπορείτε να μελετήσετε για τους πολύγωνους αριθμούς και στα εξής έργα.

4. Ε. Σταμάτη. Ιστορία των ελληνικών μαθημα­τικών, σελίδες 77-84.

5. Thomas heath, Ιστορία των ελληνικών μαθη­ματικών. έκδοση Κ.Ε.ΕΠ.ΕΚ. (Κέντρο έρευνας επι­στήμης και εκπαίδευσης),Αθήνα 2000, μεταφρασμέ­νο ,τόμ-Ι, σελίδες 101-110. Τέλος, σαν άσκηση, σας δίνω δύο ερωτήσεις για να αρχίσετε:

α) Από ποιόν και πότε δόθηκαν οι όροι: ημίτο­νο και συνημίτονο, και

β) Γιατί οι αρχαίοι δεν μπόρεσαν να θεμελιώ­σουν και τους δίγωνους αριθμούς.

Αυτά για την πρώτη ηλικία, την ελπιδοφόρα. ΙΙ. Η δεύτερη εmστολή είναι απόλυτα, τεκμη­

ριωμένη και επιβεβαιώνει ότι γράφτηκε από μαθη­ματικό και μάλιστα πολύ συγκροτημένο. Αυτό φαί­νεται, αμέσως, αφού μας δίνει τέσσερις λύσεις του προβλήματος καθώς, επίσης, και τα ονόj.ι.ατα των περιοδικών που την περιέχουν.

ΠΙ Η Τρίτη εmστολή, που εξετάζει την ίδια άσκηση με επαγωγικό τρόπο, φανερώνει μαθηματι­κό της ανεπανάληπτης ρομαντικής εποχής. Δεν νο­μίζω ότι αυτή η λύση έχει δοθεί, στο παρελθόν, από άλλο μαθηματικό.

Στη συνέχεια δίνουμε τις λύσεις του προβλή­ματος που δημοσιεύτηκε στο προηγούμενο τεύχος 44/ 2002. και είχε προταθεί από τον Ακαδημαϊκό κ. Ν. Αρτεμιάδη.

Πρόβλημα: Δίνεται τετράγωνο ΑΒΓ Δ και σημείο Ε στο

εσωτερικό . του, τέτοιο ώ-στε: <1 ΕΓΒ =<1 ΕΒΓ = 15°. Αποδείξτε ότι το τρί­γωνο ΑΕΔ είναι ισόπλευρο.

Από τις 4 λύσεις που προτείνει η μαθηματικός Κα Παναγιώτα Δ. Βάθη, θα περιγράψουμε μόνο τις 2.

[Η λύση που ακολουθεί περιέχεται στο βιβλίο Geometry Reνisited των H.S.M. Coxeter και

S.L.Greitze, σελ. 158.]

Σχ.1 Σχ.2 lη.Λύση:. Επειδή τριγ.ΒΕΑ=τριγ.ΓΕΔ θα εί­

ναι:ΕΑ=ΕΔ και ΕΒ=ΕΓ Θεωρούμε: τριγ.ΓΗΔ=τριγΒΕΓ. (Σχ.l).Το τρί-

γωνο ΕΓΗ έχει: ΓΕ=ΓΗ και ΕΓΗ = 600 . 'Αρα είναι ισόπλευρο. Η ΔΗ τέμνει την ΕΓ στο Τ. Το τρίγωνο ΠΔ ... ... ... έχει:Γ+Δ=75°+15°=90° Άρα Τ=90°. Η Δ Τ είναι μεσοκάθετος του ΕΓ. 'Αρα:ΕΑ=ΕΔ=ΔΓ=ΑΔ. και το τρίγωνο· ΑΕΔ=

ισόπλευρο. 2η λύση [Από το περιοδικό: CRUXMAΠIEMAn­

CORUM 1985, σελ.57], Έγινε μικρή «επέμβαση» για καλλίτερη παρουσίαση.Βλ.(Σχ.2).

(α). Έστω χ>30°'τότε:

y<75°· <;::>ΓΔ<ΔΕ<:>ΑΔ<ΔΕ<;::> χ<3ο· άτοπον (β). Έστω χ<30° τότε: με όμοια διαδικασία

προκύπτει ότι χ>30°, άτοπον. ... 'Αρα χ=30° και 2χ= ΑΕΔ= 60° και το τρίγωνο

ΑΕΔ είναι ισόπλευρο. Δίνουμε, τώρα και την λύση του παλαίμαχου

μαθηματικού κ. Γ. Ιωαννίδη, ο οποίος με την πα­ρουσία του στον Ευκλείδη Β', μας γεμίζει με ελπίδα και πίστη για να συνεχίσουμε.

Της λύσης του προβλήματος προηγείται το πα­ρακάτω λήμμα.

ΒΔ

� Α Γ Σχ.1 Σχ.2

Λήμμα . 'Α ν σε ορθογώνιο τρίγωνο ΑΒΓ (με υποτεί­

νουσα ΒΓ=α) ... η γωνία Γ= 15° , τότε το επί την υποτείνουσα

, ΑΔ , , α υψος, , ειναι ισο με 4. Απόδειξη: Έστω Ε το μέσον της ΒΓ (Σχ.l). Τό­

τε θα είναι:

ΕΥΚΛΕΙΔΙΙΣ Β' λστ' τ.l/9

Page 12: Ευκλειδης Β 45

Το βήμα του Ευκλείδη

ΑΕ=; . 'Αν ΑΔ_l_ΒΓ ,επειδή, στο ορθογώνιο " τρίγωνο ΑΔΕ, η γωνία Β ΕΑ=30° θα έχουμε:

ΑΔ= Υ2 ΑΕ. 'Αρα ΑΔ=% . Λύση Έστω α η πλευρά του τετραγώνου ΑΒΓ Δ(Σχ.2) Φέρω την ΔΚ_l_ ΕΓ. Τότε θα είναι:

" "

ΚΔΓ = ΒΓ Ε= 15° [έχουν κάθετες πλευρές] Φέρω: ΚΑ l_ Γ Δ και ΕΘ l_ Γ Δ. Από το ορθογώ­

νιο τρίγωνο ΚΔΓ, σύμφωνα με το λήμμα, έχουμε: ΚΛ�Δ[ =% · Εξ' άλλου, είναι: ΕΘ= 8[ = �. Από τις δύο

τελευταίες προ!Cύπτει ότι ο λfyyoς ομοιότητας των δύο τριγώνω; ΚΛΓ και ΕΘΓ είναι:;�=� ·. 'Αρα το Κ είναι μέσον του ΕΓ, οπότε. η ΔΚ είναι μεσοκά-θετος του τριγώνου ΕΔΓτο οποίον, συνεπώς, είναι ισοσκελές. δηλ., ΔΕ=ΔΓ=α. 'Αρα το τρίγωνο ΑΕΔ είναι ισόπλευρο.

ΣΗΜΕΙΩΣΗ. Το περιοδικό Ευκλείδης Β' δεν θα δημοσιεύει πλέον, ασκήσεις , που έχουν δημοσι­ευτεί σε πρόσφατα τεύχη.

� ΑΞΙΟΠΕΡΙΕΡΓ Α- Λέξεις και Αριθμοί Ο αριθμός του θηρίου

Στην Αποκάλυψη Ιωάννου αναφέρεται ότι ο αριθμός χξστ, δηλαδή ο αριθμός 666, είναι το σύμβολο του αντίχριστου.

Πέρα από την όποια συμβολική του σημασία, για τα μαθηματικά ο αριθμός 666 είναι απλά άλλος ένας αριθμός, που ωστόσο έχει (όπως και πολλοί άλλοι αριθμοί) κάποιες aξιοσημείωτες ιδιότητες. Εδώ αναφέρουμε ενδεικτικά τέσσερις από αυτές.

Ο αριθμός 666 είναι ίσος με το άθροισμα των τετραγώνων των 7 αρχικών πρώτων αριθμών (θυ­μίζουμε ότι πρώτος αριθμός είναι κάθε θετικός ακέραιος διαφορετικός του 1 που διαιρείται μόνο από τον εαυτό του και τη μονάδα).

Δηλαδή ισχύει: 666 = 22 + 32 + 52 + 72 + 1 12 + 132 + 172

Ο αριθμός 666 μπορεί να γραφτεί ως το ά­θροισμα των ψηφίων του και των κύβων των ψη-φίων του, δηλαδή: 666 =(6+6+6)+(63 +63 +63)

Ο αριθμός 666 είναι ίσος με το άθροισμα των ψήφίων του 6663 = 295408296 και των 1Cύβων των ψηφίων του 6662 = 443556 , δηλαδή:

666 = (2 + 9 + 5 + 4 + ο+ 8 + 2 + 9 + 6) +

+(43 +43 +33 +53 +53 +63 ) Η τριάδα των αριθμών 6 · 6 · 6 , 666-6 · 6 και

666 έχει μια αξιοσημείωτη ιδιότητα που έχει και γεωμετρική σημασία. Μπορείτε να βρείτε ποια είναι η ιδιότητα αυτή;

του Ιωάννη Μαντά

ΠΕΙΡΑΜΑ τΙΚ ΟΣ ΠΡΟΣΔΙΟΡΙΣΜΟΣ ΤΟΥ π

1) Θεωρούμε ορθογώνιο κουτί γνωστών διαστάσεων α, β, γ άρα έχει όγκο ν= α · β · γ

2) Γεμίζουμε το κουτί με άμμο την οποία στη συνέχεια ζυγίζουμε. Έστω ότι το βάρος της είναι Β, συνεπώς σε όγκο ν = α · β · γ αναλογεί βάρος άμ-μου Β.

3) Μπήγουμε στην άμμο κυλινδρικό σωλήνα γνωστής ακτίνας ρ μέχρι να φτάσει στον πάτο του κουτιού. Άρα ο όγκος του κυλινδρικού σωλήνα είναι ν' = π · ρ2 • γ όπου γ το ύψος του σωλήνα.

u α β

EJ] α β

Ej α β

4) Με ένα κουτάλι αδειάζουμε την άμμο μέσα από το σωλήνα και τη ζυγίζουμε.

Έστω ότι το βάρος της είναι b. Άρα σε όγκο ν' αναλογεί βάρος άμμου b.

' ' b v 7ψ2 ·γ 7ψ2 � Τοτε θα ισχυειΒ=ν= αβy =�::::::>�

ΕΥΚΛΕΙΔΗΣ Β' λστ' τ.l/10

Page 13: Ευκλειδης Β 45

ΒΙΒΛΙΑ ΓΙΑ ΤΑ ΜΑθΗΜΑΤΙΚΑ ΤΟΥ ΕΝΙΑΙΟΥ ΛΥΚΕΙΟΥ

ΑΠΟ ΤΙΣ ΕΚΔΟΣΕΙΣ METAIXMIO Q"Ί

I Γ' ENIAIOY ΛΥΚΕΙΟΥ

Α' ENIAIOY ΛΥΚΕΙΟΥ I

ΗΛIΑΣ ΛΟΥΒΗΣ

ΜΑΘΗΜΑτΙΚΑ ΑΛΓΕΒΡΑ

r· ΕΝΙΑΙΟΥ ΛΥΚΕΙΟΥ θEfiKHI & ΤΕΧΝΟΛΟΓΙΚΗΙ Κ Α Τ Ε Υ Ο Υ Ν Σ Η Σ

ΗΛΙΑΣ ΛΟVΒΗΣ

ΜΑΘΗΜΑτΙΚΑ ΚΑΙ ΣΤΟΙΧΕΙΑ ΣΤΑτΙΣτΙΚΗΣ

Β' ENIAIOY ΛΥΚΕΙΟΥ I

Γ' ΕΝΙΑΙΟΥ λΥΚΕΙΟΥ ΓΕΝIΚΗΙ ΠΑΙΔΕΙΑΙ

Page 14: Ευκλειδης Β 45

• Ο Ακαδημαϊκός κος Νικόλαος Αρτεμιάδης είχε προτείνει στο προηγούμενο τεύχος ( αρ. 44) το παρακάτω θέμα: Αποδείξτε ότι υπάρχει μια απειρία ακεραί­

ων αριθμών η, τέτοιων ώστε καθένας από τους αριθμούς n, n + 1 , n + 2 να είναι άθροισμα τε­τραγώνων δύο ακεραίων αριθμών.

Λύση Εάν καθένας από τους η, η+ 1 , η+ 2 είναι ά­

θροισμα δύο τετραγώνων ακεραίων αριθμών, τότε καθένας από τους η2+2η , η2+2η+1 , η2+2η+2 είναι επίσης άθροισμα δύο τετραγώνων ακεραίων αριθμών.

Πράγματι: η2+2η+1=(η+1)2=(η+1)2+02 , η2+2η+2=(η+ 1 )2+ 1 =(η+ 1 )2+ 12

Κι ακόμη οι ισότητες: η=a2+b2 , η+2=c2+d2 συνεπάγονται την ισότητα:

η (η+2) = (a2+b2 ) (c2+d2)=(ad+bc)2+(ac-bd)2 όπου a, b, c, d = «ακέραιοι αριθμοί)).

Συνεπώς, για να βρούμε τη ζητούμενη απειρία των η, αρκεί να βρούμε μια κατάλληλη τριάδα ό­που η ε Ν* . Για παράδειγμα έχουμε: 8=22+22 , 9=32+02 ' 10=32+1 . • Ο συνάδελφος Α. Καλάκος από τα Κάτω Πα­

τήσια μας έστειλε την παρακάτω: Λύση Θεωρούμε την ακολουθία θετικών ακεραίων

αριθμών, που ορίζεται αναδρομικά ως εξής: β1=8 και βν+1 =β�+2βν Ι ν ε Ν• .

Προφανώς η (βy) είναι γνήσια αύξουσα (στην πραγματικότητα οι όροι της αυξάνονται αλματω­δώς: 8, 80, 6.650 . . . ).

Θα δείξουμε με τη μέθοδο της Τελείας Επα­γωγής ότι: «κάθε όρος της ακολουθίας έχει την ι­διότητα του ζητήματος».

Για ν= 1 έχουμε β1 =8 , οπότε παίρνουμε τους αριθμούς: 8, 9, 10, που είναι και οι τρεις αθροί­σματα δύο τελείων τετραγώνων ακεραίων αριθμών 8=22+22 ' 9=32+02 ' 10=32+ 12 .

Υποθέτουμε τώρα ότι για κάποιο θετικό ακέ­ραω η ισχύει: β0 =α2+γ2 , β" +1=x2+y2 και β" +2=z2+ω2 όπου α, γ, χ, y, z, ω φυσικοί αριθμοί.

Κι επειδή: βη+Ι+1=β�+2βν +1=(βν +1)2=(βν +1)2+02 ,

Ο Ευκλείδης ,

προτεινει . . . και • • •

Ευκλείδη Διόφαντο

βη+1 +2=(β0 +1 )2+12 και β0 ε Ν· , αρκεί να δείξουμε ότι: «ο Pn+J έχει την ιδιότητα του ζητήμα­τος». Έχουμε:

β η+ ι =β� +2β" =β" (β" +2)=(α2+γ2)(z2+ω2)= =(αz+γω)2+(αω-γz)2 , και συνεπώς ο βη+ι εί­

ναι άθροισμα δύο τετραγώνων ακεραίων αριθμών. Σχόλιο: Εύκολα βρίσκουμε έναν κλειστό τύπο

για τη (βν). Έχουμε: β ν+ ι + 1 =(β ν+ 1 )2 I ν ε Ν• Θέτουμε: αν= β ν+ 1 και παίρνουμε: αν+ ι = α� ,

α1 =9 . Με τη βοήθεια της Τελείας Επαγωγής βρί-σκουμε: αν =αΓ' =( 32 (·' =32• και τελικά:

βν=32• -1 l ν ε Ν• . • Ο συνάδελφος Σ. Σκοτίδης από την Καρδίτσα

μας έστειλε την παρακάτω: Λύση Έστω α άρτιος ακέραιος τέτοιος ώστε ο α2 + 1

να είναι σύνθετος. Για παράδειγμα: α=34λ+4 , λ ε Ν• .

[α2+1=(34λ+4)2+1=1 156λ 2+272λ+17= = 17 ( 68λ + 16λ + 1 )= «σύνθετος))] Έχουμε: α2 + 1 =pq όπου p, q = «περιττοί ακέ-

ραιοι)) με p>q> 1 ( 1 ) Κι επειδή:

pq=(�+pi}(�q _p;ι)=(p;q)2 -(Ρ;ι)2 (2)

Θέτουμε: χ= p;q , y= Ρ;ι κι έχουμε, λόγω

των (1) και (2), α2+1=χ2 -y2 ή α2+f=χ2-1 (3) Άρα μια λύση του προβλήματος είναι:

η=α2 +y2 . Πραγματικά: (3)

η+1=α2 +y2+1 = χ2-1+1=χ2+02 και η+2=χ2+12 . Για το συγκεκριμένο α έχουμε: η=(34λ+4)2+(34λ+8λ-8)2 Ι λ ε Ν• η+ 1 =(34λ 2 +8λ +9)2 +02 και η+ 2=(34λ 2 +8λ +9)2 + 12 Οι τιμές του η είναι προφανώς άπειρες.

• Ο συνάδελφος Δημήτρης Πιλάλας από τα Γιαννιτσά μας έστειλε την παρακάτω διαπραγ-

ΕΥΚΛΕΙΔΗΣ Β' λστ' τ.l/12

Page 15: Ευκλειδης Β 45

---------- Ο Ευκλεiδης προτείνει .. . Ευκλεiδη και . . . Διόφαντο ----------μάτευση του θέματος: Λύση Έστω η-ι , η, η+ ι η τριάδα των ζητουμένων

αριθμών. Θεωρώ την ειδική περiπτωση η=χ2+02 . Τότε η+ι=χ2+ι2 . Προκειμένου να αποδειχθεί η α­πεφία τριάδων που πληρούν τη ζητούμενη υπόθεση, αρκεί να δειχθεί ότι υπάρχουν άπεφοι αριθμοί της μορφής m=x 2 +y2 +ι , όπου m τετράγωνο φυσικού.

Παρατηρούμε ότι καθώς ο t διατρέχει το σύ­νολο των φυσικών αριθμών οι αριθμοί (2t2 )2 +(2t)2 , (2t2+ι)2 +02 , (2t2+ι)2 +ι2 αποτε­λούν τριάδα διαδοχικών ακεραίων αριθμών, που γράφονται ως άθροισμα δύο τελείων τετραγώνων.

Οι αριθμοί που βρήκαμε είναι προφανώς ά­πειροι. • Στο ίδιο τεύχος ο Ακαδημαϊκός κος Ν. Αρτε­

μιάδης είχε προτείνει και το παρακάτω θέμα: Δίνονται τρία διακεκριμένα σημεία Ρ, Q, R

με συντεταγμένες ακεραίους αριθμούς, τα οποία είναι σε κύκλο κέντρου Ο και ακτίνας r. Να α­ποδείξετε ότι: «δύο τουλάχιστο απ' τα σημεία αυτά έχουν απόσταση μεγαλύτερη ή ίση απ' τη � )).

Λύση Έστωσαν α, β, γ οι πλευρές του τριγώνου και

Κ το εμβαδόν αυτού. Τότε η ακτίνα r του περιγγε­γραμμένου κύκλου είναι (ως γνωστόν) ίση προς αβγ/4Κ. Αφού οι κορυφές του τριγώνου έχουν συ-ντεταγμέΝες ακεραίους αριθμούς, είναι Κ � i . Συνεπώς r � αβγ i , οπότε

maχ(α,β,γ) � (αβγ)Ιι3 � (2r)ΙΙ3 > rΙι3 • Μια λύση με απαγωγή σε άτοπο έδωσε ο συ­

νάδελφος Γιάννης Στρατής. • Ο συνάδελφος Α. Καλάκος από τα Κάτω Πα­

τήσια μας έστειλε μια αρκετά εκτενή λύση ό­που αποδεικνύει την εξής βοηθητική πρόταση: Η ύπαρξη τριών διαφορετικών σημείων πάνω σε κύκλο ακτίνας r συνεπάγεται ότι r �1/2.

• Ο συνάδελφος Αντώνης Ιωαννίδης από τη Λάρισα είχε προτείνει στο τεύχος 44, το παρα­κάτω θέμα: Να βρεθούν οι πραγματικές λύσεις της εξί-

1 σωσης: (1 - χγ + (x - y}2 + (y - z}2 + z2 = 4 (1).

Λύση Έστω ότι η (ι) έχει κάποια λύση (χ, y, z). Η εξίσωση (ι) γίνεται: 4 [(ι -x)2+(x-y}2+(y-z}2+z2] =ι ή

(ι2+ι2+ι2+ι2 } [(ι-x}2+(x-y}2+(y-z)2+z2] =ι (2) Καθώς γνωρίζουμε ισχύει η ταυτότητα

(Lagrange ):

(Σ •. :1 α�)(Σ •. :1 β� )-(Σ •. :1 αi βi )2 = Σ (αίβj --αjβi )2 i,je{l,2,3,4} Και συνεπώς για α1 =α2=α3=α4=ι και

β1 =ι-χ , β2=χ-y , β3 =y-z , β4=z παίρνουμε: 4 [(ι -x}2+(x-y}2+(y-z)2+z2 ] -ι= (2χ-y-ι }2 +(x+y-z-ι }2 +(χ+z-ι }2 + (2y-x-z}2 +(y+z-x}2 +(2z-y)2 Οπότε, λόγω της (ι) θα πρέπει το δεύτερο μέ­

λος της πιο πάνω ισότητας να γίνει Ο. Άρα θα πρέ­πει: 2χ-y=ι , x+y-z=ι , χ+z=ι , 2y-x-z=O , y+z-x=O , 2z-y=O .

Επιλύουμε το σύστημα και βρίσκουμε: 3 ι ι χ=4 , y=2 , z=4 .

Α ντίστροφα: Οι τιμές αυτές εύκολα διαπι­στώνεται ότι επαληθεύουν την εξίσωση (ι) • Λύσεις μας έστειλαν οι συνάδελφοι Α. Καλά­

κος απ' τα Κάτω Πατήσια, Ηλίας Κώστας απ' το Αλιβέρι, οι μαθητές Κώστας Φράγκος απ' το Νέο Ηράκλειο Αττικής, Δημήτρης Πι­λάλας απ' τα Γιαννιτσά και ο κος Λουκάς Χυτή ρης υπάλληλος ΕΛΤΑ.

• Ο συνάδελφος Αντώνης Κυριακόπουλος, μέλος της Συντακτικής Επιτροπής, είχε προ­τείνει στο τεύχος 44, το παρακάτω θέμα: Ν α βρείτε το μεγαλύτερο πραγματικό α­

ριθμό λ, για τον οποίο ισχύει λαβγ ::S; (α + β)2 + (α + γ)2 , για όλους τους πραγ­β + γ

ματικούς θετικούς αριθμούς α, β και γ. Λύση Για οποιουσδήποτε θετικούς αριθμούς α, β

και γ, η δοσμένη σχέση είναι ισοδύναμη με τη σχέση: β + γ [(α+β)2 + (α+γ)2 ] � λ (Ι) αβγ

Έχουμε:

:[(α+β)2 +(α+γ)2] �: (4αβ+ 4αγ) =

= 4 (β + γγ � 4 · 4βγ = ι 6 . aβγ βγ

Εξάλλου, το αριστερό μέλος της (Ι) για α = β = γ = _.!._ λαμβάνει την τιμή ι6. Συμπεραίνου-

2 με ότι, με α, β και γ θετικούς αριθμούς, το ι6 είναι η ελάχιστη τιμή του αριστερού μέλους της (1). Ά­ρα, η μεγαλύτερη τιμή του λ είναι λ= ι6. • Ο συνάδελφος Γιώργος Τασόπουλος έδωσε

μια λύση που στηρίζεται στην ανισότητα

ΕΥΚΛΕΙΔΗΣ Β' λστ' τ.l/13

Page 16: Ευκλειδης Β 45

---------- Ο Ευκλείδης προτείνει • . • Ευκλείδη και . . . Διόφαντο ----------

(α+ β )(β+ γ)( γ+ α) � 8αβγ για οποιουσδήπο­τε θετικούς αριθμούς.

• Ο συνaδελφος Α. Ιωαννίδης μας απέστεtλε μια λύση (αρκετά εκτενή) με παράγωγους.

• Ο μαθητής της Γ Λυκείου Κώστας Φράγκος από το Ηράκλειο Αττικής έστεtλε μια λύση όμοια κατά βάση με εκείνη που προτείνει ο συνάδελφος Α. Κυριακόπουλος.

• Ο συνάδελφος Α. Καλάκος από τα Κάτω Πα­τήσια μας έστεtλε μια λύση όμοια κατά βάση με εκέινη που προτείνει ο συνάδελφος Γ. Τασ­σόπουλος.

• Ο συνάδελφος Θανάσης Κυριακόπουλος, μέλος της Συντακτικής Εmτροπής, είχε προ­τείνει στο τεύχος 44, το παρακάτω θέμα: Να βρεθούν συναρτήσεις f : JR ---+ JR με

f (χ) > Ο , για κάθε χ e JR , τέτοιες ώστε:

f (χ) f (-χ) = α2 (1) όπου α > Ο (δοσμένος). Λύση Επειδή f (χ) > Ο , για κάθε χ Ε IR , ορίζεται η

συνάρτηση g (χ) = Lnf (χ) για όλους τους πραγμα­τικούς αριθμούς χ.

Ορίζουμε τώρα τη συνάρτηση h (χ)= Ln f(x) . Οπότε f (x)=αeh<x> . α

Κι επομένως f ( -x)=αeh<-x> , για κάθε χ Ε IR . Άρα: f (χ) f ( -x)=α2eh (x)+h(-x) ή λόγω της (Ι) :

eh (x)+h(-x) =e0 δηλαδή h (x)+h (-χ)=Ο ή h ( -x)=-h (χ) , για κάθε χ Ε IR .

Με άλλα λόγια: f (x)=αeh(x) όπου h μια κά­ποια περιττή συνάρτηση.

Αντίστροφα. Έστω f (x)=αeh(x) όπου h : IR ---+ IR μια περιττή συνάρτηση. Τότε: f (-x)=αe·h(x) και συνεπώς f (x) f(-χ)=α2 , για κάθε χ E IR .

Συμπεραίνουμε λοιπόν ότι η συνάρτηση της μορφής f (x)=αeh<x> όπου h : IR ---+ IR μια περιττή συνάρτηση ικανοποιεί τα επιτάγματα του προβλή­ματος .. • Ο συνάδελφος Σ. Σκοτίδας από την Καρδίτσα

μας έστεtλε την παρακάτω λύση: Λύση I. 'Εστω α= Ι . Τότε έχουμε: f (χ) f (-χ)= Ι , για κάθε χ Ε IR . Καθώς γνωρίζουμε: «κάθε συνάρτηση που εί-

ναι ορισμένη σ ' ένα σύνολο Α -συμμετρικό ως προς την αρχή Ο των αξόνων- μπορεί να αναλυθεί στο άθροισμα μιας περιττής και μιας άρτιας συνάρτη­σης» (άσκηση).

Συνεπώς: f(χ)=φ (χ)+σ (χ) I IR (Ι) όπου: φ (χ) = «περιττή συνάρτηση» και σ (χ) = «άρτια συνάρτηση». Επομένως: f (-χ)=φ (-χ)+σ (-χ)=-φ (χ)+σ (χ) Άρα: Ι =σ2 (χ)-φ2 (χ) ή σ2 (χ)=φ2 (χ)+ Ι (2)

Ισχυριζόμαστε ότι: σ (χ)=�φ2(χ)+ Ι (3) για κάθε χ Ε IR .

Πραγματικά αν υπήρχε κάποιος χ Ε IR τέ-τοιος ώστε σ (χ)=-�φ2 (χ)+Ι , τότε από την (Ι) θα παίρναμε: f (χ)=φ (χ)-�φ2 (χ)+Ι .

Οπότε θα είχαμε: O<f (χ)=φ (χ)-�φ2 (χ)+ Ι <φ (χ)-jφ (x)j ::; Ο : άτοπο

Η (Ι) λόγω της (3) γίνεται: h (χ)=φ (χ)+�φ2(χ)+Ι I IR

όπου φ (χ) μια περιττή συνάρτηση. Αντίστροφα. 'Εστω f(χ)=φ (χ)+�φ2 (χ)+Ι

όπου φ: IR ---+ IR μια κάποια περιττή συνάρτηση. Τότε: f (-χ)=φ (-χ)+�φ2(χ)+Ι=-φ (χ)+�φ2(χ)+Ι

Και συνεπώς: f(x) f(-χ)=(�φ2(χ)+Ι J -φ2(χ)=

=φ2 (χ)+ Ι -φ2 (χ)= 1 , για κάθε χ Ε IR . Έτσι για α=Ι η h (χ)=φ (χ)+�.-φ

-2(-χ)_+_Ι όπου

φ : IR ---+ IR τυχαία περιττή συνάρτηση είναι μία απ' τις συναρτήσεις που ζητάμε.

11. Έστω α -:;:. 1 . Η δοθείσα σχέση γίνεται: f (x) _ f (-x) =I (4) α α

Θέτουμε: g (χ)= f (χ) (5) α Οπότε η (4) γίνεται: g (χ) · g (-χ)=Ι , για κάθε

χ Ε IR και συνεπώς από το πρώτο μέρος έχουμε: g (χ)=φ (χ)+�φ2 (χ)+Ι I IR όπου φ (χ) τυχαία πε­ριττή συνάρτηση.

Τελικά μια από τις συναρτήσεις που ζητάμε είναι της μορφής: f (χ)=α (φ (χ)+�φ2 (χ)+ Ι ) I IR .

Όπου φ( χ) μια κάποια περιττή συνάρτηση. Σχόλιο της Σ.Ε. Με τους τρόπους που προ­

τείνονται βρέθηκαν 2 κλάσεις συναρτήσεων που ικανοποιούν τα επιτεύματα του προβλήματος. Υ­πάρχουν και άλλες;

• Ο συνάδελφος Σωτήρης Σκοτίδας από την Καρδίτσα είχε προτείνει στο τεύχος 44, το παρακάτω θέμα: Σε τρίγωνο ΑΒΓ είναι Β = 120° . Η διχοτό-

ΕΥΚΛΕΙΔΗΣ Β' λστ' τ.l/14

Page 17: Ευκλειδης Β 45

---------- Ο Ευκλείδης προτείνει . . . Ευκλείδη και . . . Διόφαντο ----------

μος της γωνίας Β τέμνει την ΑΓ στο Ρ. Η διχο­τόμος της εξωτερικής γωνίας Βi'Δ (το Δ είναι στην προέκταση της ΑΓ προς το μέρος του Γ) τέμνει την ευθεία ΑΒ στο Q. Τέλος η PQ τέμνει τη ΒΓ στο R. Ν α υπολογιστεί το μέτρο της γω-νίας PRA .

Λύση Εύκολα διαmστώνουμε ότι το Q είναι παρά­

κεντρο του τριγώνου ΡΒΓ. Άρα η PR είναι διχοτό-μος της γωνίας ΒΡΓ .

Επίσης παρατηρούμε ότι στο τρίγωνο ΑΒΡ, η PR είναι εξωτερική διχοτόμος. Συνεπώς το R είναι παράκεντρο του τριγώνου ΑΒΡ. Άρα η AR είναι διχοτόμος της γωνίας ΒΑΡ .

Q

Α Ρ Δ Έτσι έχουμε: 2φ=2θ+60° (από το τρίγωνο

ΑΒΡ). Οπότε: φ-θ=30° (1 ) Όμως: φ=θ+χ (από το τρίγωνο APR) Άρα: χ=φ-θ=30° . Δηλαδή: PRA=30° .

• Ο μαθητής Γ τάξης Κώστας Φράγκος από το Νέο Ηράκλειο Αττικής μας απέστεtλε την πα­ρακάτω:

Λύση Η ευθεία PRQ είναι διατέμνουσα του τριγώ­

νου ΑΒΓ. Συνεπώς από το Θεώρημα Μενελάου παίρνουμε: RB . PF . QA = 1 (1) RΓ ΡΑ QB

Εφαρμόζουμε τώρα το Θεώρημα των Διχοτό­μων για τις διχοτόμους ΒΡ και ΓQ του τριγώνου

, ΡΓ ΒΓ QA ΑΓ ΑΒΓ κι εχουμε: - = - - = - (2) ΡΑ ΒΑ ' QB ΒΓ Η (1), λόγω των (2) γίνεται:

RB ΒΓ Γ Α _ 1 , RB _ ΒΑ - · - ·- η - -RΓ ΒΑ ΓΒ RΓ ΒΓ RB ΒΑ Η (3) γίνεται: ---RB+RΓ ΒΑ+ΑΓ

RB ΒΓ · ΒΑ αγ Οπότε: ΑΒ+ΑΓ γ+β

(3)

(4)

Q Φέρνουμε τώρα την Ακ παράλληλη προς τη

ΒΡ και παρατηρούμε ότι: «τα τρίγωνα Γ ΑΚ και ΓΡΒ , , Ά ΒΡ ΒΓ ΒΓ Κ ειναι ομοια>>. ρα -=- = . αι ΑΚ ΚΓ ΚΒ+ΒΓ συνεπώς: ΒΡ ΒΓ . ΑΚ (5) ΚΒ+ΒΓ

Ωστόσο: κΒΑ=60° και ΑΚΒ=ΡΒΓ=60° Κι επομένως το τρίγωνο ΑΚΒ είναι ισόπλευ­

ρο. Δηλαδή: ΑΚ=ΚΒ=ΒΑ . Έτσι η (5) γίνεται: ΒΡ ΒΓ ·ΑΒ αγ (6) ΒΓ+ΑΒ α+γ

Επειδή η ΑΕ είναι διχοτόμος της εξωτερικής γωνίας Α του τριγώνου, συμπεραίνουμε ότι:

ΕΒ- ΒΓ · ΑΒ αγ (7) ΑΓ-ΑΒ β-γ Από τις (6), (7) παίρνουμε: ΕΒ α+γ (α+γ)(β+γ) (α+γ)(β+γ) ΒΡ β-γ (β-γ)(β+γ) βz -γz

(α+γ)(β+γ) (α+γ)(β+γ) β+γ γ2+α2 -2αγσυνΒ-γ2 α2+αγ α

AB C4J β+γ Από την άλλη έχουμε: - = _1_=-RB αγ α

Άρα:

β +γ ΕΒ ΑΒ , ΕΒ ΒΡ ΒΡ = BR η ΑΒ = BR

(8)

(9)

(10)

Κι επειδή: EBA=60°=PBR , τα τρίγωνα ΑΕΒ και PBR είναι όμοια.

'Ετσί BRP=EAB=EAR-BAR=90°- Α (1 1) 2 Άλλωστε έχουμε: ARr= 120°+ Α .

2 Οπότε: BRA=60°- Α . Συνεπώς: 2

� � � O I J Α Α ARP=BRP-BRA = (90°-2)-(60°-2)-30° • Ο συνάδελφος Α. Ιωαννίδης από τη Λάρισα

μας απέστεtλε μια λύση η οποία κατά βάση εί­ναι όμοια με εκείνη που προτείνει ο συνάδελ-

ΕΥΚΛΕΙΔΗΣ Β' λστ' τ.l/15

Page 18: Ευκλειδης Β 45

---------- Ο Ευκλεfδηc; προτεiνει • • • Ευκλεiδη και . . . Δι6φαντο ----------φος Σ. Σκοτίδας.

• Στην ίδια λοyική κινείται και η λύση του ιαιρί­ου Λουκά Χυτήρη που εργάζεται στα ΕΛΤΑ.

• Παρόμοια λύση μας απέστειλε και ο συνάδελ­φος Ηλίας Κώστας από το Αλιβέρι Ευβοίας.

• Ο συνάδελφος Σ. Σκοτίδας από την Καρδί­τσα είχε προτείνει στο τεύχος 44, το παρακά­τω θέμα: Σε τρίγωνο ABI' ισχύει: 81' = ΑΙ' +.!. ΑΒ • 2

Αν το σημείο Ρ χωρίζει την πλευρά ΑΒ σε )Jηο 3 : 1 , να αποδειχθεί ότι Ρ λr = 2ΓΡΑ •

Λύση Ας είναι Μ το μέσο της ΑΒ, τότε το Ρ είναι το

μέσο του ΜΒ. Προεκτείνουμε την ΑΓ προς το μέρος του Α

κατά ΑΗ=ΑΜ . Οπότε: Ρ Ar=2x . Φέρνουμε την Μτ παράλληλη προς τη ΓΡ.

Θα αποδείξουμε ότι: «η ΑΜ είναι εσωτερική διχο­τόμος της γωνίας Μ του τριγώνου ΙΠΜ». Οπότε θα είναι θ=χ .

Η

Β

Και συνεπώς θα έχουμε: Ρ Ar=2x=2θ=2rP Α . Για να αποδείξουμε τώρα τον ισχυρισμό που

θέσαμε, είναι αρκετό, σύμφωνα με το θεώρημα

δ ό δ ίξ . ΑΤ ΜΤ των ιχοτ μων, να απο ε ουμε on: ΑΗ = ΜΗ

Έχουμε: α=β+.!.γ . Οπότε: α2=β2+βγ+.!.γ2 • 2 4 Κι επειδή: α2 =β2 +γ2 -2βγσυνΑ , συμπεραίνουμε

ότι: συνΑ= 3γ-4β ( 1)

8β Από το τρίγωνο ΑΓΡ και το νόμο των συνημι­

τόνων έχουμε: ΓΡ2=ΑΓ2+ΑΡ2-2ΑΓ · ΑΡσυνΑ ή

2 2 (3 )2 3 ΓΡ =β + 4"γ -2β4γσυνΑ

(I) β(4β+ 3γ) Και συνεπώς: ΓΡ2 (2) 4

Τέλος:

ΜΗ2=ΑΜ2+ΑΗ2-2ΑΜ · ΑΗσυν (ΜΑΗ )*=

γ γ γ γ . ( )2 ( )2

2 + 2 +22 ' 2

συνΑ η 2 (I) 2 3 -4β 4β+3 ΜΗ2 =r_(l + συνΑ) =r_(l+-γ-)=γ2 --

γ (3) 2 2 8β 1 6β

2 1! . ΑΤ ΑΜ 4 2 Έχουμε τωρα: - = - = - = - .

ΑΓ ΑΡ 3_!! 3

Οπότε ΑΤ= 2β . 3 2β

Συνεπώς: ΑΤ = 3 = 4β

ΑΗ γ 3γ 2

β Όμοια:

ΜΤ = ΑΜ =

24 =

�.

ΓΡ ΑΡ 3 _!! 3 4

Οπότε: ΜΤ= 2(ΓΡ) 3

4

Συνεπώ : (. ΜΤ )2 = ΜΤ2 � 4(ΓΡ)2 · 16β � ς ΜΗ ΜΗ2 <J> 9γ2(4β+3γ) 4 β(4β+3γ) · 16β . 2

4 16β2 -(4β) 9γ2(4β+3γ) 9γ2 3γ

Άρα: ΜΤ =

4β ΜΗ 3γ

(4)

(5)

(6)

Από τις (4), (6) συμπεραίνουμε όn: ΑΤ _ΜΓ ΑΗ ΜΗ

• Ο μαθητής Γ τάξης Κώστας Φράγκος από το Ηράκλειο Αmκής απέστειλε μια λύση με το Νόμο των Συνημιτόνων:

• Όμοια σχεδόν λύση μας απέστειλε και ο μαθη­τής Γ Λυκείου Δημήτρης Πιλάλης από τα Γιαννιτσά.

• Ο συνάδελφος Α. Ιωαννίδης από τη Λάρισα απέστειλε μια λύση με το Νόμο των ημιτόνων και το θεώρημα Stewart.

• Ο συνάδελφος Α. Καλάκος από τα Κάτω Πα­τήσια μας απέστειλε μια ενδιαφέρουσα λύση που κινείται στην ίδια βασικά κατεύθυνση με εκείνη του Α. Ιωαννίδη.

• Ο κ. Λουκάς Χυτήρης που ασχολείται με τα Μαθημαnκά ερασιτεχνικά έστειλε την παρα­κάτω: Λύση

• Η ΜλΗ είναι παραπληρωματική της Α ΕΥΚΛΕΙΔΗΣ Β' λστ' τ.l/16

Page 19: Ευκλειδης Β 45

' ---------- Ο ΕυκUίδης προτείνει • . • Ευκλείδη και • • . Διόφαντο ----------

Καταρχάς αποδεικνύουμε το λήμμα: «Ας πάρουμε ένα τρίγωνο ΑΒΓ. Για να εί­

ναι Β=2Γ πρέπει και αρκεί: αγ = β2 - γ2 >>. Απόδειξη Ι. Ορθό. Ας πάρουμε ένα τρίγωνο ΑΒΓ για το

οποίο ισχύει: Β=2Γ . Φέρνουμε τη διχοτόμο της γωνίας Β του τριγώνου (σχήμα 1 ).

Από το θεώρημα των διχοτόμων έχουμε: ΔΑ =1 . Οπότε: ΔΑ γ ή ΔΑ= βγ (1) ΔΓ α ΔΑ+ΔΓ α+y α+γ

Β

ω Γ Α

Σχήμα 1 Κι επειδή τα τρίγωνα ΑΔΒ και ΑΒΓ είναι ό-

μοια (έχουν κοινή την Α και ΑΒΔ=ΒfΆ ) συμπε-ραίνουμε ότι: ΔΑ =Ι ή ΔΑ= γ

2 � βγ . γ β β α+γ

Συνεπώς: γ(α+γ)=β2 ή αγ=β2 -γ2 11. Αντίστροφο. Ας πάρουμε ένα τρίγωνο

ΑΒΓ, για το οποίο ισχύει: αγ=β 2 -γ2 • Από το νόμο

των ημιτόνων έχουμε: 2RημΑ · 2RημΓ=4R 2ημ 2B-4R 2ημ 2Γ ή ημΑημΓ=ημ 2Β-ημ 2Γ=ημ(Β+Γ) ημ(Β-Γ) Οπότε: ημΑημΓ ημΑημ(Β-Γ) Συνεπώς: ημΓ=ημ(Β-Γ) . Κι επομένως: Γ=Β-Γ ή Γ=Π-(Β-Γ) . Η τελευταία προφανώς αποκλείεται. Συνεπώς:

2Γ=Β . Με τη βοήθεια τώρα του Λήμματος και του

Θεωρήματος Διαμέσων αποδεικνύει τη πρόταση.

• Ο συνάδελφος Γιάννης Στρατής, μέλος της Συντακτικής Εmτροπής, είχε προτείνει στο τεύχος 44 το παρακάτω θέμα:

Για μια συνάρτηση f δεχόμαστε ότι: 2r<x> +f (χ) = 3χ + 5 , για κάθε χ e R .

Ν α αποδείξετε ότι: i) η συνάρτηση είναι γνήσια αύξουσα ii) η συνάρτηση είναι συνεχής iii) το σύνολο τιμών της είναι όλο το R iv) lim f (χ) = Ο

Ι-+Φ Χ

Λύση

ί) Έστω ότι υπάρχουν Χι , χ2 ε IR με Χι < Χ2 και f (χι ) � f (χ2 ) . Τότε 2f(x, ) � 2f(x2 ) και συνεπώς 2f(x, )+f(xι) � 2f (x2 >+f(x2 ) ή, λόγω της συνθήκης, 3χι +5 � 3χ2 +5 ή Χι � χ2 , άτοπο. Άρα για κάθε Χι , χ2 ε IR με Χι < χ2 ισχύει: f (χι ) < f (χ2 ) .

ίί) Έστω χ0 ε IR τυχαίος. Θα δείξουμε ότι: lim f(x)=f(x0) . Χ-+Χο

Για χ > χ0 έχουμε: 2f(x) -2f (xo ) +f (x)-f (χ�)=3 (Χ-Χο) Κι επειδή: 2r(x)_2r<xo >>o , συμπεραίνουμε ότι:

Ο < f (x)-f (χ0)<3 (χ-χ0) . Οπότε, σύμφωνα με το «Κριτήριο Παρεμβο­

λής)), παίρνουμε: lim. (f(x)-f(x0)} =0 ή lim. f(x)=f(x0) (1)

Χ-+Χο Χ-+Χο

Για χ < χ0 έχουμε: 2f (x) -2f(xo> +f (x)-f (Χο)=3 (Χ-Χο)

Κι επειδή: 2f(x) _2f(xo) <O , συμπεραίνουμε ότι: 3 (x-x0)<f (x)-f (χ0)<0.

Οπότε σύμφωνα με το «Κριτήριο Παρεμβο-λής)), παίρνουμε : ·

lim_ (f(x)-f(x0)}=0 ή lim_ f (x)=f(x0) (2) Χ-+Χο Χ_..Χο

Από τις (1) και (2) παίρνουμε: lim f(x)=f(x0)

Χ-+Χο

ίίί) Ισχυριζόμαστε ότι: <<Κάθε πραγματικός α­ριθμός y είναι τιμή που μπορεί να πάρει η συνάρτη­ση δηλαδή υπάρχει κάποιος πραγματικός χ τέτοιος ώστε: f(x)=y ».

Έστω ότι δεν ισχύει αυτό. Τότε για κάποιον πραγματικό αριθμό y ισχύει « f (χ) * y , για κάθε χ ε IR >>.

Η συνθήκη f (χ) < y δεν μπορεί να ισχύει για όλους τους πραγματικούς αριθμούς χ.

Πραγματικά, αν f (χ) < y , για κάθε χ ε IR , τότε 2r <x>+f(x) < 2Y+y=K , για κάθε χ ε IR .

Κι επομένως: 3χ+5<Κ ή χ< Κ;5 , για κάθε χ ε !R .

Η τελευταία δεν ισχύει πάντοτε. Για παρά-δ δ , Κ-4 ειγμα εν ισχuει για χ=-

3- .

Άρα υπάρχει κάποιος Χι ε IR τέτοιος ώστε: f (χι ) > Υ ( 1 )

Όμοια η συνθήκη f(x) > y δεν μπορεί να ι­σχύει για όλους τους πραγματικούς αριθμούς χ.

Πραγματικά, αν f (χ) > y , για κάθε χ ε IR τό-

ΕΥΚΛΕΙΔΗΣ Β' λστ' τ.l/17

Page 20: Ευκλειδης Β 45

---------- Ο Ευκλείδης προτείνει . . . Ευκλείδη και . . . Διόφαντο ----------

τε για ξ= Κ;6 θα είχαμε: f(ξ) > y , 2r(ξ)>2Υ . Κι επομένως: f (ξ)+2r<ξ> > y+2Y =Κ .

Οπότε: 3 Κ;6 +5>Κ ή Κ-6+5>Κ : άτοπο. Άρα υπάρχει χ2 Ε � τέτοιος ώστε:

f (x2) < y (2) Προφανώς χ2 <χ, ( αφού η f είναι γνήσια αύ­

ξουσα). Άρα f (x2 )<y<f (x1 ) . Κι επειδή η f είναι συνεχής, συμπεραίνουμε, σύμφωνα με το θεώρημα ενδιαμέσων τιμών, ότι υπάρχει κάποιος χ0 Ε (xz , χ, ) τέτοιος ώστε f (x0 ) = y : άτοπο.

ίν) Καταρχάς δείχνουμε το λήμμα: «Αν χ> 1 , τότε 2χ >χ )). Θεωρούμε τη συνάρτηση: σ (χ)=2χ -χ I [ 1 , +οο) και παρατηρούμε ότι σ' (x)=2xLn2-1 , σ"(x)=2x (Ln2)2 >0 .

Οπότε «η σ' I [ 1 , +οο) είναι γνήσια αύξουσω). και συνεπώς: «για χ> 1 ισχύει σ' (χ)>σ' (1)=2Ln2-1=Ln4-Lne>O )).

Επομένως: σ (χ)>σ (1)=1>0 , για κάθε χ με χ>1 . Άρα: 2χ >χ , για κάθε χ με χ>1 .

Επίσης ισχύει: « f (χ) > 1 , για κάθε χ με χ> 1 )). Πραγματικά, αν υπήρχε κάποιος χ με χ> 1 τέ­

τοιος ώστε: f (χ) :ς 1 , τότε θα είχαμε 2r<x> :ς 2 . Κι επομένως: 2r<x> +f (χ) :ς 3 . Συνεπώς θα εί-

χαμε: 3χ+5 :ι; 3 ή χ :ι; -� : άτοπο (είναι χ>1 ). Τώρα για κάθε χ με χ> 1 έχουμε: 3x+5=2f(x) +f (χ)<2 . 2f(x) ή 2f(x) > 3χ

2+5 .

3χ+5 Κι επομένως: f (x)>log2 -2- (3)

Όμως: . 3χ+5 lιm --=+οο και lim log2u=+oo . x�+ao 2 u---..+ao

Και συνεπώς, σύμφωνά με το θεώρημα «όριο σύν-. 3χ+5 θετης συνάρτησηφ, έχουμε: lιm log2 -2-=+οο . Χ-Η«>

Άρα, λόγω της (3), έχουμε: lim f (x)=+oo (4) Χ-Η«> Για τον υπολογισμό τώρα του ορίου έχουμε: lim f (χ) = lim 3f (χ) = lim 3f (χ) <:; Χ-Η«> Χ Χ-Η«> 3χ Χ-Η«> 2f(x) +f (χ)-5

3t (:) 3 lim -- = lim Ο t-+1«> 2 t +t-5 Η+«> 2 t .t'n2+ Ι ( σύμφωνα με τον κανόνα De L 'Hospital).

• Ο συνάδελφος Α. Ιωαννίδης από τη Λάρισα μας έστειλε την παρακάτω:

Λύση Πρώτα δείχνουμε ότι η συνάρτηση

φ (t)=t+21 I � είναι " 1 -1 " . Πραγματικά έχουμε: φ' (t)=l+21Ln2>0 , για κάθε χ Ε � . Άρα η φ είναι . γνήσια αύξουσα και συνεπώς εί­

ναι "1-1 " . Μια άμεση συνέπεια του γεγονότος αυ­τού είναι ότι: «Αν υπάρχει συνάρτηση f : � � � , που ικανοποιεί τη δοσμένη συνθήκη, είναι μοναδι­κή». Θεωρούμε τη συνάρτηση:

g (x)=�(2x +x-5) I � που είναι προφανώς παραγωγίσιμη με g' (x)=�(2xLn2+1) . Κι επειδή για κάθε χ Ε � ισχύει: g' (χ)>Ο , συμπεραίνουμε ότι: «η g είναι γνήσια αύξουσα σ' όλο το � με παράγωγο θετι­κή)). Άρα η g-1 είναι γνήσια αύξουσα και παραγω­γίσιμη στο πεδίο ορισμού της.

Βρίσκουμε το πεδίο ορισμού της g-1 , που ως γνωστόν είναι το σύνολο τιμών της g. Έχουμε: lim g (χ)=-οο και lim g (χ)=+οο . Κι επειδή η g

X---io--ct> Χ-++-ω είναι γνήσια αύξουσα και συνεχής, συμπεραίνουμε ότι: «το σύνολο τιμών της g είναι όλο το � )). Άρα το πf:δίο ορισμού της g-1 είναι το � .

Δείχνουμε τώρα ότι f (x)=g-1 (χ) I � . Από τον τύπο της g έχουμε g(g-1 (x))=�(2g·' <x> +g-' (x)-5) , για κάθε χ Ε � . Και συνεπώς για κάθε χ Ε � ι­σχύει: 2g·' <χ> +g-' (χ)=3χ+5 . Άρα, σύμφωνα με την παρατήρηση που κάναμε στην αρχή, ισχύει: f (x)=g·' (χ) . Κι αυτό αποδεικνύει τα τρία πρώτα ερωτήματα.

Για το όριο έχουμε: Ι. f (χ) - ι· g-ι (χ) - ι· g-' (g (χ)) ιm --- ιm --- ιm ..:............:...:....-� Χ--+-1«> Χ Χ-++«> Χ Χ--+-1«> g (Χ)

lim -........:..:x __

Χ--+-1«> ! (2χ +χ-5) 3

3χ (:) 3 lim = lim ---x--++«> 2χ +χ-5 Χ-++«> 2'fn2+ 1

(σύμφωνα με τον κανόνα De L'Hospital).

ο

• Στην ίδια βασικά κατεύθυνση κινείται και η λύση του συναδέλφου Α. Καλάκου από τα Κάτω Πατήσια.

• Ο συνάδελφος Επαμεινώνδας Ευθύμογλου από την Καβάλα είχε προτείνει στο τεύχος 44, το παρακάτω θέμα: Στις απέναντι πλευρές ΑΒ, Γ Δ ενός παραλ­

ληλογράμμου ΑΒΓ Δ θεωρούμε τα σημεία Ε, Ζ αντίστοιχα. Αν Η είναι το σημείο τομής των ΑΖ, ΕΔ και Σ το σημείο τομής των ΒΖ, ΕΓ να δει­χθεί ότι: «η ευθεία ΗΣ χωρίζει το παραλ-

ΕΥΚΛΕΙΔΗΣ Β' λστ' τ.l/18

Page 21: Ευκλειδης Β 45

---------- Ο Ευκλείδης προτείνει . • . Ευκλείδη και • . • Διόφαντο ----------

ληλόγραμμο ΑΒΓ Δ σε δυο ισοδύναμα τραπέ­ζια)).

Λύση Θα δείξουμε ότι: (ΑΒΛΚ)=(Γ ΔΚΛ)

�Ν Μ Δ Ζ Γ

Προφανώς είναι αρκετό να δείξουμε ότι ΒΛ=ΔΚ ή ΒΝ=ΔΜ (1 ) Από τα τρίγωνα ΖΑΒ και ΕΓ Δ με διατέμνου­

σες τις ευθείες ΝΣΗ, ΜΗΣ αντίστοιχα, σύμφωνα με το θεώρημα του Μενελάου, έχουμε:

ΝΑ . ΣΒ . ΗΖ =1 (2) ΝΒ ΣΖ ΗΑ και (3) Ωστόσο κατά το θεώρημα Θαλή ισχύει:

ΣΒ ΣΕ ΗΖ ΗΔ Ά δ , ' ΣΖ = ΣΓ και ΗΑ = ΗΕ . ρα ιαιρωντας κατα

μέλη τις (2) και (3), παίρνουμε: : = �� . Συνε-ΝΑ-ΝΒ _ ΜΓ-ΜΔ πώς: ---ΝΒ ΜΔ

Κι επομένως: ΑΒ = ΔΓ . Άρα: ΝΒ=ΜΔ . ΝΒ ΜΔ Άρα: «τα τρίγωνα ΒΑΝ, ΔΚΜ είναι ίσα (δύο

πλευρές ίσες και οι προσκείμενες γωνίες επίσης ίσες). Συνεπώς τα τραπέζια ΑΒΛΚ, Γ ΔΚΛ έχουν ίσα εμβαδά (ίσες βάσεις και ίσα ύψη). • Ο συνάδελφος Ηλίας Κώστας από το Αλιβέρι

Ευβοίας μας απέστειλε την παρακάτω: Λύση Θεωρούμε ως αρχή των αξόνων το σημείο Ο

τομής των διαγωνίων ΑΓ, ΒΔ του παραλληλο­γράμμου ΑΒΓ Δ, ως άξονα των χ την παράλληλη από το Ο προς την ΑΒ και ως άξονα των y την κάθετη από το Ο προς την ΑΒ.

Χωρίς βλάβη της γενικότητας δεχόμαστε ότι «η τετμημένη του Β είναι 1».

Θα προσπαθήσουμε να δείξουμε ότι τα ση-μεία Η, Ο, Σ είναι συνευθειακά.

Η εξίσωση της ευθείας ΔΕ είναι y-β 2β ' 2β - = - η y=β+-(χ-ε) . χ-ε 1+ε 1+ε

Όμοια η εξίσωση της ΑΖ είναι: y+β = 2β ή y=-β+ 2β (δ-χ) . χ-δ -γ-δ γ+δ

Επιλύουμε το σύστημα και βρίσκουμε: εδ-γ 1 -γ+δ-ε χ 1+γ+δ+ε ' y=βl+γ+δ+ε ( 1 )

Συνεπώς ο συντελεστής διεύθυνσης του δια-- 1 -γ+δ-ε νύσματος ΗΟ είναι λ =β (2) ΗΟ εδ-γ

Επίσης η εξίσωση της ευθείας ΒΖ είναι: y-β = -β-β ή y=β+ 2β (χ-1) . χ- 1 δ-1 1 -δ

Όπως η εξίσωση της ΕΓ είναι: y+β = β+β ή y=-β+ 2β (χ-γ) . χ-γ ε-γ ε-γ

Επιλύουμε το σύστημα και βρίσκουμε: εδ-γ β ( 1 -γ+δ-ε) χ ε-γ+δ-1 ' y= ε-γ+δ-1

Συνεπώς ο συντελεστής διεύθυνσης του δια-νύσματος ΟΣ είναι λ- β ( 1-γ+δ-ε) (3) ΟΣ εδ-γ

Από τις (2) και (3) συμπεραίνουμε ότι: λΗ0 =λ0Σ δηλαδή τα σημεία Η, Ο, Σ είναι συνευ-θειακά.

Εύκολα τώρα διαπιστώνουμε ότι «το εμβαδό του τραπεζίου ΑΚΛΒ είναι ίδιο με το εμβαδό του τριγώνου ΑΔΒ=� (ΑΒΓΔ) ». Οπότε τα τραπέζ�α ΑΚΛΒ και ΔΓ ΛΚ είναι ισεμβαδικά. • Την ίδια ιδέα, ότι δηλαδή η ευθεία ΗΣ διέρχε­

ται από το κέντρο του παραλληλογράμμου, εί­χε και ο μαθητής της Γ Λυκείου Κώστας Φράγκος από το Νέο Ηράκλειο. Η ενδιαφέ­ρουσα εργασία του κυρίου Κ.Φ. (με διανύ­σματα) παραλείπεται λόγω έλλειψης χώρου.

• Ο συνάδελφος Α. Ιωαννίδης από τη Λάρισα μας έστειλε μια λύση που βασίζεται στο θεώ­ρημα Μενελάου, που συνάδει με τη λύση του συναδέλφου Ε. Ευθύμογλου (παραπάνω).

• Ο συνάδελφος Επαμεινώνδας Ευθύμογλου από την Καβάλα είχε προτείνει στο τεύχος 44, το παρακάτω θέμα: θεωρούμε τρίγωνο ΑΒΓ και τον κύκλο του

Απολλωνίου, που αντιστοιχεί στην πλευρά ΒΓ. Αν Ρ τυχαίο σημείο του κύκλου και Ρι, Pz, Ρ3 οι προβολές του Ρ στις πλευρές ΒΓ, ΓΑ, ΑΒ αντί­στοιχα, να δειχθεί ότι: ΡιΡz=ΡιΡ3.

Λύση Το τετράπλευρο Ρ1ΓΡ2Ρ είναι προφα­νώς εγγράψιμο. Το κέντρο του περιγεγραμμένου κύκλου είναι το μέσο της ΓΡ ή με άλλα λόγια η ΡΓ είναι διάμετρος του κύκλου αυτού.

ΕΥΚΛΕΙΔΗΣ Β' λστ' τ.l/19

Page 22: Ευκλειδης Β 45

---------- Ο Ευκλείδης προτείνει . • . Ευκλείδη και . . . Διόφαντο ----------

Δ

Τώρα από το νόμο των ημιτόνων στο τρίγωνο ΡιΓΡ2 παίρνουμε: Ρ1Ρ2 =(ΓΡ)ημΓ=(ΓΡ)

2� (1)

όπου R η ακτίνα του περιγεγραμμένου κύ­κλου του τριγώνου ΑΒΓ.

Όμοια εργαζόμαστε και δείχνουμε: Ρ1Ρ3 =(ΒΡ)ημΒ=(ΒΡ)_1__ (2)

2R

Κ ιδ , ΒΡ ΒΑ γ , , ι επε η ΓΡ = Γ Α =β, συμπεραινουμε on:

β(ΒΡ)=γ(ΓΡ) (3) Από nς (1) και (2), λiJγω της (3), παίρνουμε: ΡιΡ2=ΡιΡ3.

• Στην ίδια λογική ιανείται και η λύση που μας έστειλε ο συνάδελφος Α. Ιωαννίδης από τη Λάρισα.

• Ό μαθητής της Γ Λυκείου Κώστας Φράγκος

από το Νέο Ηράκλειο μας έστειλε την παρα­κάτω λύση: Λύση

Φέρνουμε nς Ρ2Κ, Ρ3Λ και ΑΔ κάθετες στη ΒΓ. Τα ορθογώνια τρίγωνα Ρ1Ρ3Λ και ΡΒΡ3 είναι όμοια, αφού το τετράπλευρο Ρ1ΒΡ3Ρ είναι προφα­νώς εγγράψιμο ια επομένως τα μέτρα των γωνιών ΒΡ1Ρ3 και ΒΡΡ3 είναι ίσα.

(I)

Παρόμοια τα ορθογώνια τρίγωνα ΡΡ2Γ και ΡιΚΡ2 είναι όμοια, αφού το τετράπλευρο ΡΡ2ΓΡ1 είναι εγγράψιμο και συνεπώς τα μέτρα των γωνιών Ρ2ΡΓ και Ρ2Ρ1Γ είναι ίσα.

, Ρ Ρ Ρ Κ ΑΔ Άρα: .Ρ/ = p2 Γ = ΑΓ . Κι επομένως: 2

Ρ Ρ =(ΑΔ) ΡΓ ι 2 ΑΓ (11)

Ωστόσο, το Ρ ανήκει στον Απολλώνιο κύκλο

, ΡΒ ΑΒ , ΡΒ ΡΓ (lll) και συνεπως: -=- η -=-ΡΓ ΑΓ ΑΒ ΑΓ Από nς (1), (11), (111) συμπεραίνουμε όn: ΡιΡ2=ΡιΡ3.

• Ο κος Λουκάς Χυτήρης μας έστειλε μια λύση που αποδεικνύει ότι τα μέτρα των γωνιών Ρ1Ρ2Ρ3 και Ρ1Ρ3Ρ2 είναι ίσα.

ΠΡΟΤΕΙΝΟΜΕΝΑ ΘΕΜΑΤΑ Στο εξής θα διατηρούμε εννιαία αρίθμηση

για τα 4 τεύχη της χρονιάς. Παρακαλούμε τους συναδέλφους να στέλ­

νουν τις εργασίες τους καλογραμμένες και σε τρία(3) αντίγραφα. Σε κάθε αντίγραφο να υπάρ­χει όνομα διεύθυνση (ταχυδρομική - ηλεκτρο­νική - FAX) και όλα τα τηλέφωνα θα μας βοη­

θούσε αν οι εργασίες αποστέλλονταν με e-mail ή δισκέτα.

1. (το προτείνει ο Ακαδημαϊκός κος Νικόλαος Αρτεμιάδης)Έστω ABC ορθογώνιο τρίγωνο με κορυφή της ορθής γωνίας το Α. Πάνω στη BC κατασκευάζεται ισόπλευρο τρίγωνο DCD, που βρίσκεται στο εσωτερικό του τριγώνου ABC. Να αποδείξετε ότι και τα τρία μεγέθη ΑΒ, AC, AD δεν είναι δυνατό να είναι ρητοί αριθμοί.

2. (το προτείνει ο Ακαδημαϊκός κος Νικόλαος Αρτεμιάδης). Για τους πραγrατικούς αριθμούς a, b, c δεχόμαστε ότι: a�+ b• + c� + abc = 4 και a � Ο , b � Ο , c � Ο . Να αποδείξετε ότι:

) Ο � ab + bc + ca - abc � 2 3. (την προτείνει ο συνάδελφος Αντώνης

Κυριακόπουλος, μέλος της Συντακnκής Επιτρο­πής). Να βρείτε τους ακέραιους αριθμούς χ, y με χ > y , για τους οποίους ισχύει:

··::. x3+ y3 + xy (x + y - 8) = 8 (x1 + y1 + 1) 4. (την προτείνει ο συνάδελφος Α ντώνη·ς

Κυριακόπουλος). Θεωρούμε ένα ισόπλευρο τρί­γωνο ΑΒΓ, τον περιγεγραμμένο του κύκλο και τη διάμετρό του ΑΔ. Στις πλεύρές ΑΒ, ΑΓ παίρ­νουμε τα σημεία Ε, Ζ αντίστοιχα έτσι ώστε: ΕΖ = ΕΒ + ΖΓ . Να βρείτε το μέτρο της γωνίας ΕΔf, (σε μοίρες).

5. (την προτείνει ο μηχανολόγος Νικόλαος

Ζαφειρόπουλος από το Περιστέρι). Α ν Ο < χ < t, 1 Ο < Υ <2 και χ * y , να δείξετε ότι:

'-. ο < ι - 2χ1 - 2y1 + (xz - Yz J < ι . Το ζήτημα προέκυψε από την προσπάθεια επίλυσης

ενός προβλήματος Φυσικής από τη Θεωρία της Σχετικό­τητας, Η παράσταση δηλώνει το ·μέτρο της ταχύτητας ενός κινητού με μη μηδενική μάζα ηρεμίας, σε μονάδες όπου η ταχύτητα του φωτός είναι 1 (μονάδα). Κι έτσι

ΕΥΚΛΕΙΔΗΣ Β' λστ' τ.l/20

Page 23: Ευκλειδης Β 45

---------- Ο Ευκλείδης προτείνει . . . Ευκλείδη και . • . Διόφαντο ----------πρέπει να είναι μεταξύ του Ο (ακίνητο) και 1 (κινείται με την ταχύτητα του φωτός).

,J6. (την προτείνει ο συνάδελφος Ελευθέριος

Πρωτόπαπας). Για μια συνάρτηση f : [α, β] � R. δεχόμαστε ότι είναι δύο φορές παραγωγίσιμη και ότι f (α) = α , f (β) = β . Να αποδείξετε ότι αν fw(x) < Ο , για κάθε χ Ε [α, β] , τότε f (χ) > χ για κά�ε χ !!Ξ (α, β) )).

7. (την προτείνει ο συνάδελφος Γιώργος Τασσόπουλος, μέλος της Συντακτικής Επιτροπής). Αν για ένα σημείο Μ του ύψους ΑΔ οξυγωνίου τριγώνου ΑΒΓ με ΑΒ :ι: ΑΓ ισχύει η ισότητα ΑΒΜ = Ai'M , τότε να αποδείξετε ότι το σημείο Μ ε_tαι το ορθόκεντρο του τριγώνου ΑΒΓ.

8. (την προτείνει ο συνάδελφος Γιώργος

Τασσόπουλος). Δυο κύκλοι (Κ, R) και (Λ, r) εφάπτονται εξωτερικά στο Α. Θεωρούμε δύο κάθετες τέμνουσες ΒΑΓ, ΔΑΕ αυτών μεταβλη­τές. Να αποδείξετε ότι το άθροισμα ΑΒ . ΑΓ + ΑΔ · ΑΕ είναι σταθερό.

,.ι 9. (την προτείνει ο κος Λουκάς Χυτή ρης, ε­ρασιτέχνης Μαθηματικός). Δίνεται κύκλος κέ­ντρου Ο και δύο διάμετροι ΑΓ, ΒΔ τέτοιες ώστε το μέτρο της γωνίας ΒΟΓ είναι 60°. Στο ελάσ­σον τόξο ΒΓ παίρνουμε ένα τυχαίο σημείο Μ και φέρνουμε τις ΜΑ, ΜΒ, ΜΓ, ΜΔ. Να δείξετε ότι: :�:� = 2 + J3 .

10. (την προτείνει ο συνάδελφος Αθα,•άσιο.; Καλάκος από τα Κάτω Πατήσια). Ας πάρουμε δυο συγκεκριμένους πραγματικούς αριθμούς θ, ρ με Ο < θ < π και ρ > Ο . Θεωρούμε όλα τα τρί-

γωνα ABC με Α = θ και περίμετρο Ρ. Να βρείτε το σύνολο τιμών της πλευράς ΑΒ.

Το πρόβλημα προήλθε από μια ιδέα του Καθηγητή Murray Klamkin στο περιοδικό Crux Mathematicorum, Απρjιος 2001.

· 11 . (την προτείνει ο συνάδελφος Αθανάσιος Καλάκος). Να βρεθεί η ικανή και αναγκαία {x + y +z = α } συνθήκη ώστε το σύστημα 2 2 2 να

χ + y + z = β έχει μία τουλάχιστον πραγματική λύση. (Οι α, β είναι συγκεκριμένοι πραγματικοί αριθμοί - πα­ρά�ροι).

12. (την προτείνει ο συνάδελφος Γιάννης Στρατής, μέλος της Συντακτικής Επιτροπής). Να βρείτε συνεχή συνάρτηση f : [Ο, π] � R. τέτοια

π 13 ώστε J[r (χ) [(4συνχ +6ημχ) - f (x)]J dx ;?: 2π . ο

Το θέμα ξαναπροτείνεται γιατί ο «δαίμων του τυ­πογραφείου>> το είχε παραπ9ιήσει στο προηγούμενο.

13. (το προτείνει ο μαθητής της Γ Λυκείου Δημήτρης Πιλάλας). Να εξετασθεί αν υπάρχει ν Ε Ν με ν > 3 τέτοιος ώστε καθένας από τους αριθμούς κ + 1 , κ + 2 , . • • , κ + ν με κ τυχαίο ακέραιο αριθμό μπορεί να γράφεται ως άθροι­σμα δύο τετραγώνων ακεραίων αριθμών.

Χρήστος Μπαλόγλου

Σ κ ό ρ π ι ε ς σ τ α γ ό ν ε ς Γ ε ω μ ε τ ρ ί α ς

Κεντρική Διάθεση: Μαθηματική Βιβλιοθήκη - Χ. Βαφειάδης Διαλέττη 2 7 • 5462 1 Θεσσαλονίκη τηλ. : 03 1 0 263 1 63 • FAX : 03 1 0 240595

Page 24: Ευκλειδης Β 45

ΗΟΜΟ MATHEMAτiCUS

ΗΟΜΟ MA THBMA TICUS Η Homo Mathematicus είναι μια στήλη στο περιοδικό μας, με σκοπό την ανταλλαγή απόψεων και την ανά­

πτυξη προβληματισμού πάνω στα εξής θέματα: 1) τι είναι τα Μαθηματικά, 2) Πρέπει ή όχι να διδάσκονται, 3) Ποιοι είναι οι κλάδοι των Μαθηματικών και ποιο το αντικείμενο του καθενός, 4) Ποιες είναι οι εφαρμο ­

γές τους, 5) Ποιες εmστήμες ή κλάδοι εmστημών απαιτούν καλή γνώση των Μαθηματικών για να μπορέσει κάποιος να τους σπουδάσει. Κανόνας: Η στήλη Homo Mathematicus απευθύνεται μόνο σε μαθητές.

Ι. ΠΟΔΟΣΦΑΙΡΟ ΚΑΙ ΜΑΘΗΜΑΤΙΚΑ [ Νο 1 ] Προλεγόμενα. Αν σας 'πω πως για να χτυπήσετε σφαίρου.

Τού Γιάννη Κερασαρίδη

ένα επιτυχημένο φάουλ χρειάζεται να ξέρετε Μα- Το πρόβλημα του φάουλ. Βρισκόμαστε στη δια­θηματικά , θα με χαρακτηρίσετε φαντασιόπληκτο. δικασία εκτέλεσης φάουλ. Οι αμυνόμενοι σχημα­Εγώ το εmχείρησα και .. . απότυχα. Ίσως φταίει η τίζουν τείχος με τα σώματά τους ανάμεσα στον ηλικία μου ίσως, όμως, και κάτι πολύ σημαντικό «εκτελεστή)) και την εστία. Η μπάλα πρέπει να που δεν έλαβα υπ' όψη μου. Ενδέχεται κάποιος περάσει πάνω από τους παίκτες. Οι αμυνόμενοi από σας να μου το επισημάνει. Σ' αυτό το τεύχος, έχουν ύψος y=1,8 m και σχηματίζουν τείχος σε λiΥyω έλλειψης χώρου, θα ασχοληθούμε περιλη- απόσταση χ=9,1 5 m από τον σουτέρ. Αυτός σου­πnκά με την εκτέλεση ενός φάουλ στο ποδόσφαι- τάρει τη μπάλα με ταχύτητα υ0=35 m/s. Σε πόση ρο και υποσχόμαστε πως σε κάποιο άλλο τεύχος απόσταση από το τείχος είναι αδύνατο να προ­θα αναπτύξουμε λεπτομερώς σημεία του ποδο- σγειωθεί η μπάλα;

Η λύση του προβλήματος. Το ζητούμενο είναι να βρούμε δύο γωνίες για βρίσκουμε τα σημεία όπου αυτές οι δύο τροχιές

τις οποίες ο «εκτελεστής)) μπορεί να σουτάρει τη συναντούν το έδαφος. Τα σημεία προσγείωσης της μπάλα με αρχική ταχύτητα υ0=35 m/s και να τη μπάλας απέχουν από το σημείο εκτέλεσης του ε-στείλει στο σημείο με συντεταγμένες (9, 1 5 m, 1 ,8 λεύθερου κτυπήματος χΊ=10, 17m και x'2=61 ,32m m). Εφόσον υπολογίσουμε αυτές τις δύο γωνίες, αντίστοιχα. Συμπεραίνουμε, λοιπόν, ότι η περιοχή μπορούμε να βρούμε τις αντίστοιχες τιμές που θα που δεν μπορεί να προσγειωθεί η μπάλα, αρχίζει φανερώνουν τη θέση όπου θα προσγειωθεί η μπά- ακριβώς πίσω από το τείχος και εκτείνεται μέχρις λα. Γράφουμε, λοιπόν, την εξίσωση της τροχιάς: αποστάσεως 1 ,02 m απ' αυτό.

gx2 2 gx2 y= -- εφ θ+χεφθ - - . 2υδ 2υδ

Εάν εισαγάγουμε τις τιμές των χ, y (x=9, 15m, y=1,8m) και θέσουμε υ0=35 m/s, προκύπτει μια δευτεροβάθμια εξίσωση με άγνωστο το εφθ, απ' την οποία παίρνουμε: εφθ1=27,078 και εφθ2=0,235, α1t' τις οποίες, τελικά έχουμε θι=87,89° και θ2=13,23°. Χρησιμοποιώντας την εξίσωση του βεληνεκούς,

2υ2 υ2 Xmax= _ο συνθημθ = -2. ημ2θ, g g

9,15m 10,17 m

61 ,32 m

110• ΕΝΑΣ ΜΑΘΗΜΑ ΤΙΚ ΟΣ ΑΠΑΝΤΑ ΣΤΗ ΧΡΙΣΤΙΝΑ Προλεγόμενα. Στο προηγούμενο τεύχος, στη στήλη ''Homo mathematicus", δημοσιεύσαμε μια ιδέα της Χριστίνας Πατσιούρα (μαθήτριας της Δ' τάξης του 1300 Δημοτικού Σχολείου Νέας Ιωνίας), σχετικά με μια <<νέα δοκιμή της πρόσθεσης)) και καταλήγαμε με την έκκληση: « Οι Αριθμοθεωρητικοί μας, ελπί­ζουμε, να τιμήσουν αυτό το παιδί, δίνοντάς μας μια θεωρητική επένδυση της ιδέας της Χριστίνας, διαφο­ρετικά, θα aπογοητεύσουμε την υποψήφια Υπατία μας)) Περί ευαισθησίας. Δεν πέρασε αρκετός καιρός και, ένας σημαντικός φίλος μαθηματικός, ο Ανδρέας Αρβανιτογεώργος, ανταποκρίθηκε στην έκκληση της στήλης και μας έστειλε μια θεωρητική επένδυση. Η στήλη και, μέσω αυτής, η Χριστίνα, τον ευχαριστούν για την παιδαγωγική και μαθηματική του ευαισθη­σία

ΕΥΚΛΕΙΔΗΣ Β' λστ' τ.l/22

Page 25: Ευκλειδης Β 45

ΗΟΜΟ MATHEMAτiCUS Το κείμενο της απάντησης και ένα πρόβλημα για σας

«Στο προηγούμενο τεύχος του Ευκλείδη Β' (44) η μαθήτρια Χριστίνα Πατσιούρα μας είχε γράψει για μια δοκιμή της πρόσθεσης, παρόμοια με αυτή του πολλαπλασιασμού, ρωτώντας μας πώς εξηγείται. Η εξήγησή της βασίζεται στις ισοτιμίες από τη θεωρία αριθμών (βλ. και παλαιότερο άρθρο μου για την εξήγηση της δοκιμής του πολλαπλα­σιασμού, Α. Αρβανιτογεώργος: "Η περίεργη δοκι­μή του πολλαπλασιασμού και η εξήγησή της", Ευκλείδης Β' κζ . 1/52.).

Θα δώσουμε την εξήγηση με ένα απλό παρά­δειγμα, ώστε να γίνει κατανοητό από τη φίλη μας.

Θεωρούμε την πρόσθεση 35782 + 7813= 43595. Η Χριστίνα είχε παρατηρήσει το εξής:

3 + 5 + 7 + 8 +2 = 25 � 7 7 + 8 + 1 + 3 = 19

7 +10 = 17 � 8 4 + 3 + 5 + 9 + 5 = 26 � 8

Η ισότητα των δύο 8 δεν είναι τυχαία. Η εξή­γησή της είναι η εξής:

Έστω Μ = 35782 και Ν= 7813 . Θεωρούμε τη δεκαδική αναπαράσταση των αριθμών αυτών

Μ= 3 . 104+5 . 103+7 . 102+8 . 101+2. 10°

Ν=7 . 103+8.102+1 . 1 01+3 . 10° και θέτουμε

S�3+5+7+8+2=25 και S�7+8+1+3=19 Παρατηρούμε ότι 35782 - 25 = 35757 = πολ­

λαπλάσιο του 9 = 9κ. Παρόμοια 7813 - 19 = 9λ. Γενικά ισχύει ότι Μ- S�πολλ9, το οποίο συμβολί­ζεται στη θεωρία αριθμών ως M=SM(mod9) (για την απόδειξη βλέπε υποσημείωση 1 ). Σε αυτή την περίπτωση οι αριθμοί Μ και SM ονομάζονται ισότι­μοι modulo 9.

Ισχύουν τώρα οι σχέσεις SM+N = (SM+SN)(mod9) και SMN Ξ SMSN(mod9) Η πρώτη σχέση δικαιολογεί τη δοκιμή της φί­

λης μας και η δεύτερη τη δοκιμή του πολλαπλα­σιασμού με τον γνωστό σταυρό.

Έχουμε λοιπόν για το παράδειγμά μας, SM+N=S43595=26, SM+S�4 και 26 - 44 = - 18 = πολλ 9, όπως αναμένεται. Με άλλα λόγια, οι α­ριθμοί 26 και 44 είναι ισότιμοι με το 8 modulo 9 όπως παρατηρεί η φίλη μας!

Οι ισοτιμίες έχουν και άλλες ενδιαφέρουσες εφαρμογές, όπως για παράδειγμα δίνουν απάντηση σε ερωτήματα του τύπου σε"ποια ημέρα της εβδο­

μάδας θα πέσει η 18 Απριλίου του 2008; " >>

Η Σύνταξη της στήλης περιμένει, από τους α­ναγνώστες, απάντηση στο πρόβλημα αυτό.

Ilp . ΕΝΑ ΤΗΛΕΦΩΝΗΜΑ ΑΠΟ ΦΙΛΟ ΤΗΣ ΣΤΗΛΗΣ

Το ιστορικό ... Μια καυτή 'μέρα του Ιουλίου, ένας πολύ καλός φίλος της στήλης, με πήρε τηλέφωνο και με έψεξε γιατί aπασχολώ τη στήλη με «ιδέες παιδιών του Δημοτικού Σχολείου . . . ».

... και η απάντηση. Τον άκουσα με πολύ προσοχή και του υπενθύμισα ορισμένα πράγματα , όπως:

α) τα Μαθηματικά, απ' την ίδια τους τη γενε­σιουργό αιτία, μελετούν τις μορφές του χώρου και τις ποσοτικές σχέσεις στον πραγματικό κόσμο. Κατά συνέπεια οι τέσσερις πράξεις της Αριθμητι­κής (μαζί κι η απλοϊκή Γεωμετρία) αποτελούν το θεμέλιο λίθο τους και μακάρι να διατηρούταν η διδασκαλία της Πρακτικής Αριθμητικής. Του υ­πενθύμισα τα ποσοστά αποτυχίας στην Αριθμητική σε διάφορους διαγωνισμούς, σε βαθμό που να παίρνει διαστάσεις κοινωνικού φαινομένου.

το συμπέρασμα πως η ενθάρρυνση ή η αποθάρ­ρυνση ενός νέου απ' τους «καθ' ύλην αρμόδιους» ανοίγει ή κλείνει δρόμους. Μια (ευτυχώς μικρή), μερίδα δασκάλων πιστεύει πως "τα μυαλά των νέ­ων εύκολα παίρνουν αέρα" και, άρα, ο δάσκαλος θα πρέπει να τα προσγειώσει ψαλιδίζοντας τα φτε­ρά τους. Αλλά ας μου επιτρέψουν αυτοί οι δάσκα­λοι να τους aντιτείνω την άποψη πως θα έπρεπε να aνησυχούσαμε αν τα νέα παιδιά δεν είχαν αυτή τη ροπή. Είναι φανερό πως η αποκατάσταση της "ε­παφής με την πραγματικότητα", ενός νέου παιδιού, δεν επιτυγχάνεται από έναν δάσκαλο " άψυχο", στο ρόλο "κρεατομηχανής". Επιτυγχάνεται από ένα δάσκαλο που αγαπά τους ανθρώπους και πριν απ' όλα τα νέα παιδιά που αποτελούν εγγύηση ότι η ζωή θα συνεχισθεί.

γ) Στην επικεφαλίδα της στήλης αναφέρε-β) κάποτε, η στήλη αυτή, θα αφιερώσει χώρο ται ρητά: «Κανόνας: Η στήλη Homo

όπου θα παραθέτουμε aυτοβιογραφικές μαρτυρίες Mathematicus απευθύνεται μόνο σε μαθη­σημαντικών ανθρώπων, απ' τις οποίες προκύπτει τές »

ΕΥΚΛΕΙΔΗΣ Β' λστ' τ.l/23

Page 26: Ευκλειδης Β 45

Μαθηματικοί Διαγωνισμοί - Μαθητικές Ολυμπιάδες

Επιμέλεια : Επιτροπή Διαγωνισμών

• 43η ΔΙΕΘΝΗΣ ΜΑΘΗΜΑΤΙΚΗ ΟΛΥΜΠΙΑΔΑ ΓΛΑΣΚΩΒΗ Μ. ΒΡΕΤΑΝΙΑΣ, 19-30 ΙΟΥΛΙΟΥ 2002

Π Ρ Ο Β Λ Η Μ Α ι 'Εστω n θετικός ακέραιος και Τ το σ6νολο

των σημείων (x,y) του επιπέδου, όπου χ και y είναι μη αρνητικοί ακέραιοι αριθμοί με χ + y < n . Κάθε σημείο του συνόλου Τ χρωματί-ζεται κόκκινο ή μπλε. Αν ένα σημείο (x,y) είναι κόκκινο, τότε θα είναι κόκκινα όλα τα σημεία (χ',y�του Τ με χ' :S χ και y' :S y . Κάθε σ6νολο που αποτελείται από n μπλε σημεία με διαφορε­τικές τετμημένες χ ονομάζεται σ6νολο τ6που Χ και κάθε σ6νολο που αποτελείται από n μπλε σημεία με διαφορετικές τεταγμένες y ονομάζε­ται σ6νολο τ6που Υ. Να αποδείξετε ότι ο αριθ­μός των συνόλων τ6που Χ είναι ίσος με τον α­ριθμό των συνόλων τύπου Υ.

Λύση ι Έστω ότι ο αριθμός των μπλε σημείων με τε­

τμημένη i είναι ai και έστω ακόμη ότι ο αριθμός των μπλε σημείων με τεταγμένη i είναι bi. Σύμφω­να με τη βασική αρχή της απαρίθμησης, ο αριθμός των σuνό'λιnν τύπου Χ θα είναι a0a1 . . . an-ι , ενώ ο αριθμός των σuνό'λιnν τύπου Υ θα είναι b0b1 . . . b0_1 . Αρκεί να αποδείξουμε ότι:

aoa ι . . . an-1 = bobι . .. bn-1 (*) ή, αρκεί να αποδείξουμε ό­τι τα στοιχεία του συνόλου { ao ' al ' . . . , an-1 } είναι μια μετάθεση των στοιχείων του συνόλου {bo , bι ' . . . , bn-1 } (**). Σχήμα ι

Θα χρησιμοποιήσουμε επαγωγή ως προς το θετικό ακέραιο η. • Για η = ι είναι φανερό ότι a0 = b0 • • Υποθέτουμε ότι η πρόταση (**) αληθεύει για

όλους τους θετικούς ακέραιους που είναι μι­κρότεροι του η.

• Θα· αποδείξουμε ότι η πρόταση (**) αληθεύει και για το θετικό ακέραιο η. Διακρίνουμε δύο περιπτώσεις:

α) Κάθε σημείο (χ, y) της ευθείας χ + y = η - ι είναι μπλε. Τότε αγνοώντας αυτά τα σημεία, αναγόμαστε στην περίπτωση του θετικού ακέραιου η - ι με η - 1 μπλε στήλες μεγέθους a0 - 1 , a1 - 1 , . .. , a0_2 - 1 και η - 1 μπλε γραμμές μεγέθους b0 - ι , b1 - ι , . . . , b0_2 - ι . Σύμφωνα με την υπόθεση της επαγωγής οι αριθμοί a0 - 1 , a1 - 1 , . . . , a0_2 - 1 είναι μια μετάθεση των αριθμών b0 - 1 , b1 - 1 , . . . , b0_2 - ι και επειδή εm­π'λiον είναι an-1 = 1 = bn-1 ' έπεται ότι ισχύει το ζητούμενο, δηλαδή οι αριθμοί είναι μια μετάθεση των αριθμών b0 , b1 , . . . , bn-ι .

β) Έστω ότι ιςάποιο σημείο (κ, η - ι - κ) είναι κόκκινο (σχ. 1). Τότε το ορθογώνιο ό'λιnν των ση­μείων (x,y) με χ :S κ και y :S η - 1 - κ είναι κόκκινο. Επομένως, θεωρώντας όλα τα σημεία (x,y) με χ < κ , από την υπόθεση της επαγωγής έχουμε ότι οι αριθμοί ao ' al ' . . . , aιc-1 είναι μια με­τάθεση των αριθμών bn-ιc • bη-κ+ι • . . . , bn-ι . Ομοίως έ­χουμε για χ > κ ότι οι αριθμοί aιc+l ' a ιc+2 > . . . , an-1

είναι μια μετάθεση των αριθμών b0 , b1 , . . . , b0_2_" • Επειδή είναι και a" = bn-Ι-ιc = Ο , έχουμε ότι οι αριθμοί a0 , a1 , . . . , a0_1 είναι μια μετάθεση των α-ριθμών b0 , b1 , . . . , bn-ι .

Λύση 2 Όπως και στην πρώτη λύση, θα αποδείξουμε

ότι οι αριθμοί a0 , a1 , . . . , a0_1 είναι μια μετάθεση των αριθμών b0 , b1 , . . . , bn-ι .

Θα χρησιμοποιήσουμε επαγωγή ως προς τον αριθμό των κόκκινων σημείων.

Όταν ο αριθμός των κόκκινων σημείων είναι μηδέν, τότε όλα τα σημεία είναι μπλε και το ζη­τούμενο ισχύει.Όταν υπάρχουν κόκκινα σημεία, τότε επιλέγουμε ένα από αυτά, έστω το (x, y) , με χ + y μέγιστο.

ΕΥΚΛΕΙΔΗΣ Β' λατ' τ.l/24

Page 27: Ευκλειδης Β 45

Μαθηματικοί Διαγωνισμοί - Μαθηματικές ολυμπιάδες

Τότε θα έχουμε: a = n - 1 - x - y = b ·Χ Υ

Αν αλλάξουμε το ση­μείο (χ, y) από κόκκινο Υ σε μπλε, τότε στο σχή-μα θα έχουμε λιγότερα κόκκινα σημεία.

(0, n-1)

χ

Σχήμα 2 Οι γραμμές και οι στήλες μένουν αναλλοίωτες

ως προς τον αριθμό των μπλε σημείων, εκτός από τους αριθμούς aχ και by οι οποίοι ελαττώνονται κα­τά 1 :Ετσι, από την υπόθεση της επαγωγής, οι α­ριθμοί a0 , • • • , a._1 , . . . , a0_ 1 είναι μια μετάθεση των αριθμών b0 , • • • , by-Ι , . . . , b0_1 . Επειδή είναι a. = bY , και οι αριθμοί a0 , • • • , a. , . . . , a0_1 θα είναι μια μετά­θεση των αριθμών b0 , • • • , bY , . . . , b0_1 , δηλαδή ι­σχύει το ζητούμενο.

Λύση 3

η- ι·

π-Ι Σχήμα 3 Παρατηρήσεις

Το πρόβλημα αυτό χαρακτηρίζεται ως εύκολο με μια ενδιαφέρουσα ποικιλία λύσεων. Πέραν από τις τρεις λύσεις που δώσαμε υπάρχουν και άλλες λύσεις των οποίων θα αναφέρουμε τη βασική ιδέα. • Υπολογισμός του αριθμού των γραμμών ή

στηλών με κ μπλε σημεία. Η βασική ιδέα εί­ναι να αποδείξουμε ότι ο αριθμός των i με ai � κ ισούται με τον αριθμό των μπλε ση­μείων στη διαγώνιο χ + y = η - κ . Είναι φανε­ρό ότι ο αριθμός των i με ai � κ , για κάθε κ,

Θα βρούμε μια αμφιμονοσήμαντη και επί α- •

ντιστοιχία μεταξύ των συνόλων

προσδιορίζει τον αριθμό των j με a j = κ . Υπάρχουν τρεις τουλάχιστον λύσεις που βα­σίζονται σε τύπους υπολογισμού του αριθμού

Α = {a0 , a1 , . . . , a0_1 } και Β = {b0 , b1 , . . . , b0_1 } . Βασική ιδέα της απόδειξης αυτής είναι «η διαγώ-νια κίνηση» έτσι ώστε να αντιστοιχίσουμε μια μπλε στήλη σε μια μπλε γραμμή με τον ίδιο αριθμό μπλε σημείων.

Αν είναι a. = Ο , τότε επίσης θα είναι και bn-Ι-x = Ο και στο a. αντιστοιχίζουμε τον αριθμό bn-1-x ·

Αν είναι a. > Ο , θεωρούμε το μπλε σημείο (χ, y) με την ελάχιστη τετμημένη χ. Μεταξύ των σημείων

. (x, y) , (x - 1 , y + 1 ) , (x - 2, y + 2) , . . . θα υπάρχει ένα τουλάχιστον το οποίο μεταξύ

των μπλε σημείων της γραμμής του θα έχει την ε­λάχιστη τετμημένη. Έστω το πρώτο από αυτά είναι το (χ', y') . Τότε έχουμε την αντιστοίχιση του aχ

στο by· ( σχ. 3). Η παραπάνω αντιστοιχία είναι αντιστρέψιμη. Πράγματι, έστω bY > Ο . Τότε, αν είναι (χ, y)

το μπλε σημείο της γραμμής με τεταγμένη y που έ­χει την ελάχιστη τετμημένη χ, επιλέγουμε μεταξύ των σημείων (x, y) , (χ + 1 , y - 1) , (x + 2, y - 2) , . . . το πρώτο από εκείνα που στη στήλη τους έχουν την ελάχιστη τεταγμένη. Αν αυτό είναι το (χ', y') , τότε αντιστοιχίζουμε τοv αριθμό by στον αριθμό

των συνόλων τύπου Χ και τύπου Υ. Η κατα­σκευή του πρώτου τύπου βασίζεται στη μέ­τρηση του αριθμού των ai και bj τα οποία εί­ναι μεγαλύτερα ή ίσα του κ, όπως στην προη­γούμενη λύση. Η κατασκευή του δεύτερου τύπου βασίζεται στην έκφραση σε όρους δυωνυμικών συντελεστών του αριθμού των συνόλων τύπου Χ με di σημεία επιλεγμένα α­πό τη χ + y = i διαγώνιο. Ο τρίτος τύπος δίνει τον αριθμό των συνόλων τύπου Χ ως γινόμε­νο των λόγων παραγοντικών, αφού γίνει ανά­λυση του συνόλου των μπλε σημείων σε τρα­πέζια . Π Ρ Ο ΒΛΗΜΑ 2 Έστω BC μια διάμετρος του κύκλου Γ με

κέντρο Ο. Έστω Α ένα σημείο πάνω στον κύκλο Γ τέτοιο ώστε 0° < ΑόΒ < 120° . Έστω D το μέ­σο του ΑΒ που δεν περιέχει το σημείο C. Η ευ­θεία που περνά από το σημείο Ο και είναι πα­ράλληλη προς την ευθεία DA τέμνει την ευθεία AC στο σημείο J. Η μεσοκάθετη του ευθυγράμ­μου τμήματος ΟΑ τέμνει τον κύκλο Γ στα ση­μεία Ε και F. Ν α αποδείξετε ότι το σημείο J εί­ναι το κέντρο του εγγεγραμμένου στο τρίγωνο CΕF κύκλου.

Λύση l Η ακτίνα ΟΑ είναι κάθετη στη χορδή EF. Άρα

το Α είναι μέσον του τόξου EF, οπότε η CA είναι διχοτόμος της γωνίας ECF .

ΕΥΚΛΕΙΔΗΣ Β' λστ' τ.l/25

Page 28: Ευκλειδης Β 45

Μαθηματικοί Διαγωνισμοί - Μαθηματικές ολυμπιάδες

Επειδή είναι ΟΑ = OC θα έχουμε: AOD =.!..AOB= OAC

2 Β c

Α

οπότε η D είναι παράλ­ληλη στην JA και το ODAJ είναι παραλλη­λόγραμμο. Σχήμα 4

Επιπλέον το τετράπλευρο OEAF είναι ρόμβος, αφού οι διαγώνιές του ΟΑ και EF διχοτομούνται και τέμνονται κάθετα.

Άρα θα είναι AJ = OD = ΟΕ = AF Επομένως JFΈ = JF Α - E:F Α = A1F - ECA

= A1F - JCF = .Wc οπότε η FJ διχοτομεί τη γωνία EFC και το σημείο J είναι το έκκεντρο του τριγώνου CEF, ως σημείο τομής δύο διχοτόμων του.

Η συνθήκη 0° < ΑΟΒ < 120° εξασφαλίζει ότι το σημείο J είναι εσωτερικό σημείο του τριγώνου CEF.

Λύση 2 Όπως στην προηγούμενη λύση, το ODAJ είναι

παραλληλόγραμμο και το τετράπλευρο OEAF εί­ναι ρόμβος, οπότε τα σημεία Ο και J βρίσκονται σε κύκλο S κέντρου Α και στο ίδιο τόξο χορδής EF. Έστω J0 το έκκεντρο του τριγώνου CEF. Επειδή Α είναι το μέσον του τόξου EF του κύκλου Γ, που δεν περιέχει το σημείο C, και τα δύο σημεία J και J0 βρίσκονται πάνω στο ευθύγραμμο τμήμα CA που είναι η εσωτερική διχοτόμος της γωνίας ECF .

Επιπλέον, από τις ισότητες AO = OE = EA = AF = FO

έχουμε ότι τα τρίγωνα ΑΕΟ και AFO είναι ισό­πλευρα και ίσα, οπότε θα είναι και EOF = 120° . Επειδή J0 είναι το έκκεντρο και Ο είναι το περίκε­ντρο του τριγώνου CEF θα έχουμε:

EJ 0F = 90° + .!_ ECF = 90° + .!_ EOF = 120° 2 4

Επομένως το σημείο J, όπως και το J0, βρί­σκονται στον κύκλο S. Επειδή ο κύκλος S έχει μο­ναδική τομή με τη διχοτόμο AC, συμπεραίνουμε ότι J = J0 •

Παρατηρήσεις • Ο μοναδικός σημαντικός περιορισμός του

προβλήματος είναι η σχέση 0° < ΑΟΒ < 120° . Γεωμετρικά ο περιορισμός αυτός είναι ισοδύ­ναμος με το ότι τα σημεία Α και C βρίσκονται σε διαφορετικά ·μέρη του επιπέδου ως προς τη χορδή EF ή ισοδύναμα ότι το J βρίσκεται στο εσωτερικό του τριγώνου CEF. Όταν είναι ΑΟΒ = 120° , τότε το τρίγωνο CEF

εκφυλίζεται. Όταν είναι ΑΟΒ > 120° , τότε το σημείο J εί­ναι ένα από τα παράκεντρα του τριγώνου CEF.

• Μια ακόμη λύση του προβλήματος βασίζεται στην ισότητα AJ = AF , σύμφωνα με το πα­ρακάτω λήμμα: Έστω τρίγωνο CEF με CA διχοτόμο της γω-νίας C , όπου το Α α­νήκει στον περιγε­γραμμένο κύκλο του τριγώνου. Έστω ακό­μη σημείο J της διχο­τόμου CA.

Α

Σχήμα 5 Τότε το J είναι το έκκεντρο του τριγώνου CEF, αν και μόνον αν, ισχύει η ισότητα AJ = AF .

• Η λύση μπορεί να γίνει και με χρήση Αναλυ­τικής Γεωμετρίας. Απαιτεί όμως την επιλογή κατάλληλου συστήματος συντεταγμένων, για­τί διαφορετικά οι πράξεις είναι πάρα πολλές. Π Ρ Ο Β Λ Η Μ Α 3 Να βρείτε όλα τα ζεύγη ακεραίων m, η ;:::: 3

για τα οποία υπάρχουν άπειροι θετικοί ακέραιοι αm + α - 1

αριθμοί α τέτοιοι ώστε ο αριθμός: ---=---απ + α2 - ι

να είναι ακέραιος.

Λύση ι Έστω ότι όλοι οι ακέραιοι m, η είναι ένα κα­

τάλληλο ζεύγος. Βήμα ι. Τότε θα είναι η < m . Πράγματι, αν

ίσχυε ότι η ;:::: m , δεδομένου ότι αm + α - 1 > Ο και α η + α2 - 1 > Ο για κάθε θετικό ακέραιο α, θα είχα-με: Ο < Α = αm + α - 1 � 1 <=>

α0 + α2 - 1 αm (1 - αn-m) + α (1 - α) � Ο που ισχύει για κάθε θετικό ακέραιο α. Η ισό­

τητα ισχύει μόνο για α = 1 . Επομένως για η ;:::: m δεν μπορεί ο αριθμός Α να είναι ακέραιος για ά­πειρες τιμές του α, που είναι άτοπο.

Βήμα 2. Στη συνέχεια θα αποδείξουμε ότι το πολυώνυμο f (x) = xm + χ - 1 διαιρείται από το πολυώνυμο g (χ) = xn + χ2 - 1 στο δακτύλιο των πολυωνύμων με ακέραιους συντελεστές Ζ [χ] . Πράγματι, επειδή το πολυώνυμο g (χ) έχει συντελεστή του μεγιστοβάθμιου όρου τη μονάδα, ο αλγόριθμος της διαίρεσης δίνει:

ΕΥΚΛΕΙΔΗΣ Β' λστ' τ.l/26

Page 29: Ευκλειδης Β 45

Μαθηματικοί Διαγωνισμοί - Μαθηματικές ολυμπιάδες

f (x) = q (χ) + r (χ) (Ι)

g (χ) g (χ) όπου r (χ) = Ο ή βαι3r (χ) < βα8g (χ) και

q (χ) Ε Ζ [χ] . Εmπλέον έχουμε ότι: lim r (x) =0 x...->g (x) ενώ, λόγω της (1) και της υπόθεσης, ο αριθμός r (α) , , , , Ε -- ειναι ακεραιος για απειρες τιμες του α. -g (α) .

, θ , r (α) Ο , , πομενως α ειναι -- = για απειρες τιμες του g (α)

θετικού ακέραιου α, δηλαδή θα είναι r (α) = Ο για άπειρες τιμές του α. Άρα r (χ) = Ο , για κάθε χ Ε R . Επιπλέον, είναι φανερό ότι f (α) Ε Ζ

g (α) για κάθε ακέραιο α.

Βήμα 3. Επειδή οι συναρτήσεις f (χ) και g (χ) είναι γνησίως aύξουσες στο διάστημα Ι = (0, 1] και f (I) = g (Ι) = [- 1 , 1] , θα έχουν μοναδική ρίζα στο διάστημα (0, 1) . Δεδομένου ότι το πολυώνυμο g (χ) διαιρεί το f (χ) η ρίζα αυτή, έστω ρ, θα είναι κοινή.

Βήμα 4. Στη συνέχεια θα χρησιμοποιήσουμε την κοινή ρίζα ρ για να αποδείξουμε ότι m < 2η . Καταρχήν είναι ρ > ω = 0,618 . . . , όπου ω είναι η θετική ρίζα του τριωνύμου h (χ) = χ2 + χ - 1 . Αυτό ισχύει γιατί η f είναι γνησίως αύξουσα στο [0, 1 ] και f(ω) < h (ω) = Ο = f(ρ) . Εmπλέον, αν ήταν m > 2η , τότε: 1 _ ρ = ρm :::; ρ2η = (ρ η γ = ( l _ ρ2 )2 � ( 1 - ρ) - (1 - ρ2 ) ( 1 + ρ2 ) :::; ο

� ρ (ρ - 1) (ρ2 + ρ - 1) ;:: Ο � ρ :::; ω , άτοπο. Βήμα 5. Θα αποδείξουμε ότι η μοναδική λύση

με m < 2η είναι το ζεύγος (m, η) = (5, 3) . Για α = 2 θέτουμε c = g (2) = 2° + 3 . Ε­

πειδή f(2) = 2m + 1 , θα είναι - 2m Ξ 1 (modc) (2)

Έστω m = η + κ , όπου 1 :::; κ < η . Τότε θα είναι: - 2m = - 2° · 2" = (3 - c) 2κ = 3 · 2κ - c · 2κ Οπότε - 2m Ξ 3 · 2κ (modc) από την οποία προκύπτει ότι - 2m ψ. 1 (modc) , ό­ταν Ι :::; κ :::; η - 2 .

Για κ = η - 1 , δηλαδή για m = 2η - 1 , έ­χουμε: - 2m = 3 · 2°-1 -πολ (c) = (2 + 1) · 2π-ι _πολ (c)

= c + (2η-ι - 3) - πολ (c) = πολ(c) + 2η-ι - 3

με 1 :::; 2η-ι - 3 < c , αφού η ;:: 3 . Η μοναδική τιμή του η που ικανοποιεί τη σχέση (2) είναι η η = 3 , οπότε (m, η) = (5, 3) .

Για (m, η) = (5, 3) έχουμε: α5 + α - 1 2 Α = = α - α + l Ε Ζ α3 + α2 - 1

για κάθε θετικό ακέραιο α, οπότε το ζεύγος (m, η) = (5, 3) είναι η μοναδική λύση του προ-βλήματος.

2η Λύση [S. Grozdev and Ν. Nikolov (Βουλγαρία)] Έστω m = η + κ, τότε

xm + x - l = xιc (X0 + χ2 -l) + x" - χ κ+2 + χ - 1, = χ"(χ0 + χ2 -l) + (x - 1)[ xιc(x + l) - 1]

οπότε θα έχουμε ότι χ η + χ2 1χ"(χ + Ι) - ι ή χ η + χ2 - ιlx"+l + χ" - ι .

Έστω ότι κ+l-n

χκ+Ι + χ" -1 = Σ αjxj (X0 + χ2 - 1) (1) j=O

Συγκρίνοντας τους αντίστοιχους συντελε-

, , {1, αν j άρτιος στες λαμβανουμε: αj = . , , Ο, αν J περιττος για κάθε j < κ - 2.

Τότε χ"+1 +χ" -l=(xn +χ2 -ι{ χ"-2Ρ(χ)+ tx2j)=

e

= xn Σx2i + (χ2 - l)x"-2P(x) + x2U+Ι> , (2) i=O

όπου .e = [;]- ι και p(x) E Z[x] .

Θεωρώντας το συντελεστή του xn προ­κύπτει ότι η ;:: κ - 2 .

Επιπλέον, η ισότητα των βαθμών του δύο μερών της (2) δίνει η + 2.e :::; κ + 1 .

Άρα κ - 2 :::; n :::; κ - 2[;] + 3, δηλαδή κ ::s; 5

και η :::; 4. Αν είναι n=4, τότε κ=3 ή κ=5 και με απ'

ευθείας επαλήθευση καταλήγουμε ότι και οι δύο περιπτώσεις είναι αδύνατες .

Αν έχει n=3, τότε κ Ε {2,3,4,5} και με έ-

λεγχο βρίσκουμε ότι κ=2. Επομένως είναι n=3, m=5. Π Ρ Ο Β Λ Η Μ Α 4 Έστω ακέραιος αριθμός μεyαλύτερος του

1. Όλοι οι θετικοί διαιρέτες του n είναι οι d1 , d2, ••• , dκ , όπου

ΕΥΚΛΕΙΔΗΣ Β' λστ' τ.l/27

Page 30: Ευκλειδης Β 45

Μαθηματικοί Διαγωνισμοί - Μαθηματικές ολυμπιάδες

ι = dι < d2 < •.. < dκ = η

Ορίζουμε D = d1d2 + d2d3 + ••• + dκ_ιdκ •

α) Να αποδείξετε ότι D < η2 • β) Να προσδιορίσετε όλες τις τιμές του η

για τις οποίες ο αριθμός D είναι διαιρέτης του η:z.

Λύση α) Αν d; , i = 1 , 2, . . . , κ είναι ένας διαιρέτης

' θ ' η δ ' τοu η, τοτε α ειναι και ο d. ιαιρετης το� η και I

μάλιστα :. = dιc+Ι-i . Έτσι έχοuμε: I

2 ιc-Ι ( 1 1 ) 2 ( 1 ) 2 = η � - - - = η 1 - - < η

Ι=l d; di+l η β) Έστω ρ ο ελάχιστος πρώτος διαιρέτης του

η. ,Τότε d2

= . ρ , dιc-Ι = � και dιc = η . ρ Διακρίνουμε δύο περιπτώσεις:

• Αν η πρώτος, τότε η = ρ , κ = 2 και ο D = ρ διαιρεί το η2•

• Α ν η σύνθετος, τότε κ > 2 και 2 D > dιc_1dιc = � . Αν το D διαιρεί το η2, τό­ρ

η2 τε και το - θα διαιρεί το η2• Όμως D n2 , , , 1 < - < ρ , που ειναι αδυνατο, γιατι ο ρ D

είναι ο ελάχιστος πρώτος διαιρέτης του η, άρα και του η2• Άρα ο D διαιρεί το η2 , για κάθε ακέραιο η

που είναι πρώτος. Π Ρ Ο Β Λ Η Μ Α 5 Να βρείτε όλες τις συναρτήσεις f : R � R ,

όπου R είναι το σύνολο των πραγματικών αριθ­μών, για τις οποίες ισχύει:

(f (x)+f (z)) (f(y)+f(t)) =f (:xy-zt)+f(xt +yz) (*) για όλα τα χ, y, z, t που ανήκουν στο R.

Λύση Για χ = y = z = Ο, t Ε R , η εξίσωση (*)

δίνει 2 f (Ο) [ f (Ο) + f (t)] = 2 f (Ο) (1) για κάθε t Ε R ,οπότε για t = Ο έχουμε:

. 1 f (Ο) = Ο ή f (Ο) = l . Α ' f (O) 1 ' ' (1) ' ν ειναι = - , τοτε απο την εχουμε

2

f(t) = .!.. , για κάθε t Ε R , 2

η οποία εύκολα επαληθεύουμε ότι είναι λύση του προβλήματος.

Αν είναι f (Ο) = Ο , τότε από την (*) για z = t = Ο έχουμε την ισότητα:

f (x) f (y) = f (xy) (2) για κάθε χ, y Ε R , δηλαδή η f είναι πολλαπλα­σιαστική.

Από τη (2) για χ = y = 1 προκύπτει f(1)2 = f(1) � f (1) = ο ή f(1) = 1 .

Α ν είναι f (1) = Ο , τότε f(x) = f (x · 1 ) = f (x) f(1) = Ο , για κάθε χ Ε R , η οποία είναι μία ακόμη λύση του προ­βλήματος.

Α ν είναι f ( 1) = 1 , επειδή έχουμε υποθέσει ότι f (O) = Ο , η εξίσωση (*) για χ = Ο , y = t = 1 , δίνει f (- z) + f (z) = 2f(z) ή

f (- z) = f (z) (3) για κάθε z Ε R , δηλαδή η f είναι άρτια συνάρτη­ση.

Από τη (2) για y = χ λαμβάνουμε f (x2 ) = f (x)2 � Ο , οπότε για χ = .JY , y � Ο θα είναι: f (y) = f(.JY)2 � Ο (4)

για κάθε y � Ο . Επειδή η f είναι άρτια η σχέση ( 4) αληθεύει για κάθε y Ε R .

Επιπλέον, για t = χ και z = y η σχέση (*) δίνει: f (x2 + y2 ) = [f (x) + f(y)Γ (5)

για κάθε χ, y Ε R , οπότε θα είναι: f (x2 + y2) � f(x)2 = f(x2) (6)

για κάθε χ, y Ε R . Επομένως, αν είναι u � ν � Ο , τότε θα είναι f (u) � f(v) , δηλαδή η συνάρτηση f είναι αύξουσα στο σύνολο [0, + οο) .

Από τη σχέση (*) για χ = z = t = 1 λαμβά-νουμε: f(x - 1) + f(x + 1) = 2 [ f(x) + 1] (7)

Με επαγωγή ως προς το η, αφού f (O) = Ο = 02 και f(1) = 1 = 12 , μέσω της σχέσης (7) εύκολα προκύπτει ότι f(η) = η2 , για κάθε η στο σύνολο των μη αρνητικών ακεραίων. Επειδή η f είναι άρτια θα είναι:

f (η) = η2 για κάθε η Ε Ζ Από την πολλαπλασιαστικότητα της f, για κά­

θε p, q Ε Ζ, q -=ι:. Ο , θα έχουμε:

1 = f(1) = r (q ·*) = f (q) f (*)

ΕΥΚΛΕΙΔΗΣ Β' λστ' τ.l/28

Page 31: Ευκλειδης Β 45

Μαθηματικοί Διαyωνισμοί - Μαθηματικές ολυμπιάδες

και ακόμη

1 f (q)

r (�) � r (p ·�) � f(p) fω � �� � ω

δηλαδή ισχύει f(α) = σ! , για κάθε ρητό αριθμό α. Υποθέτουμε ότι ισχύει f (x) ::F- χ2 για κάποιο

θετικό πραγματικό αριθμό χ. Αν είναι f (x) < χ2 , τότε θεωρούμε ρητό α-

ριθμό α τέτοιον ώστε χ > α > .Jr (χ) . Τότε θα είναι f (α) = α2 > f (x) . Όμως, επειδή στο [0, + οο) η f είναι αύξουσα θα έχουμε f (α) � f (x) , που αντίκειται στην προηγούμενη σχέση. Επομένως δεν μπορεί να ισχύει η σχέση f(x) < χ2 • Ομοίως είναι αδύνατη η σχέση f(x) > χ2 , για τον τυχόντα θετικό πραγματικό α­ριθμό χ.

Άρα είναι f(x) = χ2 , για κάθε χ Ε [0, + οο) , και αφού η f είναι άρτια θα είναι f(x) = χ2 , για κάθε χ Ε R .

Παρατηρήσεις 1. Το πρόβλημα αυτό είναι μεσαίας δυσκολίας

και απαιτεί κατάλληλη επιλογή των μεταβλητών για την εύρεση των διαφόρων συνθηκών που οδη­γούν τελικά σε όλες τις λύσεις. Η κύρια δυσκολία του προβλήματος είναι η μετάβαση από τους ρη­τούς στους πραγματικούς αριθμούς. Για να γίνει α­ντιληπτή η δυσκολία συλλογής βαθμών από τους διαγωνιζόμενους στις Διεθνείς Μαθηματικές Ολυ­μmάδες, αναφέρουμε ότι για την απόδειξη της σχέσης f (α) = α2 στους ρητούς η βαθμολογία ή­ταν 2 μονάδες με άριστα το 7, ενώ για την εύρεση των δυο σταθερών λύσεων δεν δινόταν καμία μο­νάδα.

2. Η δημιουργία του προβλήματος αυτού έχει υποκινηθεί από τη γνωστή ταυτότητα του Lagrange:

(xz + zz) (yz + tz ) = (xy - zt)z + (xt + yz)z που χρησιμοποιείται για τη μελέτη αθροισμάτων δύο τετραγώνων στη θεωρία αριθμών.

Π Ρ Ο Β Λ Η Μ Α 6 Δίνονται οι κύκλοι Γ I ' Γ 2 ' ••• , Γ n ' η ;;:: 3 ' που

βρίσκονται στο ίδιο επίπεδο και ακτίνα ίση με 1. Τα κέντρα τους είναι τα σημεία 01 , 02 , ••• , On

αντίστοιχα. Δίνεται ακόμα ότι δεν υπάρχει ευ­θεία που να έχει κοινά σημεία με περισσότερους από δύο κύκλους.

Ν α αποδείξετε ότι Σ 1 (n - 1) π -- �

I S I < jSn ορj 4

Όπου Σ 1 1 1 = + -- + ...

IS i< jSn ορj 0102 ο.ο3

1 1 1 +-- + + ... +

o.on 0203 on·lon

Λύση Θα χρησιμοποιήσουμε τη βοηθητική πρόταση

που ακολουθεί. Λήμμα 1. Έστω Ω κύκλος ακτίνας ρ και PR,

QS δύο χορδές του που τέμνονται στο σημείο Χ έ-τσι ώστε PXQ = RXS = 2α . Τότε ισχύει:

L (PQ) + L (RS) = 4αρ όπου με L (PQ}, L (RS) συμβολίζουμε το μήκος του τόξου RQ, RS αντίστοιχα.

Απόδειξη Έστω Ο το κέντρο του κύκλου Ω,

ΡόQ = 2λ , Rόs = 2μ . Τότε: QSR = λ και RPS = μ .

Επομένως RXS = 2α = λ + μ και L (PQ) + L (RS) = 2λρ + 2μρ

= 2 (λ + μ) ρ = 4αρ

Σχήμα 6 Επανερχόμενοι στη λύση του προβλήματος,

θεωρούμε ένα μεγάλο κύκλο Ω ακτίνας ρ, που στο εσωτερικό του περιέχει όλους τους δεδομένους κύκλους Γ1 , Γ2 0 . . . , Γn .

Θεωρούμε και τους κύκλους Γ; , Γ! Από τις υποθέσεις του προβλήματος οι κύκλοι αυτοί δεν έ­χουν κοινά σημεία. Έστω 2α η γωνία των δύο ε­σωτερικών κοινών εφαπτομένων PR, QS. Τότε θα είναι: ορj =

και

χ

2 η μα

2 α ;;:: ημα =

ορj

Σχήμα 7

(1)

Ω

ΕΥΚΛΕΙΔΗΣ Β' λστ' τ.l/29

Page 32: Ευκλειδης Β 45

Μαθηματικοί Διαyωνισμοί - Μαθηματικές ολυμπιάδες

Σύμφωνα με το Λήμμα 1 και τη σχέση (1) θα έχουμε: L (PQ) + L (RS) = 4αρ � �

ορj

δηλαδή _1_ < L (PQ) + L (RS) (2) ορj - 8ρ ·

Στη συνέχεια θα θεωρήσουμε το άθροισμα των μηκών των τόξων που εμφανίζονται στον α­ριθμητή της σχέσης (2), καθώς το ζεύγος (i, j) παίρνει όλες τις δυνατές nμές για 1 � i < j � η . Θα αποδείξουμε όn κάθε σημείο του κύκλου Ω καλύπτεται από τέτοια τόξα το πολύ η - 1 φορές.

Θεωρούμε τυχόν σημείο Τ του κύκλου Ω και την εφαπτομένη ημιευθεία Τ χ του Ω, η οποία περι­στρέφεται γύρω από το Τ κατά 1 80°, όπως φαίνε­ται στο σχήμα 7. Κατά την περιστροφή της η η μι­ευθεία τ χ θα συναντήσει για πρώτη φορά ένα ζευ­γάρι κύκλων, έστω με νέα αρίθμησή τους Γι , Γ2. Η

ημιευθεία Τ χ δεν είναι δυνατόν να συναντά ταυτό­χρονα τρεις κύκλους, οπότε κάποια στιγμή κατά την περιστροφή της θα συναντήσει έναν από τους δύο κύκλους, έστω το Γ1 , προτού συναντήσει κά­ποιον άλλον από τους κύκλους. Ο κύκλος Γ 1 δεν θα συναντηθεί ξανά από την ημιευθεία Τ χ· Συνεχί­ζοντας ομοίως και με νέα κατάλληλη αρίθμηση των κύκλων, συμπεραίνουμε όn ο μέγιστος αριθ­μός ζευγών κύκλων που συναντά η ημιευθεία Τ χ

είναι η - 1 , δηλαδή τα ζεύγη Γι και Γ2, Γ2 και Γ3, . . . , Γη-Ι και Γ0•

Δεδομένου όn το σημείο Τ είναι τυχαίο, το ά­θροισμα των μηκών των τόξων του αριθμητή της σχέσης (2) είναι μικρότερο ή ίσο του (η - 1) · 2πρ .

Σ 1 (η - 1) 2πρ (η - 1) π Άρα θα είναι: -- < = �___:._-ι,; i<j,;n ορj 8ρ 4

.19η ΒΑΛΚΑΝΙΚΉ ΜΑΘΗΜΑΤΙΚΉ ΟΛΥΜΠΙΑΔΑ ΑΤΤΑΛΕΙΑ ΤΟΥΡΚΙΑΣ, 27 ΑΠΡΙΛΙΟΥ 2002

Π Ρ Ο Β Λ Η Μ Α ι Τα σημεία Αι, A:z, . • • An, π � 4 , του επιπέ­

δου είναι τέτοια ώστε να μην υπάρχουν τρία α­πό αυτά που να είναι συνευθειακά. Μερικά ζεύ­γη διαφορετικών σημείων από τα Αι, A:z, • . . An συνδέονται με ευθύγραμμα τμήματα έτσι ώστε κάθε ένα από τα σημεία να συνδέεται με άλλα τρία τουλάχιστον σημεία. Να αποδείξετε ότι υ­πάρχει κ > ι και διαφορετικά μεταξύ τους ση­μεία Χ1, X:z, • • . Χ:zκ του συνόλου {Αι , Α1 , ••• , An } τέτοια ώστε για κάθε ί με ι :::; i :::; 2κ - 1 ' το ση­μείο Xi είναι συνδεδεμένο με το σημείο Χi+ι και το σημείο Χ:zκ με το σημείο Χι.

Λύση ι Δυο σημεία που συνδέονται με ευθύγραμμο

τμήματα λέμε όn είναι γειτονικά. Έστω Ρ: = Χ1Χ2 . . . Χ. με Χ; ε {Α1 , Α2 , . . . , Α0 } , Χ; =#:- Χ j για ί =#:- j , η μακρύτερη ακολουθία σημεί­ων τέτοιων ώστε τα Χ; , Χ+ι είναι γειτονικά για κάθε i = 1 , 2, .. . , s-1 . Σύμφωνα με την υπόθεση το σημείο Χι είναι γειτονικό με τρία τουλάχιστον σημεία. Δύο από αυτά, έστω τα Υ, Ζ, πρέπει να εί­ναι διαφορεnκά από το Χ2. Όμως, λόγω του όn τα Χι, Χ2, . . . , χ. ορίζουν τη μακρύτερη δυνατή α­κολουθία με γειτονικά διαδοχικά σημεία, θα πρέ­πει {Υ ,Ζ} c {Χ3 , •• .χ. } . Έστω Υ = Χ; , Ζ = Xj , με i < j . Τότε το C: = Χ1Χ2 .. . Χ; . . . ΧjΧι είναι κύκλος με χορδή Χ1Χ; .

Αν ο j είναι άρnος, τότε C είναι ο ζητούμενος κύκλος.

Α ν ο j είναι περιπός, τότε μόνον ένας από τους κύκλους Χ1Χ2 . . . Χ;Χ1 , Χ1Χ;Χ;+ι . . . ΧjΧι έ-χει άρτιο αριθμό κορυφών και ικανοποιεί το πρό­βλημα.

Π Ρ Ο Β Λ Η Μ Α 2 Δίνεται η ακολουθία a1, a1, • • • , ao με

a. = 20 ' 3:z = 30 ' an+:Z = 3an+1 - an ' π � ι . Ν α βρείτε όλους τους θετικούς ακέραιους π

για τους οποίους ο ι + 5anan+l είναι τέλειο τε­τράγωνο.

Λύση Αν θέσουμε bn = an+ι + an , cn = 1 + 5anan+ι ,

τότε 5ao+l = bn+l + bn ' an+2 - aπ = bn-1 - bn ' οπότε co+l - cn = 5an+l ( an+2 - an ) = b�+l -b� .

Επομένως, για κάθε η , έχουμε Cn+l -b�+l = Cn -b� = C1 - b; = 501 = 3•167 .

Έ 2 ' θ ' ' στω cn = m για καποιους εnκους ακεραι-ους η και m. Επειδή οι 3 και 167 είναι πρώτοι α­ριθμοί, έπεται όn m + bn = 167 ,m- bn = 3 , ή

m + bn = 501 , m -bn = 1 . Επομένως cn = 852 ή cn = 25 12 . Εmπλέον, με μία απλή επαγωγή , αποδεικνύε­

ται όn η ακολουθία cn είναι γνησίως μονότονη.

ΕΥΚΛΕΙΔΗΣ Β' λστ" τ.l/30

Page 33: Ευκλειδης Β 45

Μαθηματικοί Διαγωνισμοί - Μαθηματικές ολυμπιάδες

c1 = 1 +5•20•30 < 842 < c2 = 1+5•30•70 Επειδή < c3 = 1+5·70·180 = 2512

η μοναδική λύση του προβλήματος είναι η = 3 . Π Ρ Ο Β Λ Η Μ Α 3 Δύο κύκλοι με διαφορετικές ακτίνες τέμνο­

νται στα σημεία Α και Β. Οι κοινές εφαπτόμενες των δύο κύκλων είναι οι ΜΝ και ST, όπου τα σημεία Μ, S ανήκουν στον ένα κύκλο και τα Ν, Τ ανήκουν στον άλλον. Να αποδείξετε ότι τα ορθόκεντρα Ηι, Hz, 83 και Κι των τριγώνων ΑΜΝ, AST, ΒΜΝ και BST, αντιστοίχως, είναι κορυφές ορθογωνίου.

Λύση Τα ζεύγη των ορθοκέντρων Η1, Η2 και Η3, l4

είναι συμμετρικά ως προς τη διάκεντρο, οπότε τα τέσσερα ορθόκεντρα είναι κορυφές ισοσκελούς τραπεζίου. Επομένως, αρκεί να αποδείξουμε ότι: Η,Β ..L ΑΒ .

Ας υποθέσουμε ότι τα Q, Ρ είναι τα σημεία τομής της ΜΝ με τις ΑΗ1 και ΑΒ, αντιστοίχως. Τότε το Q είναι μέσον της ΜΝ και επειδή είναι ΑΡ ..L ΜΝ , αρκεί να αποδείξουμε ότι το τετρά­πλευρο H1BQP είναι εγγράψιμο. Ισοδύναμα, αρκεί να αποδείξουμε ότι: ΑΗ, · ΑΡ = AB · AQ (1)

Αν θεωρήσουμε ότι το περιγεγραμμένο στο κύκλο του τριγώνου ΑΜΝ, θα έχουμε:

Επομένως:

ΡΜ - ΡΝ = ΡΗ, · ΡΑ (2)

ΑΗ, · ΑΡ = (ΑΡ - Η,Ρ) ·ΑΡ = ΑΡ2 - ΡΜ · ΡΝ = ΑΡ2 - ΡΜ · ΡΝ = (AQ2 - PQ2 ) - (MQ - PQ) (NQ + PQ) = AQ2 - PQ2 - MQ2 + PQ2 = AQ2 - MQ2 = AQ2 - QA · QB [ως προς κύκλο (Α, Β, Μ)] = AQ (AQ - QB) = AQ ·AB , που είναι το ζητούμενο. Π Ρ Ο Β Λ Η Μ Α 4 Να βρείτε όλες τις συναρτήσεις f : Ν• � Ν•

που είναι τέτοιες ώστε για κάθε η ε Ν• να ισχύ-ει:

2n + 2001 � f ( f (π)) + f (η) � 2n + 2002 (*) Λύση Λόγω της παρουσίας του όρου ( f (η)) στη

σχέση (*) και επειδή η συνάρτηση f είναι ακολου­θία θετικών ακεραίων, ορίζουμε την ακολουθία θετικών ακεραίων:

a0 εΝ * , aκ+Ι = f (aκ) , κ ε Ν* Αντικαθιστώντας το η με τον όρο aκ_2, κ � 2 ,

στη σχέση (*) λαμβάνουμε: 2001 � aκ + aκ-1 - 2aκ-2 � 2002 (1)

Θεωρούμε την . ακολουθία: bκ = aκ - 667κ , κ εΝ * (2)

Τότε η σχέση (1) γίνεται: Ο � bκ + bκ-Ι - 2bκ_2 � 1 Ο � bκ - bκ-Ι + 2 (bκ-Ι - bκ_2) � 1

οπότε, αν θέσουμε bκ - bκ-1 + 2 (bκ-l - bκ-2 ) = εκ ' κ � 2 ' τότε θα έ-

χουμε: bκ - bκ-1 = - 2 (bκ-1 - bκ-2 ) + εκ (3) Κ � 2 , όπου εκ Ε { 0, 1} . Από τη σχέση (3) προκύπτουν οι ισότητες: bκ - bκ-1 = - 2 (bκ-1 - bκ-2 ) + εκ = (- 2)2 (bκ-2 - bκ-3 ) + (- 2) εκ-Ι + εκ = (- 2)3 (bκ-3 - bκ-4) + (- 2)2 εκ-2 + (- 2) εκ-Ι + εκ = (- 2)"-' (b1 - b0) + (- 2)"-2 ε2 + (- 2)""3 ε3 + . . . (- 2) εκ-Ι + εκ (4) Προσθέτουμε κατά μέλη τις ισότητες που

προκύπτουν από την ( 4) για κ = η, η - 1, . . . 2, ο-πότε λαμβάνουμε:

b. -b1 = [(-2) + . . . + (-2)"-1 ](b 1 -b0) +[1 + ( -2) + . . . + ( -2)"-2 ]ε2 +[1 + ( -2) + .. . + ( -2)"-3 ]ε3 + . . . + [1 + ( -2)]ε._1 + ε. ,

ή μετά τους υπολογισμούς τωv αθροισμάτων

b -b = [-� +�(-2)"-' ](b -b )+ε [1 -(-2Γ' ]

+ n Ι 3 3 I Ο 2 3

[1 - (-2)"-2 ] [1 - (-2)2 ] 1 - (-2) +ε3 3 + .. . + ε._,

3 + ε. .

3 ή ισοδύναμα μετά τις πράξεις

b _ 2b0 + b, (-2)" (b -b ) . - + ι ο + 3 3

+�[ε. + ( -2)ε._, + . . . + ( -2)"-3 ε3 + ( -2)"-2 ε2 J

1 +-(ε2 + ε3 + .. . + ε. ) .

3 Επειδή εκ ε {0, 1} , το μέγιστο του αθροίσμα­

τος που περιέχει τις δυνάμεις του -2, δηλαδή του αθροίσματος

ε. + ( -2)ε0_1 + . . . + ( -2)"-3 ε3 + ( -2)"-2 ε2 ,

ΕΥΚΛΕΙΔΗΣ Β' λστ' τ.l/31

Page 34: Ευκλειδης Β 45

Μαθηματικοί Διαγωνισμοί - Μαθηματικές ολυμπιάδες

λαμβάνεται όταν θεωρήσουμε τις δυνάμεις ( -2) κ με άρτιο εκθέτη κ με αντίστοιχο εκ = 1 και τις δυ­νάμεις { -2)κ με περιττό εκθέτη με αντίστοιχο εκ = Ο . Έτσι έχουμε

ότι 2π ( 2) aπ � α +β ' η-3 γ -3 .

Επειδή ο γ είναι ακέραιος θα πρέπει γ � Ο, ο­πότε για η>4 έχουμε 2π > η2 και

1 εη + ( -2)επ-ι

+ . . . + ( -2)"-3 ε3 + ( -2)π-2 ε2 � an � α + β lη --η2 , 9

2[π-2] 1 [ 2[π-2] 2 J � 1 + 22 + 24 + . . . + 2 -2 = 3 2 -2 + - 1 . 1 που είναι άτοπο, γιατί η συνάρτηση α+ β 1 η --η 2 9

Επομένως υπάρχουν α, β, γ με γ = b, -b0 τέ­τοια ώστε, για κάθε η να ισχύει:

τελικά παίρνει αρνητικές τιμές, δηλαδή υπάρχει θετικός ακέραιος η0 τέτοιος ώστε για κάθε η > η0

να ισχύει α + β1η -_!_η2 < 0 . 2 π 2 2[ π;2]+2

b � α + βη + i=_l_γ + ---π 3 9 ή επειδή bπ = an - 667η θα ισχύει:

9 Ομοίως αποκλείουμε την περίπτωση με γ<Ο,

θεωρώντας άρτιες τιμές για το η. Επομένως θα έχουμε γ = b, -b0 = Ο , δηλαδή

( 2)π 2 2[ n;2 ]+2

aπ � α + β 'η + -3-γ +

9 ( 1 )

b , = b0 , οπότε α, = α0 + 667 , δηλαδή f ( α0 ) = α0 + 667, Επειδή το α0 επελέγη τυχαία, έ­πεται ότι f(x) = χ + 667 , για κάθε χ ε Ν. Τέλος, εύκολα επαληθεύουμε ότι η παραπάνω συνάρτηση ικανοποιεί τις συνθήκες του προβλήματος.

αν θέσουμε β 1 = β + 667η. Θα αποδείξουμε ότι είναι γ=Ο. Πράγματι, αν

ήταν γ>Ο, τότε από την ( 1) για η περιττό προκύπτει

• Επιστημονικά Για ΑΕΙ, ΤΕΙ, ΙΕΚ

• Εκπαιδευτικά Για Γυμνάσιο, Λύκειο, ΤΕΕ

• Διάφορα Λογοτεχνία, μελtτες, λευκώματα

ΠΛΗ ΡΕΙΣ ΣΕΙ ΡΕΣ ΕΚΠΑΙΔΕΥ1Ι ΚΩΝ Β Ι ΒΛΙΩΝ

θ . ΞΕΝΟΣ ΚΡΙΠΙΡΙΑ ΑΞΙΟΛΟΓΗΣΗΣ

Γ' ΛΥΚΕΙΟΥ

θεπκής • π:χν/κής κατ/ναιι�

θ. ΞΕΝΟΣ ΜΑθΗΜΑΥΙΚΑ

Α' ΓΥΜΝΑΣΙΟΥ

ΜΑΘΗΜΑτtΚΑ

θ. ΞΕΝΟΣ ΚΡΠΗΡΙΑ ΑΞΙΟΛΟΓΗΣΗΣ

Γ' ΛΥΚΕΙΟΥ

Γενική� παιδε/ας

θ. ΞΕΝΟΣ ΜΑθΗΜΑΥΙΚΑ

Β' ΓΥΜΝΑΣΙΟΥ

θ. ΞΕΝΟΣ ΚΡΠΗΡΙΑ ΑΞΙΟΛΟΓΗΣΗΣ

Β' ΛΥΚΕΙΟΥ

θmκ�� • π:χν/ιιή� κατ/ναιι�

ΣΥΝΔ ΥΑΣΊ'1ΚΗ ΑΠΑΡlθΜΗΣΗ tιτtι-ιι-........ �,.n-

Χ. ΜΟΥΣΙΑΔΗΣ ΣΥΝΔΙΑΣτιΚΗ

ΑΠΑΡΙθΜΗΣΗ

θ. ΞΕΝΟΣ ΚΡΠΗΡΙΑ ΑΞΙΟΛΟΓΗΣΗΣ

Β' ΛΥΚΕΙΟΥ

Γενικ�� παιδεlα�

Ν. ΚΑΡΑΜΠΕτΑΚΗΣ ΕΙΣΑΓΟΓΗ ΣτΗΝ

FOIΠRAN 90/95

ΜΑΘΗΜΑτΙΚΑ

θ. ΞΕΝΟΣ ΜΑθΗΜΑΥΙΚΑ

Β' ΚΥΚΛΟΥ Τ.Ε.Ε.

j Engll$h for Mathematlζs

Γ. ΔΑΝΟΥΣΗΣ • F. EVANS ENGLISH

FOR MAΠIEMAYICS

Μπορείτε να δείτε τις νέες μας εκδόσεις στο lnternet στην ιστοσελίδα:

Τα (Jιβλfα μας θα τα (Jpεfτε σε 6λα τα (Jιβλιοπωλεfα

θ. ΞΕΝΟΣ ΠΡΑΚΥΙΚΗ

ΑΡΙθΜΗτιΚΗ

Σ1'0ΙΧΕΙΑ ΜΕΙ'ΡΟ8Ε.ΩΡΙΑΣ ------------------ΠlθΑΝΟ'ΠΠ'nΝ

-----Σ. ΚΜΠΑΖΙΔΟΥ

ΠΟΙΧΕΙΑ ΜΕΥΡΟθΕΟΡΙΑΣ

ΠΙθΑΝΟτιιτΟΝ

Γ. ΚΛΑΠΑΝΑΡΗΣ ΑΛΓΕΒΡΑ

Β' ENIAIOY ΛΥΚΕΙΟΥ

At.ΙIJI: I11 Α 6 Γ Ι rΙΗ ιιιmιιuι IIW:· DfJ.FDTιι;Q UT&JAITil

• ΒΙΒΛΙΟΠΩΛΕΙΟ ΕΚΔΟΣΕΙΣ ΖΗΤΗ Αρμι;νοπούλου 27 - ΘΕΣΣΑΛΟΝΙΚΗ 546 35 • τηλ. 2310-203.720 - fax 2310-2 1 1 .305 • e-mail:[email protected]

Β. ΦΡΑΓΚΟΥ ΑΣΚΗΣΕΙΣ

ΛΟΓΙΣΜΟΥ ΣΥΝΑΡΠΙΣΗΣ

ΜΙΑΣ ΠΡΑΓΜΑΥΙΚΗΣ

ΜΠΑΒΛΗΠΙΣ

Β' ΕΚΔΟΣΗ • ΒΙΒΛΙΟΠΩΛΕΙΟ ΑΘΗΝΩΝ «ΕΝΩΣΗ ΕΚΔΟΤΩΝ ΒΙΒΛΙΟΥ ΘΕΣΣΜΟΝΙΚΗΣ» Στοά του Βιβλίου (Πεσμαζόγλου 5) - 1 05 64 ΑΘΗΝΑ • τηλ.-faχ 2 10-32 1 1 .097 • ΑΠΟΘΗΚΗ ΑΘΗΝΩΝ Βαλτετσίου 45 - ΕΞΑΡΧΕΙΑ 1 06 8 1 - ΑΘΗΝΑ • τηλ.-faχ 21 0-38 16.650 • e-mail:[email protected]

i

Page 35: Ευκλειδης Β 45

Μαθηματικά Για την Α ' τciξη του Λυκείου

.Υπάρχουν και Ανισότητες των Θανάση Χριστόπουλου - Παναγιώτη Χριστόπουλου

Α. Εισαγωγή Είναι αλήθεια ότι στην Άλγεβρα κυριαρχούν οι

ισότητες και οι εξισώσεις. Υπάρχουν όμως και οι . . . ανισότητες. Είναι απαραίτητο να μπορούμε να συ­γκρίνουμε δύο ποσότητες που ισοδύναμα σημαίνει να συγκρίνουμε μία ποσότητα (τη διαφορά τους) με το Ο. Είναι επίσης απαραίτητο να μπορούμε να επι­λύουμε ανισώσεις. (1)

Στην απόδειξη μιας ισότητας και στην επίλυση μιας ανίσωσης χρησιμοποιούμε τις παρακάτω προ­τάσεις (ιδιότητες) πολλές από τις οποίες περιέχονται στο σχολικό βιβλίο. Ας προσέξουμε όμως και κά­ποιες από αυτές που δεν τις συναντάμε στο σχολικό μας βιβλίο και αξίζει να τις επισημάνουμε.

Β . Σ τ ο ιχ ε ί α Θ εω ρ ί α ς i) α > β <::::> α - β > Ο Δηλαδή ένας αριθμός α είναι μεγαλύτερος από

έναν αριθμό β αν και μόνο αν η διαφορά α - β εί-ναι θετικός αριθμός, δηλαδή α - β > Ο . <2>

ίί) α > β <::::> α + γ > β + γ Δηλαδή αν προσθέσουμε και στα δύο μέλη

μιας ανισότητας οποιονδήποτε αριθμό η ανισότη­τα δεν μεταβάλλεται και αντίστροφα.

ίίί) α > β <=> {αγ > βγ αν γ > Ο αγ < βγ αν γ < Ο

Δηλαδή αν πολλαπλασιάσουμε και τα δύο μέ­λη μιας ανισότητας με έναν θετικό αριθμό η ανι­σότητα δεν μεταβάλλεται και αν πολλαπλασιά­σουμε και τα δύο μέλη μιας ανισότητας με αρνητι­κό αριθμό η ανισότητα αλλάζει φορά.

Παρατήρηση 1 Για την αφαίρεση και τη διαίρεση ισχύουν τα

παραπάνω, αφού η αφαίρεση είναι η πρόσθεση του αντίθετου και η διαίρεση ο πολλαπλασιασμός του αντίστροφου ενός αριθμού.

Παρατήρηση 2 Οι aντίστροφες προτάσεις των (ίί) και (ίίί) α-

( Ι) 5 + 3 = 8 � ΙΣΟΤΗΤΑ χ + 3 = 8 � ΕΞΙΣΩΣΗ

ποτελούν τους νόμους της διαγραφής.

• ) β {α2 >β2 ανα>β�Ο IV α> <:::> α2 <β2 ανΟ�α>β

Δηλαδή αν υψώσουμε τα μέλη μιας ανισότη­τας στο τετράγωνο η ανισότητα δεν μεταβάλλεται όταν οι όροι της είναι θετικοί, ενώ αλλάζει φορά αν οι όροι της είναι αρνητικοί. . { 1 1 ->-αν α > β > Ο

v) α > β <::> � � - <-αν Ο > α > β α β

( ' ο β ' 1 1 ενω αν α < < τοτε - < 0 < - ). α β

Δηλαδή aντιστρέφοντας τους · όρους μιας ανι-σότητας αλλάζει φορά αν οι όροι της είναι ομόση­μοι, ενώ δεν μεταβάλλεται αν οι όροι της είναι ε­τερόσημοι.

Σημείωση: Η απόδειξη των (iv) και (v) στη συνέχεια στο παράρτημα.

vi) Αν α > β και β > γ τότε α > γ (μεταβα-τική ιδιότητα). vii) • Α ν α > β και γ � δ τότε: α+ γ > β+ δ

ενώ • αν α > β και γ > δ τότε:

(αν α, β, γ, δ θετικοί αριθμοί). αγ > βδ

Δηλαδή αν προσθέσουμε κατά μέλη ομοιό­στροφες ανισότητες (με την ίδια φορά) προκύπτει ομοιόστροφη με αυτές ανισότητα, ενώ αν πολλα­πλασιάσουμε κατά μέλη ομοιόστροφες ανισότητες με όλους τους όρους θετικούς αριθμούς προκύπτει ομοιόστροφη με αυτές ανισότητα.

Προσοχή: Σχετικά με την ιδιότητα (νίί) ισχύει το παρακάτω:

Για να πολλαπλασιάσουμε κατά μέλη δυο ομοιόστροφες ανισότητες και να προκύψει ο­μοιόστροφη ανισότητα με αυτές αρκεί:

ί) Τρεις από τους όρους της να εί­ναι θετικοί

ίί) Μια «διαγώνιο9> να είναι θετική ..fj > .J2 � ΑΝΙΣΟΤΗΤ Α χ + 3 > 8 � ΑΝΙΣΩΣΗ π.χ. αν α > β και γ > δ και α, δ > Ο ή <2> Έτσι, ένας αριθμός α είναι μεγαλύτερος από το Ο αν β, γ > 0 τότε: αγ > βδ .

και μόνο αν η διαφορά α - Ο = α είναι θετικός αριθμός. Άρα: Θετικοί αριθμοί είναι οι μεγαλύτεροι τσυ ο. Απόδειξη του (ίί)

ΕΥΚΛΕΙΔΗΣ Β' λστ' τ.l/33

Page 36: Ευκλειδης Β 45

Μαθηματικά για την Α' Λυκείου

Έστω α > β και γ > δ και α, δ θετικοί α­ριθμοί.

Από: �: �} παίρνουμε αδ > βδ

�; �} παίρνουμε αγ > αδ

λρα: (μεταβατική ιδιότητα): αγ > βδ . Ομοίως αν β, γ θετικοί αριθμοί. Η απόδειξη της (i) αφήνεται ως άσκηση από

τον αναγνώστη! Γ . Λ υ μ έ ν ε ς Α σ κ ή σ ε ι ς κ α ι μ ε θ ο δ ο ­

λ ο γ ί ε ς Δύο μορφές ασκήσεων που συναντάμε συχνά

είναι οι εξής: ί) δείξτε ότι: Α� Β ή ASB ή

Α> Β ή Α<Β (χωρίς να δίνεται κάποια υπόθεση)

π. χ. : Δείξτε ότι α2 � 2α -ι για κάθε α e R .

Στην περίπτωση αυτή μετατρέπουμε ισοδύνα­μα την αποδεικτέα σχέση στην μορφή: Α-Β�Ο ή A-BSO ή Α-Β>Ο ή Α-Β<Ο (δηλαδή μεταφέρουμε όλα τα μέλη της ανισότητας στο πρώτο της μέλος). Για την απόδειξη της τελευταί­ας σχέσης χρησιμοποιούμε τα αξιώματα (κανόνες) που ισχύουν: • αν α > Ο και β > Ο τότε α + β > Ο • αν α < Ο και β < Ο τότε α + β < Ο

• αν α, β ομόσημοι τότε αβ > Ο και * > Ο

αν α, β ετερόσημοι τότε αβ < Ο και � < Ο β

• για κάθε α Ε R ισχύει α2 � Ο . Παράδειγμα ι : Δείξτε ότι α2 � 2α - ι για

κάθε α Ε R .

Λύση

α2 � 2α - 1 <=> α2- 2α + 1 � Ο <=> (α - 1)2 το οποίο ισχύει!

Παρατήρηση: Ισχύει: χ2 + y2 � Ο για κάθε χ, y Ε R (βλέπε παράδειγμα 3).

Όμως αν χ -=ι:. Ο ή y -=ι:. Ο (δηλαδή τα χ, y όχι συγχρόνως Ο) τότε χ2 + y2 > Ο (βλέπε παράδειγμα 2).

Παράδειγμα 2: Δείξτε ότι 2α2 > 2α - ι για κάθε α Ε R .

Λύση

2α2 > 2α - 1 <=> 2α2 - 2α + 1 > Ο <=> α2 + (α2 - 2α + 1) > 0 <=> α2 + (α - 1)2 > 0 το οποίο ισχύει, αφού τα α και α - 1 δεν μηδενίζονται συγ­χρόνως.

Παράδειγμα 3: Δείξτε ότι

2x(x-y)�2x-(y2 +ι) για κάθε x, y E R .

Λύση

2x (x - y) � 2χ - (/ + 1) <=> 2χ 2 - 2xy � 2χ - y2 -1 <=> 2χ2 - 2xy - 2χ + y2 + 1 � Ο <=> (χ2 - 2xy + y2) + {χ2 - 2χ + 1) � Ο <=> (χ - y)2 + (χ - 1)2 � Ο ισχύει! Παράδειγμα 4: Δείξτε ότι χ2 + xy + y2 � Ο . Λύση

1 χ2 + xy + y2 = - (2χ2 + 2xy + 2y2) = 2 � [<χ2 + 2xy + y2) + χ2 + Υ2 ] =

� [<χ + y)2 + χ2 + Υ2 ] � Ο

(ως άθροισμα μη αρνητικών αριθμών) Ερώτηση: Πότε ισχύει: χ2 + xy + y2 > Ο ; 11) Δίνεται μια ανισοτική σχέση και ζητεί­

ται να δειχθεί μια άλλη. π. χ.: Δείξτε ότι αν α < ι < β τότε

αβ+ ι < α + β . Στην περίπτωση αυτή μετατρέπουμε την απο­

δεικτέα σχέση Α � Β στην ισοδύναμή της Α-Β � Ο κ.λπ. όπως προηγουμένως και για την α­πόδειξή της εκτός από τα γνωστά αξιώματα ισχύει και η ανισοτική σχέση της υπόθεσης.

Παράδειγμα ι : Δείξτε ότι αν α < ι < β τότε αβ + ι < α + β .

Λύση Η αποδεικτέα σχέση ισοδύναμα γίνεται: αβ + 1 < α + β <=> αβ - α + 1 - β < Ο <=> α (β - 1) - (β - 1 ) < Ο <::::> (α - 1) (β - 1) < 0 (2)

Από την υπόθεση όμως έχουμε: α < 1 ΑΡ Α: α - 1 < Ο και 1 < β ΑΡ Α: β - 1 > Ο

0 Άρα η (2) ισχύει αφού οι αριθμοί α - 1 και β - 1 είναι ετερόσημοι.

Παράδειγμα 2: Αν α < 2 δείξτε ότι α3 < 2α2 - α + 2 .

Λύση Η αποδεικτέα σχέση ισοδύναμα γίνεται: α3 < 2α2 - α + 2 <=> α3 - 2α2 + α - 2 < 0 <=> α2 (α - 2) + (α - 2) < 0 <=> (α2 + 1) (α - 2) < 0 (3) Όμως από την υπόθεση έχουμε: α < 2 ΑΡΑ:

α - 2 < Ο ενώ α2 + 1 > Ο ως άθροισμα ενός μη αρνητικού και ενός θετικού αριθμού.

Άρα η (3) ισχύει, αφού οι αριθμοί α2 + 1 και α - 2 είναι ετερόσημοι.

111) Εκτός από τις δύο προηγούμενες γενι­κές μεθόδους για την απόδειξη ανισοτήτων

ΕΥΚΛΕΙΔΗΣ Β' λστ' τ.l/34

Page 37: Ευκλειδης Β 45

Μαθηματικά για την Α' Λυκείου

μπορούμε χρησιμοποιώντας τις ιδιότητες των ανισοτήτων που αναφέραμε να αποδεικνύουμε άλλες ανισότητες.

Παράδειγμα 1 : Να δειχθεί ότι αν α < β τότε α + β

α < -- < β . 2

Λύση Από α < β προκύπτει 2α < α + β (προσθέσαμε

και στα δύο μέλη το α) Από α < β προκύπτει α + β < 2β (προσθέσαμε

και στα δύο μέλη το β) Άρα 2α < α+ β < 2β (μεταβατική ιδιότητα)

δηλαδή α < α + β < β (διαιρέσαμε και τα 2 . τρία μέλη δια του 2).

Παράδειγμα 2 : Δείξτε ότι 2.J3 < 3Ji . Λύση 2J3< 3J2 � (επειδή 2JJ > ο, 3.J2 > ο υψώνουμε στο τε­

τράγωνο και τα δύο μέλη)

(2J3)2 < (3J2)2 � 4 · 3 < 9 · 2� 12 < 1 8 , που ισχύει! Δ. Παράρτημα Παραθέτουμε τώρα τις αποδείξεις των ιδιο­

τήτων (iv) και (ν) που αναφέραμε στην αρχή του άρθρου.

1 ) i) α < β � α2 < β2 αν ο ::; α < β

ii) α < β � α2 > β2 αν α < β ::; Ο Απόδειξη

α ;:: ο i) α < β � α2 < αβ

β > Ο α< β � αβ < β2

Άρα α2 < β2 (μεταβατική ιδιότητα) Α ντίστροφα:

αz < βz � αz - βz < Ο � (α - β) ( α + β) < Ο Όμως: α + β > Ο (άθροισμα θετικών)

Άρα: α - β < Ο Όμοια για την (ii). 2) i) α < β �.!. <!. αν Ο < α < β ή α < β < Ο

α β

ii) α < Ο < β�.!.< ο<!. α β

Απόδειξη αβ > Ο α β 1 1 1 1 i) α < β � - < - � - <- δηλαδή - >-

αβ αβ β α α β Είτε ο < α < β είτε α < β < Ο ισχύει αβ > Ο .

iii) Προφανής αφού οι aντίστροφοι αριθμοί είναι ομόσημοι!

Ε . Α σ κ ή σ ε ι ς γ ι α λ ύ σ η 1. Ν α αποδειχθεί ότι: i) (α + β)2 2: 4αβ, α, β ε R

ii) α2 + β2 + γ2 :=:: αβ + βγ + αγ, α, β, γ ε R 2. Αν α, β θετικοί με αβ = 1 τότε

(1 + α) (1 + β) :=:: 4 .

3. Αν xy > ο τότε � + ! :::: 2 . Υ χ

4. Δείξτε ότι ( 1 + χ3 )2 < ( 1 + χ2 )3 •

5. Αν χ, y θετικοί τότε: -χ + y :::; �·

(x--:2_+_1_)_(y--:2-+_1_) .

Γ · .. . . Α " ;.,ξ, Λ ί εωμετρια της _ τu __ :ης του _ _ υκε �ου Βασικά Γεωμετρικά σχήματα

της Αγγελικής Βλάχου Αγαπητοί φίλοι μαθητές, στην πρώτη σας ουσιαστική επαφή με τη Γεωμετρία στην Α' τάξη του Λυ­

κείου, είναι σημαντικό αφενός μεν να κατανοήσετε τις αρχικές έννοιες, αφετέρου να προσέξετε όλα όσα αναφέρει το σχολικό σας βιβλίο για τα ευθύγραμμα τμήματα, τις γωνίες και τον κύκλο.

Η γεωμετρία σας είναι μία αλυσίδα θεμάτων όπου το ένα ακολουθεί το άλλο και για κάθε επόμενο βήμα είναι απαραίτητο το προηγούμενο. Έτσι τίποτα δεν πρέπει να παραβλέπουμε από το κομμάτι της θεωρίας, αν θέλουμε να αντιμετωπίζουμε σωστά τη συνέχεια της ύλης μας και τις ασκήσεις.

Για παράδειγμα, πρέπει να γνωρίζουμε μα και να κατασκευάζουμε τα διαδοχικά ευθύγραμμα τμήμα­τα, τις εφεξής γωνίες, είδη και σχέσεις γωνιών, σχέσεις τόξων, κυρτή και μη κυρτή γωνία κλπ.

Η μελέτη τόσο της θεωρίας όσο και των ασκήσεων πρέπει να γίνεται γραπτά. Το παρόν άρθρο έχει ως σκοπό να σας βοηθήσει να δείτε πώς με τη βοήθεια των συλλογισμών πάνω

στα δεδομένα και τα ζητούμενα των ασκήσεων καταλήγουμε σωστά στη λύση τους.

ΕΥΚΛΕΙΔΗΣ Β' λστ' τ.l/35

Page 38: Ευκλειδης Β 45

Άσκηση ι Να βρεθεί το πλήθος των ευθειών που ορί­

ζουν ν σημεία μη συνευθειακά ανά τρία, με ν � 3 , στη συνέχεια να βρεθεί το πλήθος των ευθειών που ορίζουν 15 σημεία μη συνευθειακά ανά τρία.

Λύση

Έστω Α1 , Α2 , Α3 ' .. . , Αν-ι ' Αν σημεία. Γνωρίζουμε ότι 2 σημεία του ε­mπέδου ορίζουν μία κ:αι μόνο ευθεία.

Το σημείο Α1 ενώνεται με τα Α2 , Α3 , . . . , Αν-ι , Αν σημεία, δηλαδή ενώνεται με ν -1 στο πλήθος σημεία και ορίζει ν -1 ευθείες.

Έτσι έχουμε: Το σημείο Α1 ενώνεται με ν - 1 σημεία και

ορίζει ν - 1 ευθείες. Το σημείο Α2 ενώνεται με ν - 1 σημεία και

ορίζει ν - 1 ευθείες.

Το σημείο Αν ενώνεται με ν - 1 σημεία και ορίζει ν - 1 ευθείες.

Όμως την ευθεία Α1Α2 την έχουμε πάρει 2 φορές δηλαδή και ως ευθεία Α2Α1 •

ν · (ν -1) Συνεπώς τα ν σημεία ορίζουν ευ-2 θείες.

Για ν = 15 σημεία μη συνευθειακά ανά τρία ' 15( 15 - 1) 210 εχουμε = - = 105 στο πλήθος ευθεί-ες.

2 2 Άσκηση 2 Έστω α, β, γ γνωστά ευθύyραμμα τμήματα

με α>β>y, να κατασκευάσετε τα ευθύγραμμα τμήματα

i) 2α + Ρ + 3γ ii) 4α - 3β . Λύση Έστω ΑΒ = α , ΓΔ = β και ΕΖ = γ με

ΑΒ > ΓΔ > ΕΖ . Α α β

Γ� ιμ...z

ί) Σε ευθεία ε παίρνουμε τα ευθύγραμμα τμήματα: ΚΛ = 2α , ΛΜ = β και ΜΝ = 3γ .

__κ 2α Λ� ε • • • •

και έχουμε: ΚΝ = ΚΛ + ΛΜ + ΜΝ = 2α + β + 3γ

συνεπώς το ΚΝ είναι το ζητούμενο ευθύγραμμο τμήμα.

ii) Εφόσον α > β ισχύει 4α > 3β συνεπώς ο-ρίζεται η διαφορά 4α- 3β .

Έστω ευθύγραμμα τμήματα ΡΣ και ΡΤ επί ευ­θείας ε με ΡΣ = 4α και ΡΤ = 3β με 4α > 3β . __p 3Β Τ 4α - 3Β Σ

ε • • •

Τότε έχουμε ΡΣ = ΡΤ + ΤΣ 4α = 3β + ΤΣ

συνεπώς ΤΣ = 4α -3β έτσι το ΤΣ είναι το ζητού­μενο ευθύγραμμο τμήμα.

Άσκηση 3 Έστω Ο το μέσον ενός ευθυγράμμου τμή­

ματος ΑΒ. Στις προεκτάσεις των ΑΒ και ΒΑ παίρνουμε τα σημεία Κ και Λ αντίστοιχα. Α ν ι-

σχύει ΒΚ = .! ΑΒ και ΑΛ = ΒΚ

και Μ είναι το 3 2

μέσον του ΟΜ.

ΛΚ να υπολογίσετε το μήκος του

Λύση Λ Α Ο Μ Β κ • • • • • •

Κατασκευάζουμε το σχήμα σύμφωνα με τα δεδομένα της άσκησης (προσέχουμε ώστε να πά­ρουμε τα σημεία Κ και Λ στις προεκτάσεις των ΑΒ και ΒΑ).

ΑΒ Έχουμε: ΑΟ = ΟΒ = - . 2 ΒΚ =�ΑΒ} �ΑΒ

ΑΛ = �Κ άρα ΑΛ = Β� = 3 � =�ΑΒ .

2 1 Συνεπώς: ΒΚ = � ΑΒ, ΑΛ = _!_ ΑΒ (1 ). 3 3 Παρατηρούμε ότι:

1 ΑΒ ( 1 1 ) 5 . ΛΟ = ΛΑ + Α0 = "3ΑΒ +τ = "3+"2 ΑΒ = 6ΑΒ

και ΟΚ=ΟΒ+ΒΚ= ΑΒ +�ΑΒ=(_!_+�)ΑΒ=2ΑΒ 2 3 2 3 6 άρα ΛΟ < ΟΚ συνεπώς το μέσον Μ του ΛΚ βρί­σκεται δεξιά του μέσου Ο του ΑΒ.

1 4 Επίσης ΛΒ = ΛΑ + ΑΒ = -ΑΒ + ΑΒ = -ΑΒ 3 3 και ΒΚ = � ΑΒ έτσι ΛΒ > ΒΚ άρα το μέσον Μ 3 του ΛΚ βρίσκεται αριστερά του σημείου Β.

Το μέσον Μ του ΛΚ βρίσκεται μεταξύ των Ο και Β. Έχουμε:

ΛΜ = ΛΟ + ΟΜ έτσι ΟΜ = ΛΜ - ΛΟ (2) (ΛΜ > ΛΟ )

ΕΥΚΛΕΙΔΗΣ Β' λστ' τ.l/36

Page 39: Ευκλειδης Β 45

Μαθηματικά ΎUI την Α' Λυκείου

Όμως ΛΜ = ΛΚ = ΛΑ +ΑΒ + ΒΚ = 2 2

_!_ΑΒ +ΑΒ +�ΑΒ (! + 1 +�)ΑΒ = 3 3 = 3 3 = ΑΒ (3) 2 2 και ΛΟ =�ΑΒ (4). 'Ετσι, απ' την (2), 'λiJγω της

6 (3) και (4) παίρνουμε: ΟΜ = ΑΒ -�ΑΒ = 6 = (1 -�)AB = (�-�)AB =iAB .

λ 4 Μ�� γωνίας, το άθροισμα του τρι-

πλασίου της συμπληρωματικής και του διπλα­σίου της παραπληρωματικής της ισούται με 4 ορθές γωνίες. Να βρείτε το μέτρο αυτής της γω­νίας σε μέρη ορθής και σε μοίρες.

Λύση Έστω ω < 1 L (μία ορθή)

8 παραπληρωμαnκή της ω : 2L - ω ή 1 80° - ω • σuμπληρωμαnκή της ω : 1L - ω ή 90° -ω 8 τριπλάσιο σuμπληρωμαnκής: 3 ( 1 L - ω) 8 διπλάσιο παραπληρωμαnκής: 2( 2L - ω)

3(1L - ω)+ 2(2L - ω) = 4L <::)

3L - 3ω+ 4L - 2ω = 4L <::> 5ω = 3L <::> ω =� L 5

ω = � της ορθής ή ω = � . 90° = 3 · 1 8° = 54 ο • 5 5 λσιςηση 5 Από τον κεντρικό σταθμό στο μετρό μιας

πόλης ξεκινάνε οι γραμμές Οχ, Oy, Oz, Οω οι οποίες οδηγούν σε 4 διαφορετικές περιοχές της πόλης. Οι γραμμές Oy και Oz σΧηματίζουν ορθή γωνία ενώ οι Οχ και Οω είναι ημιευθείες αντι­κείμενες.

α) Να αναφέρετε τρία ζεύγη γωνιών οι ο­ποίες να είναι εφεξής.

Λ Λ β) Να δείξετε ότι οι γωνίες xOy και zΟω

είναι συμπληρωματικές. Λ ' ουση . , θ ' . , ι γραμμες ειναι ημιευ ειες με κοινη αρχη το

σημείο Ο. Η Oy θα είναι κάθετη στην Oz και Λ

χΟω =L ευθεία γωνία. Λ Λ

α) xOy εφεξής με την yOz Λ Λ

yOz εφεξής με την zΟω

Λ Λ xOy εφεξής με την yΟω .

β) Αρκεί να δείξου-Λ Λ

με όn: xOy+ zΟω = 1L . Λ

Όμως: χΟω = 2L ή Λ Λ Λ

xOy+ y0z+ zΟω = 2L ή

Λ Λ Λ Λ

z

x0y+ 1L + z0ω = 2L ή x0y+ z0ω = 2L - 1L ή Λ Λ

xOy+ zΟω = 1L . Άσκηση 6

Λ Λ Δίνονται οι διαδοχικές γωνίες ΑΟΒ , ΒΟΓ ,

Λ Λ ΓΟΔ και ΔΟΑ η πρώτη είναι ορθή, η δεύτερη

2 θ ' , , 5

τα - της ορ ης και η τριτη ειναι τα - της ορ-3 6

θής. Καλούμε ΟΚ τη διχοτόμο της γωνίας Λ Λ

ΑΟΒ , ΟΛ τη διχοτόμο της γωνίας ΒΟΓ και Λ

ΟΜ τη διχοτόμο της γωνίας ΛΟΓ . i) Να δείξετε ότι οι ημιευθείες ΟΚ και ΟΔ

είναι αντικείμενες. Λ

ii) Να βρείτε το μέτρο της γωνίας ΑΟΔ •

iii) Να δείξετε ότι οι ημιευθείες ΟΜ και ΟΔ είναι κάθετες.

Λύση Λ Λ

AOB = 1L ή ΑΟΒ = 90° Λ 2L ΒΟΓ = - ή 3

ΒΟΓ = � . 90° = 60° 3 Λ 5 L ΓΟΔ = - ή 6

ΓΟΔ =� · 90° = 75° 6 Κατασκευάζουμε προσεκτικά το σχήμα

και παρατηρούμε ότι

ΑΟΔ = 4L - ( ΑΟΒ+ ΒΟΓ+ ΓΟΔ ) =

- 4L [1L 2L 5LJ - 4L 1 5L - 9L ' 1350 - - +- +- - -- - - η 3 6 6 6 Φέρουμε τις διχοτόμους ΟΚ, ΟΛ, ΟΜ των

Λ Λ Λ ΑΟΒ , ΒΟΓ και ΛΟΓ .

ΕΥΚΛΕΙΔΗΣ Β' λστ' τ.Ι/37

Page 40: Ευκλειδης Β 45

Μαθηματικά yια την Α' Λυκείου 1\ 1\ 1\ 1 1\ 1\ 1\

i) ΚΟΔ = ΚΟΒ+ ΒΟΔ =-ΑΟΒ+ ΒΟΓ + ΓΟΔ = 2 1 2L 5L 3L 4L 5 L 12L =-1L +- + - =- +- +- = - = 2L άρα 2 3 6 6 6 6 6 1\

ΚΟΔ = 2L άρα ΟΔ, ΟΚ αντικείμενες ημιεuθείες.

ii) Έχουμε ήδη βρει την ΑΟΔ = 2 L ή 135°. 6 1\

iii) Αρκεί να δείξουμε ότι ΜΟΔ = 1 L . 1\ 1\ 1\ 1 1\ 1\

ΜΟΔ = ΜΟΓ + ΓΟΔ =-ΛΟΓ + ΓΟΔ = 2 1 1 1\ 1\ 1 2L 5L 2 L 5L = - ·- · ΒΟΓ+ ΓΟΔ = - · - + - = - +- = 2 2 4 3 6 12 6 2 L lOL 12L = +- = - = 1L άρα ΟΜ και ΟΔ κάθετες. 12 12 12

Άσκηση 7 Σε κύκλο (Ο,ρ) θεωρούμε τα διαδοχικά ση­

μεία Α, Β, Γ και Δ. Να βρεθούν τα μέτρα των ..-... ..-. ..-.. ...-....

τόξων ΑΒ , ΒΓ , Γ Δ και ΔΑ αν γνωρίζουμε ότι είναι ανάλογα με τους αριθμούς 2, 3, 4 και 9 α­ντίστοιχα.

Λύσ Α ν χ, y, ω ανάλογα με τους α, β, γ τότε χ Υ ω , χ y ω χ + y + ω - = - = - και ισχυει: - = - = -= . α β γ α β γ α + β + γ

Β

Γ --- ---

Αφού τα μέτρα των τόξων ΑΒ , ΒΓ , Γ Δ και ΔΑ είναι ανάλογα με τους αριθμούς 2, 3, 4 και 9 έχουμε:

...-.. ..-.. ..-.. ...-...

και ΑΓ = α0 , ΒΓ =β0 , ΓΔ =γ0 και ΔΑ = δ0 αο βο γο δο ' Έχουμε - = - = - = - η 2 3 4 9

αο βο γο δο αο +βο + γο + δο 3600 ' - = - = - = - = = -- = 20° ε 2 3 4 9 2 + 3 + 4 + 9 18 αο βο τm 2 = 20° άρα α0 = 40° , 3 = 20° άρα β0 = 60°

ο δο : = 20° άρα γο = 80° , 9 = 20° άρα δ0 = 1 80° . .�σκηση 8 θεωρούμε αμβλεία γωνία xOy και τις ε-

σωτερικές ημιευθείες ΟΑ, ΟΒ έτσι ώστε

ΟΑ .L Οχ , ΟΒ .L Oy . Να αποδείξετε ότι: 1\ 1\

i) οι γωνίες xOy και ΑΟΒ έχουν την ίδια δι-χοτόμο,

1\ 1\ ii) οι γωνίες xOy και ΑΟΒ είναι παραπληρω-

ματικές. Λύση Δύο γωνίες έχουν κοινή διχοτόμο.

}ιο> Θεωρούμε τη διχοτόμο της μιας γωνίας και αποδεικνύουμε ότι είναι διχοτόμος και της άλλης γωνίας.

}ιο> Όμοια αντιμετωπίζουμε το πρόβλημα ότι δύο ευθύγραμμα τμήματα έχσυν κοινό μέσο. Θε­ωρούμε το μέσο του ενός ευθύγραμμου τμή­ματος και αποδεικνύουμε ότι είναι μέσον και του άλλου.

Α i) Σχεδιάζουμε δ 1\ Β αμβλεία γωνία xOy

χ

και φέρουμε τις ημι­ευθείες ΟΑ, ΟΒ με ΟΑ .l Οχ , ΟΒ .L Oy .

1\ 1\ 1\ Έχουμε χΟΑ = χΟΒ+ ΑΟΒ

Λ Λ Λ Λ Λ Λ Άρα xCB=xO\-ACB=sσ'-& (1) και }03=}0\.+ACB

1\ 1\ 1\ Συνεπώς yOA = yOB-ΑΟΒ = 90° - ω (2).

Επομένως από (1) και (2) παίρνουμε 1\ 1\

χ0Β = y0Α = ψ . 1\

Έστω Οδ η διχοτόμος της ΑΟΒ θα δείξουμε 1\

ότι είναι και διχοτόμος της xOy . Έχουμε 1\

Αοδ = δοΒ = ΑΟΒ = ω (3). 2 2 1\ 1\ 1\ (ι),(3) ω χΟδ = χΟΒ+ ΒΟδ = 90° - ω + - =

2 = 9οο - 2ω + ω = 9οο - ω

2 2 2 1\ 1\ 1\ (2),(3) ω ω y0δ = y0Α+ Α0δ = 90° - ω + - = 90° -- συ-2 2

1\ 1\ 1\ νεπώς χΟδ = yΟδ άρα Οδ διχοτόμος και της xOy .

1\ 1\ ii) Έχουμε xOy+ ΑΟΒ =

1\ 1\ 1\ 1\ 1\ χ ΟΑ+ AOy+ ΑΟΒ = χΟΑ + yOB = 90° + 90° = 1 80°

Ασκήσεις προς εξάσκηση των Μαθητών 1) Σε ευθεία παίρνουμε διαδοχικά τα ση-

ΕΥΚΛΕΙΔΗΣ Β' λστ' τ.l/38

Page 41: Ευκλειδης Β 45

Μαθηματικά για την Α· Λυκείου

μεία Μ, Κ, Λ, Ν έτσι ώστε να είναι ΚΛ = 2α ,

ΛΜ = 4α και ΜΝ = 6α όπου α η μονάδα μέ­τρησης των ευθυγράμμων τμημάτων. Να δείξε­τε ότι τα σημεία Κ και Λ είναι μέσα των ευθυ­γράμμων τμημάτων ΛΜ και ΚΝ αντίστοιχα.

Υπόδειξη Μ Κ Λ Ν

Κ μέσον ΛΜ αρκεί ΜΚ. = ΚΑ Λ μέσον ΚΝ αρκεί ΚΑ = ΛΝ ΜΚ = ΜΑ - ΚΑ = · · · (ΜΑ > ΚΑ) ΛΝ = ΜΝ - ΜΑ = · · · (ΜΝ > ΜΑ) 2) Σε ευθεία παίρνουμε διαδοχικά τα ση­

μεία Α, Β, Γ, Δ έτσι ώστε ΑΒ = 8 , ΑΔ = 52 ,

ΒΓ =!Γ Δ . Να βρεθεί το μήκος του ευθύyραμ-4

μου τμήματος ΑΓ. Υπόδειξη

Α Β Γ Δ

Παρατηρούμε ότι: ΑΒ + ΒΓ + Γ Δ = ΑΔ όμως

ΒΓ = _!_ΓΔ . ΑΒ + _!_ΓΔ + ΓΔ = ΑΔ 4 4

8 + ( �+ 1)rΔ = 52

8 +�ΓΔ = 52 άρα ΓΔ = 35, 2 4

ΑΓ = ΑΒ + ΒΓ = . . .

3) Μία γωνία είναι ίση με τα .! της ορθής. 3

Να βρείτε την παραπληρωματική της και την συμπληρωματική της σε μοίρες και σε μέρη ορ­θής.

4) Η διαφορά δύο παραπληρωματικών γω-

νιών είναι ίση με τα .! της ορθής. Να βρεθούν 5

οι γωνίες. Υπόδειξη 2 θ ' - της ορ ης 5

Έστω ω η μία γωνία η άλλη θα είναι 1 80° - ω και θα έχουμε: ω- ( 1 80° - ω) = 36° κλπ.

5) Οι διχοτόμοι δύο εφεξής γωνιών σχημα-4

τίζουν γωνία 45°. Αν η μία ισούται με τα - της 5

άλλης να βρεθούν οι δύο γωνίες. Υπόδειξη

'Ε Α • • .(."\ "\ . θ • 4ω ο στω ω η μια γωνια η UΙ\Λη α ειναι 5 . ι

διχοτόμοι τους σχηματίζουν γωνία 45° συνεπώς

4ω ω 5 �ο · ω � �o · � + � = �if 2 +2 = η 2 + ιο = η

ή 9ω = 450° κλπ. 6) Από σημείο Ο μιας ευθείας ΑΒ φέρουμε

προς το ίδιο μέρος της ΑΒ τις ημιευθείες ΟΓ λ λ

και ΟΔ έτσι ώστε οι γωνίες ΑΟΓ , ΓΟΔ και λ

ΔΟΒ να είναι διαδοχικές. λ λ λ

i) Αν οι γωνίες ΑΟΓ , ΓΟΔ , ΔΟΒ είναι ανάλογες με τους αριθμούς 1, 2, 3 να βρεθούν τα μέτρα τους.

ii) Με τα ίδια δεδομένα και αν ΟΕ, ΟΖ είναι λ λ

οι διχοτόμοι των ΑΟΓ και ΔΟΒ αντίστοιχα και λ λ

ΓΟΔ = 60° να βρείτε τη γωνία ΕΟΖ . Υπόδειξη

Λ Έστω α, β, γ τα μέτρα των γωνιών ΑΟΓ ,

ΓΟΔ ΔΟΒ τότε � =� =1 = α+β+γ = 1800 =30° ' 1 2 3 1+2+3 6

Γ

Δ

Β Λύση

Λ Λ Λ

Α

ΑΟΓ + ΓΟΔ+ ΔΟΒ = 180° άρα Λ Λ Λ

Δ

z

Β

ΑΟΓ + ΔΟΒ = 180° - ΓΟΔ = 1 80° - 60° = 120° Λ Λ Λ Λ

Όμως ΕΟΖ = ΕΟΓ + ΓΟΔ+ ΔΟΖ = Λ Λ Λ Λ

= ΑΟΓ + 6Οο + ΔΟΒ = ΑΟΓ+ ΔΟΒ + 6Οο = 2 2 2

1200 + 60° ='60° + 60° = 120° . 2

7) Θεωρούμε κύκλο (O,R) και μία διάμε­τρό του ΑΒ. Ο κύκλος (Α,ρ) τέμνει το ένα ημι­κύκλιο διαμέτρου ΑΒ στο σημείο Γ και ο κύ­κλος (Β,ρ) τέμνει το άλλο ημικύκλιο διαμέτρου ΑΒ στο σημείο Δ ( ρ < R ). Να αποδείξετε ότι τα σημεία Γ, Δ είναι aντιδιαμετρικά στον κύκλο (O,R).

Υπόδειξη

ΕΥΚΛΕΙΔΗΣ Β ' λστ' τ.l/39

Page 42: Ευκλειδης Β 45

Μαθηματικά -yια την Α' Λύκείου

Φέρουμε τις ακτίνες ΟΓ και ΟΔ. Έχουμε ΑΓ = ΒΔ ως ακτίνες των (Α,ρ) και

..-.. ...-... Λ Λ (Β,ρ ). Άρα ΑΓ = ΒΔ άρα ΑΟΓ = ΒΟΔ ως επίκε-ντρες που βαίνουν σε ίσα τόξα του (O,R) .

Λ Λ Λ Όμως ΑΟΒ = 1 80° ή ΑΟΔ+ ΒΟΔ = 180° ή

Λ Λ Λ ΑΟΔ+ ΑΟΓ = 1 80° ή ΓΟΔ = 1 80° συμπέρασμα.

.. ΣV"G"·�, ... ,.rτ', Mf"E''T' ... . τ,:rv Ε• ''-ΡΩ�D ... ··�··ΩΝ· τrι.,�.-r .. '"f'lrlll\ .. 11 4, --���ιll�ι l 1. . ..ιι:!ιι _! Λlί:iiιi, .ι[ . ' ι .1! Q/ .IL .11 JΓ .nJ.,Y.IIJl'f'.l!ι ι Ι . i .Jl'f'JLD!Jl�.Jl..λ:_ �,ιιΙJL.,!

Κάθε χρόνο, στο ξεκίνημα κιόλας της διδα­σκαλίας της Ευκλείδειας Γεωμετρίας, διαmστώνω ( φαντάζομαι και άλλοι συνάδελφοι) τη δυσχέρεια των μαθητών να αντιμετωπίσουν προβλήματα που αναφέρονται σε σχέσεις μεταξύ ευθυγράμμων τμημάτων, όχι βέβαια εντελώς απλά. Αυτή η δυ­σχέρεια κατά ένα μέρος οφείλεται στο γεγονός ότι στις λύσεις που βρίσκουν σε διάφορα βιβλία ή που οι ίδιοι εmνοούν, γίνεται χρήση διάφορων τμημά­των του σχήματος, η οποία ναι μεν οδηγεί στο α­ποτέλεσμα, αλλά κατά την πορεία δεν έχει πείσει τους μαθητές ούτε για το αν είναι αναγκαία ούτε για το αν θα καταλήξει τελικά στο στόχο.

Πιστεύω ότι μια ενιαία μέθοδος για την αντι­μετώmση αυτών των θεμάτων θα διευκολύνει αι­σθητά τους μαθητές. Θα βασιστούμε γι' αυτό στην παρακάτω βασική πρόταση.

Βασική πρόταση: Επί ευθείας (ε) θεωρούμε τα σημεία Α, Β, το μέσο Μ του ΑΒ και σημείο Σ

, ΑΒ Τ ' , ΣΜ ΣΑ + ΣΒ

εκτος του . οτε ισχυει = 2

Η πρόταση αυτή δεν είναι βέβαια θεώρημα, αλλά άσκηση στο σχολικό βιβλίο, όμως μπορεί εύκολα να αποδειχθεί και στη συνέχεια να χρησι­μοποιηθεί σε όλες τις πολύπλοκες ασκήσεις που α­ναφέρονται σε μέσα ευθυγράμμων τμημάτων.

(ε)

Σ Α Μ Β

Πράγματι στο σχήμα έχουμε: ΣΜ = ΣΑ + ΑΜ και ΣΜ = ΣΒ - ΒΜ , οπότε 2ΣΜ = ΣΑ + ΣΒ και ΣΜ = ΣΑ + ΣΒ . Ομοί-2 ως εργαζόμαστε όταν το Σ βρίσκεται προς το μέρος του Β.

Παραθέτω ενδεικτικά δύο εφαρμογές, για να φανεί η ευκολία που παρέχει η χρήση αυτής της πρότασης.

Εφαρμογή lη: Επί ευθείας (ε) θεωρούμε τα διαδοχικά τμήματα ΑΒ, ΒΓ, Γ Δ και τα μέσα Μ, Ν των ΑΓ, ΒΔ αντιστοίχως. Α ν ΑΒ = κ και

ΓΔ = λ τότε ΜΝ = κ + λ . 2

Απόδειξη: Θεωρούμε σημείο Σ στην αντικεί­μενη ημιευθεία της ΑΔ και θέτουμε:

Γιώργος Σ. Τασσόπουλος

Σ Α Β Μ Ν Γ • (ε)

Δ

ΣΑ = α , ΣΒ = β , ΣΓ = γ , ΣΔ = δ

Τότε: ΣΝ = β + δ ΣΜ = α + γ 2 ' 2

ΣΝ > ΣΜ αφού β > α , δ > γ . και

Ά · ΜΝ - ΣΝ ΣΜ - β + δ α + γ _ ρα. - - - 2 - 2 -β + δ - α - γ (β - α) + (δ - γ) κ + λ .:...._ __ ----'- = = = 2 2 2

Εφαρμογή 2η: Επί ευθείας (ε) θεωρούμε τα διαδοχικά τμήματα ΑΒ, ΒΓ και τα μέσα Δ, Ε, Ζ των ΑΒ, ΒΓ, Γ Α αντιστοίχως. Να δείξετε ότι:

α) ΑΔ < ΑΖ < ΑΕ β) Τα τμήματα ΔΕ, ΒΖ έχουν κοινό μέσο. Απόδειξη: Θεωρούμε σημείο Σ στην αντικεί-

μενη ημιευθεία της ΑΓ και θέτουμε: ΣΑ = α , ΣΒ = β , ΣΓ = γ .

α) Τότε α < β < γ και α + β β - α ΑΔ = ΣΔ - ΣΑ = -- - α = 2 2

• • . • . � (ε) Σ Α' Δ ' Β Ζ Ε Γ

ΑΖ = ΣΖ- ΣΑ = α + γ - α = γ - α > β - α = ΑΔ 2 2 2 ' αφού γ > β . ΑΕ = ΣΕ - ΣΑ = β + γ - α = β + γ - 2α =

2 2 (β - α) + (γ - α) > γ - α = ΑΖ αφού β > α. 2 2 '

β) Για να έχουν τα ΔΕ, ΒΖ κοινό μέσο, αρκεί ως γνωστόν να ισχύει ΔΒ = ΖΕ .

, α + β β - α Πραγματι ΔΒ

= ΣΒ - ΣΔ

= β - -2- = l ' ΖΕ = ΣΕ - ΣΖ = β + γ _ α + γ = β - α . Άρα ΔΒ = ΖΕ .

2 2 2

Βασική παρατήρηση: Από το Γυμνάσιο οι μαθητές γνωρίζουν ότι, αν Μ, Ν τα μέσα των πλευρών ΑΒ, ΑΓ ενός τριγώνου, τότε

ΕΥΚΛΕΙΔΗΣ Β' λστ' τ.l/40

Page 43: Ευκλειδης Β 45

Μαθηματικά -yια την Α' Λυκείου

ΒΓ Σ , >. θη , , ΜΝ = - . το σημειο αυτο οι μα τες μπορουν 2 να αποδείξουν μια εκφυλισμένη μορφή αυτής της πρότασης, όταν τα Α, Β, Γ είναι συνευθειακά.

Α Μάλιστα, αντί να διακρίνουν όλες τις mθανές θέσεις των

J-.---_,_N Α, Β, Γ, μπορούν να θεωρήσουν τυχαίο σημείο Σ εκτός όλων

Β Γ των τμημάτων ΑΒ, ΒΓ, Γ Α και να θέσουν ΣΑ = α , ΣΒ = β , ΣΓ = γ . Τότε:

ΜΝ = IΣΜ - ΣΝI = I α + β - α + γ ι = 'β - γ' = ΒΓ 2 2 2 2

. � (ε) Σ Α Ν Γ Μ Β

(ε) Σ Β Μ · Α

Αυτή η πρόταση μας παρέχει την απόσταση των μέσων δύο τμημάτων με κοινό άκρο. Στη 2η εφαρμογή λοιπόν θα μπορούσαμε να πάρουμε α­μέσως για τα μέσα Ε, Ζ των τμημάτων ΓΒ, Γ Α με

ΑΒ κοινό άκρο Γ τη σχέση ΕΖ = - = ΔΒ . Στην 2 1 η εφαρμογή όμως, τα Μ, Ν είναι μέσα των τμημάτων ΑΓ, ΒΔ που δεν έχουν κοινό άκρο. Θε­ωρώντας όμως το μέσο ενός ακόμη τμήματος, π.χ. το μέσο Ο του ΑΒ, έχουμε αμέσως ΜΝ = ΟΝ - ΟΜ .

(ε) • Α Ο Β Μ Ν Γ Δ

Αλλά, τα Ο, Ν είναι μέσα των ΒΑ, ΒΔ με κοινό άκρο Β, οπότε ΟΝ = ΑΔ

. Επίσης, τα Ο, 2 Μ είναι μέσα των ΑΒ, ΑΓ με κοινό άκρο Α, οπότε ΟΜ = ΒΓ . Άρα: 2 ΜΝ = ΑΔ _

ΒΓ = ΑΔ -ΒΓ = ΑΒ + ΓΔ = κ+ λ 2 2 2 2 2

Σημείωση: Α{ρού οι μαθητές εξοικειωθούν με τη μέθοδο, μπορούν σε μερικές περιπτώσεις ως Σ να παίρνουν το ίδιο το Α, οπότε α = Ο . Επίσης, η -βασική πρόταση για τόξο ΑΒ με μέσο Μ και ση-μείο Σ του κύκλου του, εκτός του τόξου, γίνεται: ..- ...---- ΣΑ + ΣΒ ' ' ΑΟΑ Β δ ΣΜ = , ενω για γωνια με ιχο-2 τόμο ΟΜ και ημιευθεία ΟΣ εκτός αυτής, γίνεται: ΣΟΜ =

ΣΟΑ + ΣΟΒ . 'Ετσι, όλες οι ασκήσεις 2 επί των ευθυγράμμων τμημάτων μεταφέρονται αυ­τούσιες στα τόξα και στις γωνίες. Η απόδειξή τους εί­ναι ωτλή αντιγραφή. Εξάλλου, η βασική πρόταση γε­νικεύεται για ΑΜ = λ · ΜΒ, λ > Ο , και παίρνει τη

λ · �Β + ΣΑ Μ , , δη μορφή ΣΜ =

λ + 1 . πορουμε ετσι να -. μιουpyήσουμε γενικεύσεις των προτάσεων mu ισχfJ­ουν όταν λ = 1 και σε πεptπτώσεις ό1WU λ ::J:. 1 .

�imrι\\tWΓ �\ "11" ' ,{\ "Ε'/1�1 �\ "'C''IV'�_ Ι_ 'I:II1"'C' ' ==;� �.I!JJt'Y�.I! � ����

ι. Ποιοι αριθμοί ικανοποιούν τη σχέση 38ν - 36ι � 36ινz ;

2. Να γραφεί ως κλάσμα με ρητό παρονο-

, κλά 2J6 μαστη το σμα .fi +

.J3 +

J5 . 3. Βρείτε τον χ ώστε 64 (χ - ι)3 + 27 = Ο . 4. Να λυθεί η aνίσωση lx - 71 > lx + 21 + lx • 21 . 5. Αν είναι α + β = 5 , αz + αβ = 95 , τότε α = ; 6. Αν ισχύει αβ =6 , αzβ + αβz + α + β =63 ,

υπολογίστε: α z + β z •

. θ , 2Ζ50 7. Ν α συγκριθούν οι αρι μοι: α = ,

β = 3zοο , γ = 5ιsο . 8. Υπολογίστε:

Σ = � 7 - J48 + �.-5 -

----=J-24=4 + � 3 - .J8 .

του Σ. Σκοτίδα

, , αz . pz αβ - βz 9. Να γινουν οι πραξεις: β • z • α αβ -α ι Ο. Δείξτε ότι για κάθε α, β, γ ε R ισz6ει η

σχέση: αz +βz +γz - αβ - βγ - αγ� 3 (β-γ)(α- β) . 11. Αν lx - yl = ly ·ωl = lω - zl = ι τότε x - z = ; ι2. Αν αz +βz + γz = ι , τότε

α4 + (αβ + γ)z + (αγ - β)z = ; r-------:.-, ι 2 200ι2 200ι

13. Υπολογιστε: χ=γι+200ι + 200i + 2002 ·

ι4. Αν Ο < χ < ι , τότε ποιος από τους παρα­κάτω αριθμούς είναι ο μικρότερος;

I α) xz β) χ"1 γ) χ2 δ) x-z ε) χ

ΛΥΣΕΙΣ- ΥΠΟΔΕΙΞΕΙΣ ΘΕΜΑΤΩΝ ΕΞΑΣΚΗΣΗΣ ι. (n - 1 9)2 :::;:; o::::} n = 19

ΕΥΚΛΕΙΔΗΣ Β' λστ' τ.l/41

Page 44: Ευκλειδης Β 45

Μαθηματικά για την Α' Λυκείου

2. J2 +J3 - J5 3. (χ - Ι ) = (-�)3 � x - I = -�� x =-i-4. Α ν χ < - 2 τότε παίρνουμε - 7 < χ < - 2 Αν - 2 � χ < 2 τότε χ < 3 άρα - 2 � χ < 2 Αν 2 � χ < 7 τότε x <f άρα 2 � x <f Α ν 7 � χ τότε χ < - 7 άτοπο Τελικά το σύνολο λύσεων της ανίσωσης είναι:

s = (- 7, f) 5. α (α + β) = 95 � α · 5 = 95 � α = Ι9 6. αβ (α + β) + (α + β) = 63 �

7 (α +β) = 63 � α+β = 9 � α2 + β2 + 2αβ = 8Ι � α2 +β2 = 69

7. α = (25γο = 3250 ' β = (34γο = 8 Ι5ο ' γ = (53γ0 = 12550 , άρα γ > β > α . 8. Σ =�(2 - J3)2 +�(J3 -fi)2 +�.-Cfi-2-- I-)2 =Ι

9. (α-β)(α+β) + β (α-β) α2 - β2 +f_= l

αβ α(α-β) αβ αβ β 1 Ο. Μετά τις πράξεις παίρνουμε:

(α - 2β + γ)2 � 0 . 1 1 . Γεωμετρικά βλέπουμε ότι χ - z = ± 3 . 12. Μετά τις πράξεις παίρνουμε: α4 + α2β2 + γ2 + α2γ2 + β2 = α2 (α2 + β2 + γ2 ) + γ2 + β2 = Ι

α2 α 13. χ = Ι + α2 + + -- =

(α + 1)2 α + Ι ( 1 + α2 ) (α + Ι) 2 α ,_;_ _ ___::_��- + -- =

(α + 1)2 α + Ι

(α + 12 ) + α2 [α2 + 2 (α + 1)] α I -'------=-----=----::---___:__..:.-=. + -- = (α + Ι) 2 α + Ι

α 2 + α + Ι + _J!__ = α + 1 . Άρα χ = 2002 . . α + 1 α + Ι 14. χ-2 > χ -1 > 1 > � > χ > χ2 .

Κατάλληλο βοήθημα για όσους διδάσκουν καθώς και για τις εςετάσεις του ΑΣΕΠ.

Θα βρείτε:

• Σχέδια διδασκαλίας

για όλες τις τάξεις

με την κατάλληλη Μέθοδο, Μορφή και Πορεία διδασκαλfας.

• Την αναλυτική βαθμολόγηση θεμάτων στις προαγωγικές και απολυτήριες εςετάσεις.

• Την αξιολόγηση του μαθητή

και της διδασκαλίας.

ΕΚΔΟΣΕΙΣ ΓΕΝΝΑΔΕΙΟΥ ΣΧΟΛΗΣ Λ. ΕΙΡΗΝΗΣ: 7, 163 45 ΗΛΙΟΥΠΟΛΗ

ΊΉΛ.: 010-9937645 - 6 FAX: 010-9937211 B-mail: [email protected]

Επιοκεφθε(τε μας: http://Www.genadios.edu.gr

ΕΥΚΛΕΙΔΗΣ Β' λστ' τ.l/42

Page 45: Ευκλειδης Β 45

Μαθηματικά Β ' τά του Λυκείου

του Σταμάτη Ε. Καλίκα Σκοπός του άρθρου αυτού είναι να βοηθήσει κάθε μαθητή της Β ' λυκείου στη μελέτη των διανυσμάτων.

Θα ασχοληθούμε με θέματα που έχουν σχέση κυρίως με τις συντεταγμένες των διανυσμάτων. Θεωρούμε όμως αναγκαίο με τις εφαρμογές που ακολουθούν να θυμίσουμε στους μαθητές ότι κάθε διάνυσμα γράφεται ως γραμμικός συνδυασμός δύο μη συγγραμικών διανυσμάτων. Για τα θέματα που έχουν σχέση με τις συντεταγμέ­νες δεν θα περιοριστούμε στην παράγραφο 1.4 (συντεταγμένες στο επίπεδο) του σχολικού βιβλίου αλλά θα α­σχοληθούμε και με θέματα της παραγράφου 1.5 (εσωτερικό γινόμενο) του σχολικού βιβλίου που έχουν σχέση με τις συντεταγμένες των διανυσμάτων. Με τις ασκήσεις που ακολουθούν θα κάνουμε επανάληψη στην αντί­στοιχη θεωρία και έννοιες. Για τις ασκήσεις που ακολουθούν προτείνουμε στους μαθητές να προσπαθήσουν πρώτα να τις λύσουν μόνοι τους και στη συνέχεια να διαβάσουν την προτεινόμενη λύση.

Ερωτήσεις - Ασκήσεις 1. Αν το σημείο Μ είναι εσωτερικό ση­

μείο του τμήματος ΑΒ τέτοιο ώστε (ΑΜ)=3(ΜΒ) και Ο ένα σημείο αναφοράς, να

-+ γράψετε το διάνυσμα ΟΜ ως γραμμικό συν-

-+ -+ δυασμό των διανυσμάτων ΟΑ και ΟΒ .

.... -+ Απάντηση: Τα διανύσματα ΑΜ και ΜΒ εί-

� � � � ναι ομόρροπα και Ι ΑΜ 1=3 Ι ΜΒ Ι άρα ΑΜ =3 ΜΒ

--+ -t --+ --+ Απ' το σχ. 1 έχουμε: ΟΜ -ΟΑ =3( ΟΒ -ΟΜ )

-+ 1 .... . -+ Άρα ΟΜ =-(ΟΑ +30Β ). 4

Α Μ Σχ. 1

Β

2. Αν το σημείο Μ είναι εξωτερικό ση­μείο του τμήματος ΑΒ τέτοιο ώστε (ΑΜ)=3(ΜΒ) και Ο ένα σημείο αναφοράς, να

-+ γράψετε το διάνυσμα ΟΜ ως γραμμικό συν-

-+ -+ δυασμό των διανυσμάτων ΟΑ και ΟΒ .

-+ .... Απάντηση : Τα διανύσματα ΑΜ και ΜΒ

.... .... είναι αντίρροπα και Ι ΑΜ 1=3 1 ΜΒ Ι άρα .... ....

ΑΜ = -3 ΜΒ Απ' το σχ. 2 έχουμε: --+ --+ --+ --+ ..... 1 -+ -+ ΟΜ -ΟΑ =-3 (0Β -ΟΜ )ή ΟΜ =- (-ΟΑ+30Β ).

2

ο

Α Μ Β Σχ.2

3. Έστω το τρίγωνο ΑΒΓ και ΑΜ η διά­μεσός του που αντιστοιχεί στην πλευρά ΒΓ. Ο­νομάζουμε Δ το· μέσο του τμήματος ΑΜ και Ε

, , , -+ 1 -+ το σημειο για το οποιο ισχυει ΑΕ = -ΑΓ . Να

3 -+ -+

εκφράσετε τα διανύσματα ΒΔ και ΔΕ ως -+

γραμμικό συνδυασμό των διανυσμάτων ΑΒ και -+

ΑΓ . Στη συνέχεια να αποδείξετε ότι τα σημεία Β,Δ και Ε είναι συνευθειακά.

Απάντηση: Απ' το σχ. 3 έχουμε: -+ .... .... 1 -+ .... -+ 3 -+ 1 -+ m= Μ-ΑΒ= - (ΑΒ +ΑΓ )-ΑΒ =-- ΑΒ +- ΑΓ .

4 4 4 -+ -+ -+ 1 .... 1 -+ .... ΔΕ =ΑΕ-ΑΔ =3" ΑΓ -4 (ΑΒ +ΑΓ )=

1 .... 1 -+ .... .... -- ΑΒ +- ΑΓ . Παρατηρούμε ότι ΒΔ=3 ΔΕ ο-4 12

πότε τα σημεία Β,Δ και Ε είναι συνευθειακά. Α

Β Μ Γ Σχ.3

4. Αν το σημείο Μ έχει συντεταyμένες

ΕΥΚΛΕΙΔΗΣ Β' λστ' τ.l/43

Page 46: Ευκλειδης Β 45

Μαθηματικά για την Β' Λυκείου (x,y) ως προς το Καρτεσιανό Σύστημα Oxy δη­λαδή M(x,y) ποιες είναι οι συντεταγμένες του συμμετρικού του σημείου ως προς τον άξονα χΊ, ως προς τον άξονα y'y, ως προς την αρχή των αξόνων Ο και ως προς την διχοτόμο της πρώτης και τρίτης γωνίας των αξόνων;

Απάντηση: Με τη βοήθεια του σχ. 4 βρί­σκουμε ότι τα σημεία αντίστοιχα είναι M1 (x,-y) , M2(-x, y) , M3(-x,-y) και M4(y, x) .

M2(-x,y) M(x,y) .. _ _ _ - - - - �,,

Μι(χ,-y) Σχ.4

5. Αν το σημείο Μ έχει συντεταγμένες (x,y) ως προς το Καρτεσιανό Σύστημα Oxy δη­λαδή M(x,y) ποια είναι η απόσταση του σημείου Μ από τους άξονες χΊ και y'y;

Απάντηση: d(M,x'x) =I Υ I και d(M, y'y) =Ι χ ι . 6. Αν το σημείο Μ έχει συντεταγμένες

(x,y) ως προς το καρτεσιανό σύστημα Oxy δη­λαδή M(x,y) ποιες είναι οι συντεταγμένες του

-+ διανύσματος ΟΜ ; Πως γράφεται το διάνυσμα

-+ ΟΜ ως γραμμικός συνδυασμός των μοναδιαί-

-+ -+ ων διανυσμάτων ί και j ;

-+

Απάντηση: Μ(χ, y) <=> ΟΜ = (χ, y) <=> -+ -+ -+

OM = x(l,O) + y(O, I) = x i + y j . 7. Ποιο συμπέρασμα προκύπτει από την

-+ -+ -+ -+ ισότητα: μ ί +ν j =κ ί +λ j όπου κ,λ,μ,ν πραγ-ματικοί αριθμοί;

Απάντηση: Επειδή κάθε διάνυσμα γράφεται κατά μοναδικό τρόπο ως γραμμικός συνδυασμός

-+ -+ των μοναδιαίων διανυσμάτων ί και j , συμπεραί-νουμε ότι μ=κ και ν=λ.

8. Ποιες είναι οι συντεταγμένες των μο--+ -+

ναδιαίων διανυσμάτων ί και j ; -+ -+ -+

Απάντηση: Επειδή ί = Ι ί + Ο j είναι -+ -+ -+ -+ -+ ί = (1, 0) . Επειδή j = 0 i + I j είναι j = (Ο, Ι) .

-+ 9. Πότε το διάνυσμα α = (x,y) είναι πα-ράλληλο στον άξονα χ Ί;

-+

-+ -+ -+ -+ -+

Απάντηση: α = (χ, y) // ί <=> α =χ ί +Ο j <::::>

α = (χ,Ο) -+

10. Πότε το διάνυσμα α = (x,y) είναι πα-ράλληλο στον άξονα y'y;

-+

-+ -+ -+ -+ -+

Απάντηση: α = (χ, y) // j <=> α =Ο ί +y j <=>

α = (O, y) 11. Ας πάρουμε τα σημεία A(x1 ,y1 ) και

M(x0,y0) του Καρτεσιανού Επιπέδου Oxy και Β το συμμετρικό του Α ως προς το σημείο Μ. Να γράψετε τις συντεταγμένες του σημείου Β συναρ­τήσει των συντεταγμένων των σημείων Α και Μ.

Απάντηση: Έστω Β( Χ υ y 2 ) , τότε χ2 = 2χ0 - χ1 και y2 = 2y0 - y1 • 12. Ας πάρουμε τα σημεία A(x1 ,y1 ) και Β( χ2 , y 2 ) του Καρτεσιανού Επιπέδου Oxy και

-+ -+ Μ σημείο τέτοιο ώστε ΑΜ = λΜΒ , λ:;t -1 . Να γράψετε τις συντεταγμένες του σημείου Μ ως συναρτήσεις των συντεταγμένων των σημείων Α, Β και του πραγματικού αριθμού λ.

Απάντηση: Το σημείο Μ έχει συντεταγμένες (Χι + λχ2 , Υι + λy2 ) . Ι + λ Ι + λ

-+ -+ .

Πράγματι ΑΜ = λΜΒ<::::> (χΜ - Χ1 , ΥΜ - yι ) =

-+ 13. Το διάνυσμα α = (3,4) έχει μέτρο 5.

Μπορείτε να βρείτε ένα διάνυσμα ομόρροπο και -+

ένα αντίρροπο στο α που να έχει μέτρο 1 ; -+ 1 -+

Απάντηση: Το διάνυσμα αο = --:;- · α είναι Ι α l

-+ ομόρροπο του α και έχει μέτρο 1 . Άρα το διάνυ-

-+ Ι (3 4) -+ σ μα α ο = - · (3, 4) = - ,- είναι ομόρροπο του α 5 5 5

-+ Ι __, με μέτρο Ι . Το διάνυσμα ασ = ---:;- · α είναι α-! α !

-+ ντίρροπο του α και έχει μέτρο Ι . Άρα το διάνυσμα

�ο = -� · (3,4)=( -�,-�) είναι αντίρροπο του � με μέτρο Ι .

ΕΥΚΛΕΙΔΗΣ Β ' λστ' τ.l/44

Page 47: Ευκλειδης Β 45

Μαθηματικά yια την Β' Λυκείου --+ --+

14. Είναι αληθής η ισότητα det( α , β )= --+ --+

det( β , α ); Δικαιολογήστε την απάντησή σας. Απάντηση: Η παραπάνω ισότητα δεν είναι

--+ --+ αληθής. Αν α = (χ1 , y1 ) και β = (χ2 , y2 ) τότε: .... .... I χ det( α , β )= 1

χ2 Υι I lx2 Υ2 1 --+ --+ =- =_-det ( β , α ).

Υ2 χ, Υι 15. Ποιο συμπέρασμα προκύπτει από τις

--+ --+ παρακάτω σχέσεις: det( α , β )=Ο (1) και

--+ --+ det( α , β � (2).

Απάντηση: Από την (1) προκύπτει ότι τα δια---+ --+

νύσματα α και β είναι συyyραμικά και από την (2) --+ --+

ότι τα διανύσματα α και β είναι μη συyyραμικά. --+ --+

16. Α ν τα διανύσματα α και β είναι μη συπραμικά, τι παριστάνει η ορίζουσα

--+ --+ det( α , β );

Απ ' --+ --+ αντηση: Έστω α = (x, , y, ) και β = (χ2 0 Υ2) .

Αν είναι Ο η αρχή των αξόνων παίρνουμε τα δια---1- � � ---+

νύσματα ΟΑ = α και ΟΒ = β . Επειδή τα διανύ-σματα είναι μη συγγραμικά, τα σημεία Ο,Α και Β σχηματίζουν το τρίγωνο ΟΑΒ. Τότε

-+ -+ ldet( α , β )1=2(0ΑΒ) όπου (ΟΑΒ) το εμβαδόν του τριγώνου ΟΑΒ. Σχ.5.

Α

Σχ. 5 -+

17. Δίνονται τα διανύσματα α =(1,2), -+ -+ β =(2,1) και γ =(8,7). Να γράψετε το διάνυσμα -+ -+ γ ως γραμμικό συνδυασμό των διανυσμάτων α

-+ και β .

Απάντηση: Αναζητούμε πραγματικούς αριθ---+ --+ --+

μούς κ.λ ώστε γ =κ α +λ β <=> (8,7)=κ(1 ,2)+λ(2, 1 )

<=> { ;: :�: �} <=> κ=2, λ=3. Δηλαδή --+ --+ -+ γ =2 α +3 β

-+ -+ 18. Δίνονται τα διανύσματα α =(1,2), β =(2,4)

-+ -+ και γ =(8,7). Να γράψετε το διάνυσμα γ ως γραμ-

-+ -+ μικό συνδυασμό των διανυσμάτων α και β .

Απάντηση: Αναζητούμε πραγματικούς αριθ---+ --+ --+

μούς κ.λ ώστε γ =κ α +λ β <=> (8,7)=κ(1,2)+λ(2,4) { κ + 2λ = 8 } <=> . Το τελευταίο σύστημα είναι α-2κ + 4λ = 7

--+ δύνατο. Έτσι το διάνυσμα γ δεν γράφεται ως

--+ --+ γραμμικός συνδυασμός των διανυσμάτων α και β .

' --+ -+ Σχολιο: Τα διανύσματα είναι α , β συγγραμμικές.

-+ 19. Δίνονται τα διανύσματα α =(1,2),

-+ -+ β =(2,4) και γ =(3,6). Να γράψετε το διάνυσμα -+ -+ γ ως γραμμικό συνδυασμό των διανυσμάτων α

-+ και β .

Απάντηση: Αναζητούμε πραγματικούς αριθ--+ --+ -+

μούς κ.λ ώστε γ =κ α +λ β <=> (3,6)=κ(1 ,2)+λ(2,4)

<=> { κ + 2λ = 3 } <=> κ+2λ=3 <=> (κ,λ)=(3-2λ,λ), 2κ + 4λ = 6 ---+ ---+ ---+ -+ -+ -+ -+ -+ -+

λεΙR. Έτσι γ = α + β ή γ =3 α +Ο β ή γ =5 α - β κ.λ.π.

'λ --+ -+ -+

Σχο ιο: τα διανύσματα α , β , γ είναι συγ-γραμματικά.

Γ '

-+ -+ 20. ενικευση: Α ν τα διανύσματα α και β

-+ είναι μη παράλληλα τότε κάθε διάνυσμα γ του επιπέδου τους γράφεται κατά μοναδικό τρόπο

-+ -+ ως γραμμικός συνδυασμός των α και β . Αν τα

-+ -+ διανύσματα α και β είναι παράλληλα τότε κά-

-+ θε διάνυσμα γ του επιπέδου τους γράφεται με άπειρους τρόπους ως γραμμικός συνδυασμός

-+ -+ των α και β ή δεν μπορεί να γραφεί.

Απάντηση: Έστω -+ --+ -+

και

β = (x2 0 y2) . Αν τα διανύσματα α , β είναι μη

συγγραμμικά θα είναι det(� , β ):;t:O ή lx, Yι l:;t:O· Χ2 Υ2 --+

Έστω γ =(x,y), τότε αναζητούμε πραγματικούς α-

ΕΥΚΛΕΙΔΗΣ Β' λστ' τ.l/45

Page 48: Ευκλειδης Β 45

Μαθηματικά για την Β' Λυκείου -+ -+ -+ ριθμούς κ,λ τέτοιους ώστε γ =κ α +λ β . Από την

τελευταία ισότητα οδηγούμαστε στο σύστημα {ΚΧι + λχ2 = Χ} κyl + λy2 = y . Θεωρούμε ως αγνώστους τα κ

και λ. Η ορίζουσα του συστήματος είναι Ι Χι χ2 ι :;t:O. Έτσι το σύστημα έχει μοναδική λύση Υι Υ2

-+ -+ ως προς κ και λ. Α ν τα διανύσματα α , β είναι συγγραμμικά θα είναι det( � ' β )=Ο ή I Χι Υ

ι ι =0. Χ2 Υ2

Η ορίζουσα του συστήματος είναι ΙΧ ι χ2 ι =0. Έ-Υι Υ2

τσι το σύστημα έχει άπειρες λύσεις ή είναι αδύνα­το ως προς κ και λ. (Αν θυμηθούμε την αντίστοιχη θεωρία από την Α' Λυκείου).

21. Έστω το τρίγωνο ΑΒΓ και ΑΜ η διά­μεσός του που αντιστοιχεί στην πλευρά ΒΓ. Ο­νομάζουμε Δ το σημείο που ορίζεται από την ι-

-+ -+ σότητα ΑΔ = κ ΑΜ , κ:;t:2, και Ε το σημείο τομής των ευθειών ΒΔ και ΑΓ. Επειδή τα διανύσματα

-+ -+ ΑΕ και ΑΓ είναι παράλληλα υπάρχει πραγμα-

-+ -+ τικός αριθμός χ για τον οποίο ισχύει ΑΕ =χ ΑΓ •

Να εκφραστεί ο πραγματικός αριθμός χ ως συ­νάρτηση του κ.

Α

Γ Σχ. 6

Απάντηση: Απ' το σχήμα έχουμε: -+ -+ -+ κ -+ -+ ΒΔ= ΑΔ-ΑΒ = - ( ΑΒ + ΑΓ )-2 -+ κ-2 -+ κ -+ ΑΒ=- ΑΒ +- ΑΓ (1). 2 2

-+ -+ -+ ΔΕ =ΑΕ-ΑΔ -+ κ -+ κ -+ Οπότε: ΔΕ =(χ-- ) ΑΓ - - ΑΒ (2).

-+ -+ 2 2 Όμως ΒΔ // ΔΕ , άρα υπάρχει πραγματικός α--+ -+ ριθμός y τέτοιος ώστε ΒΔ=y ΔΕ (3). Από τις (1)

(2) και (3) προκύπτει το σύστημα: { κ;2 = � κ yκ - = yx--2 2

, , , κ2 -κ απο το οποιο εχουμε χ = --. κ- 2 Ερώτηση: τι συμβαίνει στην περίπτωση που

είναι κ=2; 22. Α ν ονομάσουμε φ τη γωνία που σχημα-

-+ τίζει το διάνυσμα α = (x,y) :fi!:(O,O) με τον άξονα Ί:ι.: και ρ το μέτρο του, μπορείτε να γράψετε τις συντεταγμένες του σημείου Α για το οποίο

-+ -+ ΟΑ = α , ως συνάρτηση των ρ και φ;

Απάντηση: Από τον ορισμό του ημιτόνου και συνιμητόνου που μάθαμε στην Γ γυμνασίου είναι χ=ρσυνφ και y=ρημφ. (τους αριθμούς ε, φ τους λέμε πολικές συντεταγ­μένες του Α)

A(x,y)

φ ο Σχ. 7

23. Σε ποια περίπτωση δεν ορίζεται συντε--+

λεστής διεύθυνσης για το διάνυσμα α = (x,y) ; Απάντηση: Α ν χ=Ο τότε δεν ορίζεται συντε­

λεστής διεύθυνσης και το διάνυσμα είναι παράλ­ληλο στον άξονα yy' . Η γωνία που σχηματίζει το δ ' -+ ' ξ ' ' π ' 3π ιανυσμα α με τον α ονα χ χ ειναι - η - . 2 2

24. Να βρείτε τα κ, λ ώστε τα διανύσματα -+ -+ α =(JC+l, 1) και β =(2κ-λ2,1) να είναι παράλληλα.

� -+ --+ --+ Απάντηση: α I/ β <:::::> det( α , β )=Ο <:::::>

κ2 + 1 1 2 2 2 2 =Ο <:::::> κ -2κ+ Ι+ λ =Ο <:::::> (κ- 1) +λ =Ο <:::::> 2κ-� 1 κ=1 και λ=Ο.

-+ 25. Πότε το διάνυσμα α = (x,y) είναι πα-

ράλληλο στην διχοτόμο της πρώτης και τρίτης γωνίας των αξόνων;

Απάντηση: Επειδή η διχοτόμος της πρώτης και τρίτης γωνίας των αξόνων σχηματίζει με τον θετικό -+ ημιάξονα Οχ γωνία 45°, το διάνυσμα α = (χ, y)

-+ πρέπει να είναι της μορφής α = (κ, κ) , κ:;t:Ο. -+

26. Τα διανύσματα α = (l, 1) και -+ β = (-1, -1) έχουν συντελεστή διεύθυνσης λ=1. Μπορούμε να συμπεράνουμε ότι σχηματίζουν

ΕΥΚΛΕΙΔΗΣ Β' λστ' τ.l/46

Page 49: Ευκλειδης Β 45

Μαθηματικά yια την Β· Λυκείου

την ίδια γωνία με τον άξονα χ'χ; Δικαιολογήστε την απάντησή σας.

Απάντηση: --> -->

Έστω --> --> ΟΑ = α= (l, 1) και ΟΒ = β =(-1, -1) . Τότε Α(1 , 1 ) και Β (-1, -1) . Το σημείο Α βρίσκεται στο 1 ο τεταρτημόριο ενώ το Β

--> στο 3° τεταρτημόριο. Το διάνυσμα α=(1, 1) σχη-ματίζει με τον άξονα n: γωνία 45° ενώ το διάνυ-

-+ σμα β = ( -1, -1) σχηματίζει με τον άξονα r:ι.: γω-νία 225°. (Σχ.8).

27. Ποια γωνίά σχηματίζουν τα διανύσμα--+ --> --> r;; τα α = (1, J3) , β = ( -1, J3) , γ = ( -1,- ν 3) ,

δ = (1,- J3) με τον άξονα χ'χ;

Α

Β Γ - - - - - - - - - - - - Δ

Σχ. 8 Σχ. 9 Απάντηση: Όπως φαίνεται και από τα σχή-

ματα η γωνία του � = (1, .J3) με τον άξονα χ:χ' εί-ναι 60°, του β = ( -1, .J3) με τον r:{ είναι 120°, του ; = ( -1,- .J3) με τον χι.: είναι 240° και του δ = (1, - .J3) με τον χχ· είναι 300°. (Σχ.9).

28. Ποια γωνία σχηματίζουν τα διανύσμα---> -->

τα α = (2, 5) και β = (6, 15) ; --+ -->

Απάντηση: Επειδή β =3 α , τα διανύσματα είναι ομόρροπα οπότε η γωνία των διανυσμάτων --+ --> α και β έχει μέτρο 0°.

29. Ποια γωνία σχηματίζουν τα διανύσμα---> -->

τα α = (2, -3) και β = (-6, 9) ; --> --+

Απάντηση: Επειδή β = -3 α , τα διανύσματα είναι αντίρροπα οπότε η γωνία των διανυσμάτων -+ --> α και β είναι 180°.

30. Ποια γωνία σχηματίζουν τα διανύσμα-

τα � = (1, 1) και Ρ = (1, J3) ; Απάντηση: Ο συντελεστής διεύθυνσης του

--> διανύσματος α είναι 1 και αν η αρχή του τοποθε-τηθεί στο 0(0,0) το πέρας του θα είναι το σημείο Α(1 , 1). Έτσι η γωνία που σχηματίζει με τον άξονα χ" χ είναι 45°. Ο συντελεστής διεύθυνσης του δια-νύσματος β είναι .J3 και αν η αρχή του τοποθε­τηθεί στο 0(0,0) το πέρας του θα είναι το σημείο Β(1, .J3 ). Έτσι η γωνία που σχηματίζει με τον ά­ξονα χ· χ είναι 60°. Η γωνία λοιπόν των διανυσμά-

-+ --> των α κάι β είναι 60°-45°=15°. (Σχ. lΟ). Β

Α

Σχ. lΟ 31. Ποια γωνία σχηματίζουν τα διανύσμα-

τα � = (1,- 1) και Ρ= (1,J3) ; Απάντηση: Αν σκεφτούμε με τον τρόπο της

άσκησης 3 1 θα βρούμε ότι η γωνία των διανυσμά--+ --> των α και β είναι 225° -60°= 165°. (Σχ. 1 1 ). 32. Ποια γωνία σχηματίζουν τα διανύσμα-

τα ; = (-2, 2) και Ρ= (- J3, -1) ; Β

Α

Α Β Σχ. 1 1 Σχ. 12

Απάντηση: Αν σκεφτούμε με τον τρόπο της άσκησης 3 1 θα βρούμε ότι η γωνία των διανυσμά-

-+ --> των α και β είναι 210°-135°=75°. (Σχ. 12). 33. Θυμηθείτε πότε ισχύει η ισότητα:

--> --> --> -->

11 α Ι - Ι β 1 1=1 α+ β Ι και στη συνέχεια να βρείτε σημείο Μ του άξονα r:ι..' τέτοιο ώστε η παρά-

σταση: iι ΜΑ ι - ι ΜΒ ιi να γίνειμέγιστη, όπου

Α(1,2) και Β(2,1). (Σχ.13). Απάντηση,: Η ισότητα ισχί>ει μόνο στην περί-

-+ --> πτωση που τα διανύσματα α και β είναι αντίρρο-πα. Το ζητούμενο σημείο Μ θα έχει συντεταγμένες της μορφής Μ(χ,Ο). Ακόμα

i ιΜΛ ι - ιΜΉ ιl = l ιΛΜ ι - ι ΜΒ ΙΙ :::; ιΛΜ+ΜΒ Ι =

ΕΥΚΛΕΙΔΗΣ Β" λστ" τ.l/47

Page 50: Ευκλειδης Β 45

Μαθηματικά yια την Β' Λυκείου

I ΑΒ I = J2 . Η μεγαλύτερη τιμή της παράσrασης είναι

J2 και συμβαίνει όταν τα διανύσματα ΑΜ κάι ΜΉ είναι αντίρροπα. Το ζητούμενο σημείο Μ είναι το 0,­μείο τομής της ευθείας ΑΒ και του ά!;ρνα χ:χ:.

Επειδή τα σημεία Α,Β είναι σημεία του πρώ­του τεταρτημορίου το ζητούμενο σημείο είναι ε­κτός του τμήματος ΑΒ και έτσι τα διανύσματα

-> -> ΑΜ και ΜΒ είναι αντίρροπα. Το σημείο Μ θα

-> -> βρεθεί από την ισότητα det( ΑΜ , ΜΒ )=Ο ή lx - 1 -2 ι 2 - χ 1

=Ο ή χ=3. Άρα· το ζητούμενο σημείο

είναι το Μ(3,0) 34. Να θυμηθούμε πότε ισχύει η ισότητα:

-> -> -> -+ .

I α I + I μ 1=1 α+ μ ι . Στη συνέχεια να βρείτε ση-μείο Μ του άξονα Ί:J.: τέτοιο ώστε η παράσταση:

-+ -+

I ΜΑ I + Ι Μ8 Ι να γίνει ελάχιστη, όπου Α(1,2) και 8(5,-2). Σχ.14.

ο

A(l ,2) �::]�� �(2, 1) Γ-τ�:: Μ

Α

ο Σ� 1 3 Σχ. 14

Μ " Β

Απάντηση: Η ισότητα ισχύει μόνο στην περί--> ->

πτωση που τα διανύσματα α και β είναι ομόρρο-πα. Το ζητούμενο σημείο Μ θα έχει συντεταγμένες της μορφής Μ(χ,Ο).

� � � � Ακόμα I MA I + I MB Ι = I AM I + I MB I �

I ΑΜ+ ΜΒ I = I ΑΒ I =4 J2 . Η μικρότερη δυνατή τιμή της παράστασης είναι 4 J2 και συμβαίνει ό-

-> -> ταν τα διανύσματα ΑΜ και ΜΒ είναι ομόρροπα. Το ζητούμενο σημείο Μ είναι το σημείο τομής της ευθείας ΑΒ και του άξονα n:. Επειδή τα σημεία Α,Β είναι εκατέρωθεν του άξονα 'ΧΥ.: το ζητούμενο σημείο είναι εντός του τμήματος ΑΒ και έτσι τα

-> -> διανύσματα ΑΜ και ΜΒ είναι ομόρροπα. Το σημείο Μ θα βρεθεί από την ισότητα

.... .... ιχ - 1 -2ι det( ΑΜ , ΜΒ )=Ο ή =Ο ή χ=3. Άρα το 5 - χ -2 ζητούμενο σημείο είναι το Μ(3,0).

-+ -+ -+ -+ 35. Έστω ΟΑ = (1,2), 08 = 4 i +3 j και

Γ(-1,8) . Να εξηγήσετε γιατί το τρίγωνο ΑΒΓ είναι ορθογώνιο.

Απάντηση: Είναι Α(1 ,2), Β(4,3) και Γ(-1,8) . -> ->

Άρα ΑΒ = (3, 1) και ΑΓ = ( -2, 6) . Επειδή -> ->

ΑΒ · ΑΓ =Ο το τρίγωνο είναι ορθογώνιο στο Α. 36. Δίνονται τα σημεία Α(1,2) και 8(2,1).

Να βρεθεί η προβολή του σημείου 0(0,0) στην ευθεία ΑΒ. Στη συνέχεια να βρεθεί το συμμετρι­κό του Ο ως προς την ευθεία ΑΒ. (Σχ.15).

Απάντηση: Ονομάζουμε Λ(α,β) την προβολή του Ο στην ευθεία ΑΒ και Κ(γ,δ) το συμμετρικό του Ο ως προς την ευθεία ΑΒ. Το Λ προσδιορίζε­ται από τις σχέσεις: { �/ � }<=:>{Ια�

1 β�121 = Ο}<=:> . . . <=:> α=β=3/2.

ΟΑ .l ΑΒ (α,β) · (l,-1) = 0

Κ(3,3)

Σχ. 1 5 Επειδή το Λ είναι το μέσο του τμήματος ΟΚ

ισχύει: {γ; 0 = %} <:::::> γ=δ=3. Έτσι Λ(3/2,3/2) και δ + Ο 3 -- = -2 2

Κ(3,3). 37. Δίνονται τα μέσα Μ1 (α1 ,β1 ) των

πλευρών ενός ν-γώνου Α1 Α2 •••• Α. με ν περιττό. Μπορείτε να βρείτε τις συντεταγμένες των κο­ρυφών του ν-γώνου;

Απάντηση: Έστω Μ1(α1 ,β 1) το μέσο της πλευράς ΑιΑ2, Μ2(α2,β2) το μέσο της πλευράς Α2Α3, . . . . Mv(α.,/3v) το μέσο της πλευράς ΑvΑι. Τό­τε για τις τετμημένες των κορυφών έχουμε το σύ-{ χ1 + χ2 = 2α1 , χ2 + χ3 = 2α2 , } στημα: : .

χ._1 + χ. = 2α._1 , χ. + χ1 = 2α. Πολλαπλασιάζοντας τις εξισώσεις άρτιας τά­

ξης με ( - 1) έχουμε: { χ1 + χ2 = 2α1 , - χ2 - χ3 = -2α2 } -χ._1 - χ. = -2α�_1 , χ. + χ1 = 2α.

.

Με πρόσθεση κατά μέλη βρίσκουμε 2χι =2(α, -� + ... -<χ.,..ι +α..,) οπότε Χι =α,---«z+ ... � +α.., . Στη συνέχεια βρίσκουμε διαδοχικά τα Χ2, χ3, . . . χ •.

Ομοίως βρίσκουμε τις τεταγμένες των κορυφών του ν-γώνου.

Παρατήρηση: Αν το πλήθος των πλευρών εί-ΕΥΚΛΕΙΔΗΣ Β' λστ' τ.l/48

Page 51: Ευκλειδης Β 45

Μαθηματικά pα την Β' Λυκείου ναι άρτιο, τότε έχουμε άπειρες λύσεις ή το σύστη- ι πλαίσια αυτού του άρθρου. μα είναι αδύνατο. Η εξήγηση ξεφεύγει από τα

a Μετρικές σχέσεις

Οι ασκήσεις Μετρικών Σχέσεων αφορούν κυ­ρίως λογισμό μεταξύ τετραγώνων μηκών ευθυ­γράμμων τμημάτων. Το πρωτογενές εδώ θεώρημα είναι το Πυθαγόρειο, καθ' όσον τα λοιπά σχετικά θεωρήματα (Ομάδα Θεωρημάτων για το ορθογώ­νιο τρίγωνο: Θεωρήματα Προβολών, Θεώρημα Ύψους κλπ.), το Γενικευμένο Πυθαγόρειο Θεώρη­μα (ή Θεώρημ,α Επέκτασης) και τα Θεωρήματα των Διαμέσων απορρέουν από αυτό. Εξάλλου, ο νόμος των συνημιτόνων στα τρίγωνα αποτελεί την ενιαία έκφραση του Πυθαγορείου Θεωρήματος (βλ. άσκηση 1 0).

Α Σ Κ Η Σ Ε Ι Σ 1. Από σημείο Ο στο εσωτερικό τριγώνου

ΑΒΓ φέρουμε τις καθέτους ΟΔ, ΟΕ και ΟΖ στις πλευρές ΒΓ, ΑΓ, ΑΒ αντίστοιχα. Ν α δείξετε ότι:

ΑΕ2 +ΒΖ2 + ΓΔ2 = ΑΖ2 + ΒΔ2 +ΓΕ2 Απόδειξη

Με διαδοχική εφαρμογή του Πυθαγορείου Θεωρή­ματος στα τρίγωνα ΑΟΕ, ΒΟΖ και ΓΟΔ, έχουμε:

Α

e----+--� Γ ΑΕ2 = ΑΟ2 - ΟΕ2 , ΒΖ2 = ΒΟ2 - ΟΖ2

ΓΔ2 = ΓΟ2 - ΟΔ2 Προσθέτουμε τώρα κατά μέλη τις παραπάνω σχέσεις:

ΑΕ2 + ΒΖ2 + ΓΔ2 = ΑΟ2 - ΟΖ2 + ΒΟ2 -ΟΔ2 + ΓΟ2 -ΟΕ2 = '-ν-----' '-------ν---' '------ν-"'

ΑΖ2 + ΒΔ2 + ΓΕ2 2. Δύο κάθετες χορδές ΑΓ και ΒΔ κύκλου

(Ο, R) τέμνονται στο Σ. Να δείξετε ότι: ΣΑ2 + ΣΒ2 + ΣΓ2 + ΣΔ2 = 4R2 Απόδειξη Στα ορθογώνια τρίγωνα ΑΣΒ και ΓΣΔ εφαρ-

μόζουμε Πυθαγόρειο Θεώρημα: ΣΑ2 +ΣΒ2 =ΑΒ2 (1) ΣΓ2 +ΣΔ2 = ΓΔ2 (2) Προσθέτουμε κατά μέλη τις (1), (2): ΣΑ2 +ΣΒ2 +ΣΓ2 +ΣΔ2 =ΑΒ2 + ΓΔ2 (3) Φέρουμε τη διάμετρο ΑΚ, οπότε

κfΆ = 90° ' άρα ΚΓ /I ΔΒ (διότι ΑΓ j_ ΒΔ ).

Του Θανάση Κυριακόπουλου Α

Άρα θα είναι και ΓΔ = ΚΒ (γιατί;) Έτσι, το δεύτερο μέλος της (3):

ΑΒ2 + ΓΔ2 =ΑΒ2 +ΚΒ2 =ΑΚ2 = (2R)2 = 4R2 (από το ορθογώνιο τρίγωνο ΑΒΚ). Υπενθύμιση: Τόξα περιεχόμενα μεταξύ πα­

ράλληλων χορδών είναι ίσα. 3. Εάν ΒΕ διάμεσος ορθογωνίου τριΎώνου

ΑΒΓ (Α = 90°) , να δείξετε ότι:

α) ΒΕ = Μ '

ΒΜ = 60° , όπου Δ το μέσο της ΒΓ, να υπολογι­σθούν οι πλευρές του τριΎώνου ΑΒΓ.

Απόδειξη α) Από το ορθογώνιο τρίΎωνο ΑΒΕ:

ΒΕ2 = ΑΒ2 + ΑΕ2 = ΑΒ2 + _!_ ΑΓ2 οπότε: 4 ΒΕ2 +iΑΓ2 =ΑΒ2 +_!_ΑΓ2 +iΑΓ2 = 4 4 4 = ΑΒ2 +ΑΓ2 = ΒΓ2

β) Είναι Δι = ΒΜ = 60° άρα f = 30° (διότι ΑΔ διάμεσος στην υποτείνουσα άρα τρίΎωνο ΔΑΓ ισοσκελές ή Δι = f + Α1 ή Δ1 = 2f ή f = 30° ) ο­, ΑΒ ΒΓ ποτε = - . 2

Γ

Ακόμη: Από ΑΒ2 + ΑΓ2 = ΒΓ2 είναι ΒΓ2 +ΑΓ2 = ΒΓ2 ή ΑΓ = ΒΓ.J3 . 4 2

Υπολογίζουμε τώρα την ΒΓ. Από το (α):

ΕΥΚΛΕΙΔΗΣ Β' λστ' τ.l/49

Page 52: Ευκλειδης Β 45

Μαθηματικά για την Β' Λυκείου

( .Ji4)2 +% · ΒΓ; · 3 = ΒΓ2 ή ΒΓ = 4'-"2 .

Τελικά είναι: ΑΒ = 4'-"2 = 2'-"2 , ΑΓ= 4!2..[3 =2J6 2 2 4. Δίνεται ορθογώνιο τρίγωνο ΑΒΓ

(Α = 90°) , το ύψος του ΑΔ και Ε, Ζ οι προβολέs του Δ στις πλευρές ΑΒ, ΑΓ αντίστοιχα. Να δεί­ξετε ότι: ΑΔ 3 = ΒΓ · ΔΖ · ΔΕ .

Απόδειξη Έχουμε: ΑΔ 2 = ΑΒ · ΑΕ (ορθογώνιο τρίγωνο

ΑΔΒ) ή επειδή ΑΕ = ΔΖ είναι: ΑΔ2 = ΑΒ · ΔΖ (1) Από το ορθογώνιο τρίγωνο ΑΔΓ είναι:

ΑΔ2 = ΑΓ · ΑΖ ή ΑΔ2 = ΑΓ · ΔΕ διότι ΑΖ = ΔΕ . Α

Β Γ Από τις (1), (2) με πολλαπλασιασμό κατά μέλη:

ΑΔ4 = ΑΒ · ΑΓ · ΔΖ · ΔΕ <=> ΑΔ4 = ΔΖ · ΔΕ (ΑΒ · ΑΓ) ή ΑΔ4 = ΔΖ · ΔΕ (ΒΓ · ΑΔ) ή

(διότι ξέρουμε ότι σε ορθογώνιο τρίγωνο ΑΒΓ Α = 90° , ισχύει: β · γ = α · υα ) ΑΔ3 = ΒΓ · ΔΖ · ΔΕ

5. Δίνεται ορθογώνιο τρίγωνο ΑΒΓ (Α = 90°) και σημείο Ζ της ΒΓ. Φέρουμε ΖΚ και ΖΛ καθέτους στις ΑΒ, ΑΓ αντίστοιχα. Να δείξετε ότι: ΖΒ · ΖΓ = ΑΚ · ΚΒ + ΑΛ · ΛΓ .

Απόδειξη Με Πυθαγόρειο Θεώρημα στο ορθογώνιο τρί-

γωνο ΑΒΓ, έχουμε: ΒΓ2 = ΑΒ2 + ΑΓ2 <::::>

(ΒΖ + ΖΓ)2 = (ΑΚ + ΚΒ)2+(ΑΛ+ΛΓ)2 <=> ΒΖ2 + ΖΓ2 + 2ΒΖ · ΖΓ = ΑΚ.2 + ΚΒ2 + + 2ΑΚ · ΚΒ + ΑΛ2 + ΛΓ2 + 2ΑΛ · ΛΓ (1)

Γ

Α Κ Β Όμως ΒΖ2 = ΚΒ2 + ΚΖ2 (2) (ορθογώνιο τρίγωνο ΖΚΒ) ΖΓ2 = ΛΖ2 + ΛΓ2 (ορθογώνιο τρίγωνο Γ ΛΖ) ή zr2 = ΑΚ2 + ΛΓ2 (3) Οπότε η (1), λόγω των (2), (3) γίνεται:

ΚΒ2 + ΚΖ2 + ΑΚ2 + ΛΓ2 + 2ΒΖ · ΖΓ = ΑΚ2 + ΚΒ2 + 2ΑΚ. ΚΒ+ Μ + ΛΓ2 + 2ΑΛ . ΑΓ <=> ΚΒ2 + ΚΖ2 + ΑΚ2 + ΛΓ2 + 2ΒΖ · ΖΓ = ΑΚ. 2 + ΚΒ2 + ΖΚ2 + ΛΓ2 + 2ΑΛ · ΛΓ + 2ΑΚ. · ΚΒ <=> 2ΒΖ · ΖΓ = 2ΑΚ. · ΚΒ + 2ΑΛ · ΛΓ <=> ΒΖ · ΖΓ = ΑΚ. · ΚΒ + ΑΛ · ΛΓ

6. Δίνεται κύκλος (Ο, R) και δύο κάθετες ακτίνες του ΟΑ, ΟΒ. Εάν η χορδή ΓΔ // ΑΒ τέ­μνει τις ΟΑ, ΟΒ στα Ε, Ζ αντίστοιχα, να δείξετε ότι: ΓΕ2 + ΓΖ2 = 2R2 •

Απόδειξη Φέρουμε τη διάμεσο ΟΜ στην υποτείνουσα

του ορθογωνίου τριγώνου ΟΕΖ. Είναι: ΓΕ2 + ΓΖ2 = (ΓΜ-ΕΜ)2 + (ΓΜ + ΜΖ)2 =

ΓΜ2 + ΕΜ2 - 2ΓΜ · ΕΜ + rM +.Ml2 +2IM· .MZ

Προφανώς ΕΜ = ΜΖ = ΟΜ , οπότε: ΓΕ2 + ΓΖ2 = 2ΓΜ2 + ΕΜ2 + ΜΖ2 -2ΓΜ · ΕΜ + 2ΓΜ · ΜΖ = 2ΓΜ2 + 20Μ2 - 2ΓΜ · ΟΜ + 2ΓΜ · ΟΜ = 2 (ΓΜ2 + ΟΜ2) = (Πυθαγόρειο Θεώρημα

στο ορθογώνιο τρίγωνο ΟΓΜ) = 2ΟΓ2 = 2R 2 7. Έστω κύκλος κέντρου Ο και μία διάμε­

τρος αυτού ΑΒ. Με διάμετρο την ακτίνα ΟΑ γράφουμε δεύτερο κύκλο. Σε τυχαίο σημείο Ζ της ΟΑ φέρουμε κάθετη στην ΑΒ που τέμνει τον εσωτερικό κύκλο στο Δ και τον εξωτερικό στο Ε. Να δείξετε ότι: ΑΕ2 = 2ΑΔ2•

Απόδειξη Από το ορθογώνιο τρίγωνο ΑΕΒ (Ε = 90° ,

διότι είναι εγγεγραμμένη στο ημικύκλιο): ΑΕ2 = ΑΒ · ΑΖ (1)

Όμοια από το ορθογώνιο τρίγωνο ΑΔΟ (Δ = 90°) : ΑΔ2 = ΑΟ · ΑΖ (2) Διαιρούμε κατά μέλη τις (1 ), (2): ΑΕ2 _ ΑΒ _ 2ΑΟ _ 2 ΑΔ2 - ΑΟ - ΑΟ - . 8. Θεωρούμε δύο όμοια ορθογώνια τρίγωνα

ΕΥΚΛΕΙΔΗΣ Β' λστ' τ.l/50

Page 53: Ευκλειδης Β 45

Μαθηματικά για την Β' Λυκείου

ΑΒΓ και Α'ΒΤΌ Να δείξετε ότι: ι ι ι --+--=--

β · β' γ · γ' υα · υ� Απόδειξη

(ι)

Είναι ΑΒΓ � Α 'Β'Γ' β γ υ <=>-=-=�=λ β' γ' υ'α <:::::> β = β 'λ, γ =γ'λ, υα = υ 'α· λ όπου λ ο λόγος ο­μοιότητας των τριγώνων. Οπότε:

ι ι ι , ι ι ι (ι )<=> λβ'2 + λγ'2

= λυ� η β'2 + γ'2 = λυ'�

που ισχύει, διόn το τρίγωνο Α 'Β 'Γ' ορθογώνιο (Α' = 90°) .

9. Δίνεται τετράγωνο ΑΒΓΔ πλευράς α. Τυ­χαία ευθεία από το Α τέμνει τις ΒΓ, Γ Δ στα Ε, Ζ

, Ν δ ίξ ό ι ι ι αντιστοιχα. α ε ετε τι: --2 + --2 = 2 •

ΑΕ ΑΖ α Απόδειξη Φέρουμε ΑΝ j_ ΑΖ . Τότε ΑΝΔ = ΑΒΕ

(ΑΔ = ΑΒ =α και Αι = Α3 ως οξείες γωνίες με πλευρές κάθετες). Άρα και ΑΕ =ΑΝ .

Στο ορθογώνιο τρίγωνο ΑΝΖ, ισχύει: ι ι ι ι ι ι --+--=-- ή --+--=­ΑΝ2 AZz ΑΔz AEz AZz αz

.'r-:::-τ-----iB / ι /

,./' α /

�Ε �,I'/ / Δ Γ z Σημείωση: Υπενθυμίζουμε δύο πολύ βασικές

σχέσεις για ορθογώνιο τρίγωνο ΑΒΓ (Α= 90°) , ό­που υa το ύψος στην υποτείνουσα ΒΓ:

ι ι ι α · υa = βγ και --τ=---z+---z . υα β γ ιο. Πώς από το νόμο των συνημιτόνων

προκύπτει το γενικευμένο Πυθαγόρειο Θεώρη­μα σε ένα τρίγωνο ΑΒΓ;

Απόδειξη Ως γνωστόν, ισχύει: α2 = β2 +γ2 - 2βγσυνΑ (1) Διακρίνουμε περιπτώσεις:

ι) Εάν Α =90° , τότε συνΑ = συν90° ή συνΑ = Ο οπότε από (Ι): α2 = β2 + γ2 (ΠυθαyόρεωΘεώρημα). 2) Εάν Α < 90° είναι:

Α

β

Β "--------..--____,. Γ

συν Α = ΑΔ οπότε από (1 ): γ ΑΔ αz = βz + γz _ 2βγ _ = βz + γz _ 2βΑΔ γ

3) Εάν Α> 90° είναι: Α+ Αι = ι sοο ή Α = ι sοο-Αι ή συν Α = συν(ι sο - Αι) = - συν Αι δηλαδή συν Α = - συν Αι

Β

Γ

Όμως (σχήμα): συνΑι = ΑΔ και η (ι): γ α2 = β2 + γ2 - 2βγσυνΑ = β2 +γ2 - 2βγ (-συνΑι ) = β2 +γ2 + 2βγσυνΑι =

ΑΔ βz +γz + 2βγ _ = βz +γz + 2βΑΔ. γ Βασικές επισημάνσεις

ι. Μια πολύ χρήσιμη για nς ασκήσεις πρόταση (γνωστή από την Α' Λυκείου) είναι: Εάν σε ένα ορθογώνιο τρίγωνο μια οξεία γωνία είναι 30° η α­πέναντι (από τη γωνία των 30°) κάθετη πλευρά εί­ναι ίση με το μισό της υποτείνουσας και αντιστρό­φως. Έτσι, αν στο σχήμα μας παρουσιαστεί άμεσα ή έμμεσα γωνία 30° σε ορθογώνιο τρίγωνο, εφαρ­μόζουμε το πολύτιμο αυτό συμπέρασμα.

Β

Β

Γ

Δηλαδή; αν μας δίνουν γωνία 60°, 120°, ιsοο τότε στο σχήμα μας αναζητούμε γωνία 30°. Προσο­χή στο αντίστροφο: Εάν εντοπίσουμε στο σχήμα μας ορθογώνιο τρίγωνο τέτοω ώστε μια κάθετη πλευρά του να είναι ίση με το μισό της υποτείνου­σας, τότε η γωνία που βρίσκεται απέναντι από την

ΕΥΚΛΕΙΔΗΣ Β' λστ' τ.l/51

Page 54: Ευκλειδης Β 45

Μαθηματικά -yια την Β· Λυκείου

κάθετη αυτή πλευρά, είναι ίση με 30°. 2. Για το είδος του τριγώνου ως προς τις γωνίες

του: Συγκρίνουμε το τετράγωνο της μεγαλύτερης πλευράς με το άθροισμα των τετραγώνων των ό).)..mν δύο και · εφαρμόζουμε το αντίστοιχο θεώρημα. Προ­ϋπόθεση για την ύπαρξη τριγώνου με δεδομένα μήκη είναι η ισχύς της τριγωνικής ανισότητας.

3. Αρκετά χρήσιμοι για τις ασκήσεις είναι οι τύποι των τετραγώνων των διαμέσων:

2 2β2 + 2γ2 -α2 2 2α2 + 2γ2 -β2 μα = 4 ' μβ

4

2 2α2 + 2β2 -γ2 μγ = 4

4. Η διάμεσος στην υποτείνουσα ενός ορθογω­νίου τριγώνου είναι ίση με το μισό της υποτείνου­σας και αντιστρόφως (βασική πρόταση από την Α· Λυκείου). Τριγωνόμετρική έκφραση των τύπων των

τετραγώνων των διαμέσων

2 β2 + γ2 + 2βγσuνΑ Ισχύει: μα = . Αντίστοιχα 2 2 μβ και μγ .

Απόδειξη:

4

Προεκτείνουμε τη διάμεσο ΑΜ = μα κατά τμήμα ΜΑι = ΜΑ . Οπότε: ΑΒΜ = ΑιΜΓ , άρα Αι Γ = γ και ΑιΓ // ΑΒ (γιατί;)

Α

Β

β

I Ι I Ι

I Γ I

\ / Ύ I I

�-γ' 11' Αι Άρα ΜΑι = 180° -Α . Εφαρμόζουμε τώρα το νόμο των σuvημιτόνων

στο τρίγωνο ΑΑιΓ: 2 2 2 Λ ' ΑΑι =ΑΓ + ΑιΓ - 2ΑιΓ ·ΑΓσuνΑΓΑι η

4μ� = β2 + γ2 -2βγσuν (1 80° -Α) ή 4 2 - β2 2 2R., Α , 2 β2 +γ2 +2βγσuνΑ μα - +γ + 1-'rσuv η μα · 4

11. Στο τρίγωνο ΑΒΓ είναι Β = 30° • Δείξτε ότι: β2 =α2 + γ2 -αγ..J3 .

Απόδειξη Επειδή η Β = 30° οξεία με γενίκευση του Πυ­

θαγορείου Θεωρήματος στο ΑΒΓ: β2 = α2 + γ2 - 2αΒΔ ( 1)

' ΑΔ ΑΒ , ΑΔ γ αρα =- η =- . 2 2 Α

~ Γ Δ α Β

Με Πυθαγόρειο Θεώρημα στο τρίγωνο ΑΔΒ: ΒΔ2 =ΑΒ2 -ΑΔ2 = γ2 -

(�)2 = 3

�2 ή

ΒΔ = γ.fj (2). Από (1 ), (2): β2 = α2 + γ2 -αγ.J3 2 12. Δίνεται τετράπλευρο ΑΒΓΔ και Ρ το

σημείο τομής των διαγωνίων του. Φέρουμε ΑΖ, ΓΕ κάθετες στη ΒΔ. Εάν η γωνία ΑΡΔ είναι αμ­βλεία, να δείξετε ότι:

ΑΔ2 +ΒΓ2 =ΑΒ2 + ΓΔ2 + 2ΕΖ ·ΒΔ . Απόδειξη Επειδή ΑΡ Δ > 90° θα είναι ΒΡΓ > 90° . Με

γενικευμένο Πυθαγόρειο Θεώρημα στα τρίγωνα ΑΡΔ και ΒΡΓ: ΑΔ2 = ΑΡ2 + ΡΔ2 + 2ΡΔ · ΡΖ (1)

ΒΓ2 = ΒΡ2 + ΡΓ2 + 2ΒΡ · ΡΕ (2) Επειδή ΑΡΒ οξεία

(γιατί;) θα είναι και ΓΡΔ ο­ξεία.

Α

Άρα, με γενικευμένο Β'""""',-J;Ι-�-""'ι'!-.,.,Δ Πυθαγόρειο Θεώρημα για οξεία γωνία στα τρίγωνα ΑΡΒ, ΓΡ Δ αντίστοιχα, παίρ­νουμε:

ΑΒ2 = ΑΡ2 + ΒΡ2 - 2ΒΡ · ΡΖ (3) Γ Δ 2 = ΡΓ2 + Ρ Δ 2 - 2Ρ Δ · ΡΕ ( 4) Από (1) + (2) - (3) - (4), έχουμε:

Γ

ΑΔ2 +ΒΓ2-ΑΒ2-ΓΔ2 = (� +ΡΔ2 +2ΡΔ·ΡΖ)+ (ΒΡ2 + ΡΓ2 + 2ΒΡ · ΡΕ) - (ΑΡ2 + ΒΡ2- 2ΒΡ · ΡΖ) -(ΡΓ2 + Ρ Δ 2 - 2Ρ Δ · ΡΕ) = ΑΡ2 + ΡΔ2 + 2ΡΔ · ΡΖ +ΒΡ2 + ΡΓ2 + 2ΒΡ · ΡΕ ­ΑΡ2- ΒΡ2 + 2ΒΡ · ΡΖ - ΡΓ2- ΡΔ2 + 2ΡΔ · ΡΕ = (2ΡΔ · ΡΖ+ 2ΒΡ · ΡΖ) + (2ΒΡ · ΡΕ+ 2ΡΔ · ΡΕ) = 2 ΡΖ (ΡΔ+ ΒΡ)+ 2ΡΕ (ΒΡ + ΡΔ) = 2ΡΖ · ΒΔ+ 2ΕΡ · ΒΔ = 2ΒΔ (ΡΖ+ ΡΕ) = 2ΒΔ · ΖΕ ή ΑΔ2 +Βr2 =ΑW+ΓΔ2 +2ΒΔ·ΖΕ

13. Στο τρίγωνο ΑΒΓ, α = 2Jfi , β = 8 , γ = 6 • Δείξτε ότι Α = 120° •

Απόδειξη Είναι α = 2Jfi � 12 , άρα μεγαλύτερη πλευρά

η α και επομένως μεγαλύτερη γωνία η Α . Στο ορθογώνιο τρίγωνο ΑΒΔ είναι Β= 30°

ΕΥΚΛΕΙΔΗΣ Β' λστ' τ.l/52

Page 55: Ευκλειδης Β 45

Μαθηματικά yια την Β' Λυκείου

ι ι ι ι ι ι ι ι ι ι

- h - - - _ι )..';_ -----,---"'-Δ Α β=8 Γ

Έχουμε: α2 = { 2Jfi}2 = 148 β2 + γ2 = 82 + 62 = 64+ 36 = 100 Άρα: α2 > β2 + γ2 <=> Α > 90° Οπότε με γενικευμένο Πυθαγόρειο Θεώρημα

στο τρίγωνο ΑΒΓ: ΒΓ2 = ΑΒ2 + ΑΓ2 + 2ΑΓ · ΑΔ <=> ( 2Jfi)2 = 62 + 82 + 2 · 8ΑΔ <=> 148 = 100 + 16ΑΔ <=> 48 = 16ΑΔ <=>

ΑΔ = 3 =�= ΑΒ οπότε ΑΔ = ΑΒ 2 2 2

Στο ορθογώνιο τρίγωνο ΑΔΒ είναι μια κάθετη πλευρά ίση με το μισό της υποτείνουσας, άρα ΔΒΑ = 30° , δηλαδή Α1 = 60° και άρα Α = 120° .

Παρατήρηση: Άμεση λύση έχουμε από το νόμο των συνημιτόνων.

β2 + γ2 _ α2 Πράγματι: συνΑ = ή 2βγ συνΑ = 82 + 62 - 148 - - 1 . Α - 12οο 2 ·48 -

. . . - - 2 αρα - · 14. Να βρεθεί το είδος του τριγώνου ΑΒΓ,

2μ 5μ εάν: α) α = - , β = μ , γ =-3 4

β) α = 8μ , β = 3μ, γ = 5μ όπου μ δοσμένο τμήμα.

Απόδειξη 8μ 12μ 15μ α)Έχουμε: α =u · β =u · γ =U

Μεγαλύτερη πλευρά η γ. γ2 = ( 15μ)2 = 225μ2 = α2 + β2 = 12 144 =(8μ)2 +(12μ)2 = 64μ2 + 144μ2 = 208μ2 . 12 12 144 144 144

Επειδή γ2 > α2 + β2 <=> Γ > 90° . β) Πρέπει να ικανοποιείται η τριγωνική ανι­

σότητα για να υπάρχει τρίγωνο, δηλαδή I α-γi < β < α + γ .

Είναι: α - γ = 8μ - 5μ = 3μ = β , άτοπο. Άρα δεν υπάρχει τρίγωνο με τις δοσμένες πλευρές.

Παρατηρήσεις στα θεωρήματα των διαμέσων

ι 2 2 2 • μα + μβ +μγ =

2β2 + 2γ2 -α2 2α2 + 2γ2 -β2 2α2 + 2β2 -γ2 ---=---�- + + -----'----'--

4 4 4 4(α2 + β2 + γ2) - (α2 +β2 + γ2) 3 ---'---'------''---'-----'--.....:...,_-� = _ ( α2 + β2 + γ2 ) 4 4

Α

Γ

2. Έστω Θ το κέντρο βάρους τριγώνου ΑΒΓ. Τότε το άθροισμα:

Α

Γ

ΘΑ2 +ΘΒ2 + ΘΓ2 =α � )2 +(� � )2 +α � )2 =

� (μ� + μ� +μ� ) =

i [� (α2 + β2 + γ2)] =.!.(α2 + β2 + γ2 ) 9 4 . 3 15. Σε τρίγωνο ΑΒΓ είναι μμ j_ μ1 • Δείξτε ότι:

4 α) μ: +μ; = μ; β) β2 + γ2 = 5α2 γ) συνΑ �- . 5

Απόδειξη α) Στο ορθογώνιο τρίγωνο ΒΘΓ θεώρημα

διαμέσων: ΒΘ2 + ΘΓ2 = 2ΘΜ2 + ΒΓ2 ή 2 ( 2 )2 ( 2 )2 ( 1 )2 2 3 μβ + 3 μγ = 2 3 μα + � (1)

Όμως ΒΘΓ ορθογώνιο τρίγωνο (Θ = 90°) , ΘΜ διάμεσος, άρα: ΘΜ = ΒΓ ή ΘΜ = α ή 2 2 1 α ή 3α 2μα (2) 3 μα = - μα = - ή α = 2 2 3

ΕΥΚΛΕΙΔΗΣ Β' λστ' τ.l/53

Page 56: Ευκλειδης Β 45

Μαθηματικά ΎU1 την Β' Λυκείου

Μ Γ α

Η ( 1 ) δυνάμει της (2):

(2 )2 (2 )2 ( 1 )2 (� μα)2 J μβ + J μγ = 2 J μα + 2 <=> 4 2 4 2 2 2 2 2 2 2 2 9 μβ + 9 μγ = 9 μα + 9 μα <=> μβ + μγ = μα

β) Από το (α): μ� + μ� = μ� <=> (τύποι των τετραγώνων των διαμέσων) 2α2 + 2γ2 -β2 2α2 + 2β2 -γ2 2β2 + 2γ2 -α2 -----'----'--+ __ _..:....__....;._ 4 4 4

2α2 + 2γ2 - β2 + 2α2 + 2β2 - γ2 = 2β2 + 2γ2 - α2 <=>β2 +γ2 = 5α2

Μια άλλη λύση για το (β): Εφαρμόζουμε Πυθαγόρειο Θεώρημα στο τρί-

γωνο ΒΘΓ: ΒΘ2 + ΓΘ2 = α2 <=> (% μβ )2 +(% μ1 )2 = α2 <=> 4μ� + 4μ� = 9α2 <=> 2 (α2 + γ2) _ β2 + 2 (α2 + β2) _ γ2 = 9α2 <=>

β2 + γ2 = 5α2 (3) γ) Από νόμο συνημιτόνων, είναι:

β2 + γ2 _ α2 συνΑ = 2βγ ή, λόγω της (3): 5 (β2 + γ2) - (β2 + γ2) συνΑ = ή 10βγ 2 (β2 + γ2) συνΑ = �:.....__:.__:_ 5βγ

2 2 4� Όμως β + γ :::::: 2� , οπότε συνΑ ;;::: 4 ή συνΑ � - . 5

Εφαρμογή :

5�

Δίνεται ορθογώνιο τρίγωνο ΑΒΓ (Α = 90°) . Να δείξετε ότι: μ� +μ� = 5R 2 όπου R η ακτίνα του περιγεγραμμένου κύκλου. Απόδειξη :

Στο τρίγωνο ΒΘΓ θεώρημα διαμέσων.

ΒΘ2 + ΘΓ2 = 2ΘΔ 2 + ΒΓ2 <=> 2

Υ Όμως το τρίγωνο ΑΒΓ ορθογώνιο

(Α = 90°) , ΑΔ διάμεσος στην υποτείνουσα, άρα (γνωστή πρόταση): μα = � ή α = 2μα (2)

Η ( 1 ) λόγω της (2) δίνει: 4 2 4 2 2 2 2 - μβ +-μγ = - μα + 2μα <=> 9 9 9 4 ( 2 2 ) 20 2 ' 2 2 2 9 μβ + μγ = 9 μα η μβ + μγ = 5μα (3)

α 2R ή επειδή μα = 2 = z-=R η (3) τελικά δίνει: μ� + μ� = 5R 2 •

16. Δίνεται ορθογώνιο τρίγωνο ΑΒΓ (Α = 90°) και στην υποτείνουσα παίρνουμε τα

σημεία Ζ, Η ώστε: ΒΖ = ΖΗ = ΗΓ = α •

3

Να δείξετε ότι: ΑΖ2 + ΖΗ2 + ΑΗ2 =� α2 • 3 Απόδειξη

Φέρουμε τη διάμεσο ΑΜ του ορθογωνίου τριγώνου ΑΒΓ (Α = 90°) .

Α

Β Ζ Μ Η Γ Προφανώς ΑΜ διάμεσος και του τριγώνου

ΑΖΗ (γιατί;) Ακόμη, έπειδή ΑΜ διάμεσος στην υποτείνου-θ . , , ΑΜ α σα ορ ογωνιου τριγωνου, ειναι: = -2 Με θεώρημα διαμέσων στο τρίγωνο ΑΖΗ:

ΕΥΚΛΕΙΔΗΣ Β' λστ' τ.l/54

Page 57: Ευκλειδης Β 45

Μαθηματικά για την Β' Λυκείου

AZz +AH2 = 2AM2 + zH2 � 2

ΑΖ2 +ΑΗ2 = 2 ω)�' �

2 2 ΑΖ2 +ΑΗ2 =�+� 2 18 (1)

(2) Με πρόσθεση κατά μέλη των (1), (2):

2 2 2 α2 α2 α2 2 ΑΖ + ΑΗ + ΖΗ =-+-+- =- α2 2 18 9 3 17. Θεωρούμε ισόπλευρο τρίγωνο ΑΒΓ και

το κέντρο Ο τQυ περιγεγραμμένου κύκλου και τον κύκλο που διέρχεται από τις κορυφές Β και Γ και εφάπτεται στις ευθείες 08 και ΟΓ. Εάν Μ τυχαίο σημείο του κύκλου αυτού, να δείξετε ότι: ΜΑ2 = ΜΒ2 +ΜΓ2 •

Απόδειξη Έστω Λ το κέντρο του κύκλου αυτού. Είναι ΛΒ = ΛΓ , δηλαδή το τρίγωνο ΛΒΓ ι-

σοσκελές και ακόμη, επειδή ABr=90'-3<1'=fιOC', είναι και ισόπλεuρο.

Α

Μ

Δηλαδή: ΛΒ =ΒΓ = ΓΛ = α . ΜΔ διάμεσος του τριγώνου ΜΒΓ, άρα από θεώ­

α2 ρημα διαμέσων: ΜΒ2 +Μr =2ΜΔ2 +- 1 (1) 2 Η ΜΔ διάμεσος και στο τρίγωνο ΜΛΑ: ΜΑ2 + ΜΛ2 = 2ΜΔ2 + 2ΑΔ2

Ό ΑΔ α.J3 ( , , λεu , μως = -2- υψος ισοπ ρου τριγω-2α2 · 3 νου).Άρα ΜΑ2 + α2 = 2ΜΔ2 +-- �

4 2 ΜΑ2 = 2ΜΔ2 +� (2) 2

Από (1), (2) � ΜΑ2 = ΜΒ2 +ΜΓ2 . 18. Δίνεται ισόπλευρο τρίγωνο ΑΒΓ και

σημείο Δ στην προέκταση της ΒΓ, ώστε

Γ Δ = ΒΓ . Να δείξετε ότι: 4ΑΔ 2 = 7ΒΓ2 • 2

Απόδειξη Έστω ΑΚ το ύψος του τριγώνου. Είναι:

ΒΚ = ΚΓ = ΓΔ = ΒΓ 2

άρα ΑΓ διάμεσος στο τρίγωνο ΑΚΔ .. Με θεώρημα διαμέσων στο τρίγωνο ΑΚΔ:

ΚΔ2 ΑΚ2 +ΑΔ2 = 2ΑΓ2 +-- (1) 2 Α

Β Κ Δ

Έ ΑΚ ΒΓ.J3 ( ' , λεu χουμε: = -2- υψος ισοπ ρου τρι-γώνου), ΑΓ = ΒΓ , ΚΔ = 2ΚΓ = ΒΓ , οπότε η (I) � ΑΔ2 + ( Β�,/3)' = 2ΒΓ2 + Β�'

<ο> ••

. <ο>

4ΑΔ2 = 7ΒΓ2 . 19. Δίνεται κύκλος (O,R) με διάμετρο ΑΒ

και χορδή ΓΔ // ΑΒ . Εάν Ν σημείο της ΑΒ, να , δείξετε ότι: ΝΓ2 + ΝΔ2 = ΝΑ2 + ΝΒ2 (1)

Απόδειξη Φέρουμε ΟΜ .l Γ Δ οπότε Γ Δ = 2ΓΜ , ε­

πειδή Μ μέσο της Γ Δ. Από θεώρημα διαμέσων στο τρίγωνο ΓΝΔ: ΝΓ2 + ΝΔ2 = 2ΜΝ2 + ΓΔ2 ή 2 W+ΝΔ2 =2ΜΝ2 +2� ή (επειδή ΓΔ = 2ΙΜ) ή ΝΓ2 +ΝΔ 2 = 2 (Ν02 + ΜΟ2 ) + 2 (ΓΟ2 - ΜΟ2 ) (Πυθαγόρειο Θεώρημα στα ορθογώνια τρίγωνα ΟΜΝ και ΓΜΟ αντίστοιχα) ή ΝΓ2 +ΝΔ 2 = 2ΝΟ2 + 2ΜΟ2 + 2ΓΟ2 - 2ΜΟ2 =

2 (ΝΟ2 + ΓΟ2 ) = 2 (ΝΟ2 + R 2 ) (2)

Β

Το β' μέλος της (1) δίνει: ΝΑ2 + ΝΒ2 = (R + ΟΝ)2 + (R - ΟΝ)2 = 2 (R2 +0Ν2 ) (3) Από (2), (3) προκύπτει η (1). 20. Σε τρίγωνο ΑΒΓ είναι ΒΘΓ = 60° , όπου

Θ το κέντρο βάρους του τριγώνου. Να δείξετε ΕΥΚΛΕΙΔΗΣ Β' λστ' τ.l/55

Page 58: Ευκλειδης Β 45

Μαθηματικά yια την Β· Λυκείου

, μz + γz _ 5αz οτι: μμ . μΥ = 4

Απόδειξη Είναι ΒΘΓ = 60° και εάν Γ Δ .l ΘΒ , τότε

Α ΘΓ . θ . ΘΓ Δ = 30° . Οπότε: ΔΘ = - απο το ορ ογωνιο 2 τρίγωνο ΘΔΓ.

Α

Γ Εφαρμόζουμε τώρα γενικευμένο Πυθαγόρειο

Θεώρημα στο τρίγωνο ΘΒΓ: ΒΓz = ΘΒ2 + ΘΓz- 2ΘΒ · ΘΔ <=>

αΖ = (% μβ )2 + (% μΥ )z -2(% μβ )�(% μΥ) 9αz = 4μ� + 4μ;- 4μpμy <=>

z 4 z 9 z 4μpμy = 4μp + μΥ- α <=> (τύποι των τετραγώνων των διαμέσων)

4μβμy = 2γz + 2αz- β2 + 2αz + 2βz - γz - 9αz <::::> βz + γ2 _ 5αz <::::> μβ · μy = 4

21. Τριγώνου ΑΒΓ με Α< 90° , έστω η διά-μεσος ΑΜ και το ύψος ΒΔ. Ν α δείξετε ότι:

ΑΜ2 = ΒΜ2 + ΑΔ · ΑΓ . .

Απόδειξη Με θεώρημα διαμέσων στο τρίγωνο ΑΒΓ:

z ABz + ΑΓz = 2AMz + ΒΓ ή επειδή ΒΓ = 2ΒΜ 2 ABz + ΑΓz = 2ΑΜ2 + 2BMz (1) Με Πυθαγόρειο Θεώρημα στο ορθογώνιο τρί­

γωνο ΑΒΔ: ABz = ΑΔ z + ΒΔ 2 . Ακόμη: ΑΓz = (ΑΔ+ΔΓ)z =ΑΔz +ΑΓz + 2ΑΔ ·ΑΓ Οπότε:

ABz +ΑΓz = 2ΑΔz + (ΒΔΖ +ΓΔz) + 2ΑΔ · ΓΔ = '-.r------'

2ΑΔ (ΑΔ + ΓΔ) + ΒΓz = 2ΑΔ ·ΑΓ + ( 2ΒΜ)2 = 2ΑΔ ·ΑΓ + 4BMz (2)

Α

Β Μ Γ Η (1) λόγω της (2) δίνει: 2ΑΔ ·ΔΓ + 4BMz = 2AMz + 2ΒΜ2 ή AMz = BMz +ΑΔ ·ΑΓ 22. Δίνεται παραλληλόγραμμο ΑΒΓ Δ στο

οποίο ισχύει: ΑΒ4 + ΑΔ 4 = ΑΓ2 · ΒΔ 2 • Να δείξε­τε ότι Δ = 45° •

Απόδειξη Έστω ΑΓ < ΒΔ οπότε Β < 90° και Α > 90° .

Εφαρμόζουμε γενικευμένο Πυθαγόρειο θεώρημα στα τρίγωνα ΑΒΓ και ΑΒΔ.:

ΑΓz = ABz + ΒΓz- 2ΑΒ · ΒΚ (1) ΒΔΖ =ΑΔΖ +ΑΒΖ + 2ΑΒ ·ΑΛ (2)

�-------;;ιιΔ ι ι ι ι ι ι ι ι

��-----�---Α­Α : Λ Β Κ α

Με πρόσθεση κατά μέλη των (1), (2) και ε­πειδή ΒΚ = ΑΛ (γιατί;) και ΑΔ = ΒΓ , παίρ­νουμε: ΑΓz + ΒΔΖ = 2 (ABz + ΒΓz)

Από υπόθεση: ΑΓz · ΒΔ z = ΑΒ4 + ΑΔ 4 Θέτουμε ΑΒ = Γ Δ = α , ΒΓ = ΑΔ = β

και ΑΓ = κ , ΒΔ = λ οπότε: κz + λz = 2 (αz + β2) (3) κzλz = α4 + β4 (4) Δηλαδή τα κ2, λ2 είναι ρίζες της εξίσωσης xz _ 2 (αz + βz) + α4 + β4 = 0

Είναι χ = αz + βz ± αβJ2 ή κ2 = αz + βz - αβJ2 (5) Από (l) κz = αz + βz - 2α · ΒΚ (6)

Από (5), (6) <::::>2α ·ΒΚ=α/Ν2 <::::>β =ΒΚ../2 Δηλαδή το ορθογώνιο τρίγωνο ΒΚΓ είναι ι-

σοσκελές, άρα Β = 45ο . Είναι Β = Δ από το παραλληλόγραμμο

ΑΒΓΔ, άρα Δ = 45ο . 23. Σε τρίγωνο ΑΒΓ ισχύει β2 + γ2 = 2α2 •

Με βάση την ΒΓ κατασκευάζουμε εκατέρωθεν αυτής δύο ισόπλευρα τρίγωνα ΒΔΓ και ΒΕΓ. Να δείξετε ότι ΔΑΕ = 90ο •

ΕΥΚΛΕΙΔΗΣ Β" λστ" τ.l/56

Page 59: Ευκλειδης Β 45

Μαθηματικά Ύlfl την Β' Λυκείοο

Απόδειξη Το τετράπλευρο ΔΒΓΕ είναι ρόμβος (γιατί;)

και οι διαγώνιές του διχοτομούνται, οπότε το Μ είναι μέσο. και.της ΒΓ και του ΔΕ.

Δ

Ε

Στο τρίγωνο ΑΒΓ από θεώρημα διαμέσων α2 παίρνουμε: β2 + γ2 = 2μ� + - ή 2

α2 α.J3 2α2 = 2μ� + 2 ή 3α2 = 4μ� ή μα = -2-

Ε, ΔΜ α.J3 ' ' λευ ιναι = -2- ως υψος ισοπ ρου τρι-γώνου πλευράς α. Άρα ΔΕ = αJ3 .

Οπότε στο τρίγωνο ΑΔΕ η διάμεσος ΑΜ είναι ίση με το μισό της πλευράς στην οποία αντιστοι­χεί, δηλαδή της ΔΕ. Επομένως ΔΑΕ ορθογώνιο τρίγωνο, δηλαδή ·ΔΑΕ = 90° .

j ΑΣΚΗΣΕΙΣ ΠΡΟΣ Λ ΥΣΗ 1. Δίνεται κύκλος (O,R) και δύο κάθετες

χορδές του ΑΒ και ΓΔ που τέμνονται στο Σ. Να δείξετε ότι: ΑΓ2 + ΒΓ2 + ΒΔ 2 + ΔΑ 2 = 8R 2 •

ώστε ΑΒ = 2ΒΓ . Εάν Ρ τυχαίο σημείο που δεν ανήκει στην ευθεία, να δείξετε ότι:

ΡΑ2 + 2ΡΓ2 = 3ΡΒ2 + 6ΒΓ2 • �- Δίνεται τρίγωνο ΑΒΓ και τα σημεία Δ, Ε

της ΒΓ, ώστε ΒΔ = ΔΕ = ΓΕ . Να δείξέτε ότι: ΑΔ2 + ΑΕ2 = ΑΒ2 + ΑΓ2 -� ΒΓ2 •

9

5. Σε τρίγωνο ΑΒΓ είναι ΒΔ = ΒΓ . Να δεί-3

ξετε ότι: 6ΑΒ2 + 3ΑΓ2 = 2ΒΓ2 +�2 • � 6. Προεκτείνουμε τη διάμεσο ΑΜ τριγώνου

ΑΒΓ κατά ΜΕ = 2ΑΜ . Να δείξετε ότι:

ΓΕ �4β2 + 12γ2 - 3α2

� Μ

' ' 2 ' δ 7. ε κεντρο το σημειο τομης των ιαγω-νίων ενός παραλληλογράμμου ΑΒΓ Δ, γράφουμε κύκλο. Εάν Μ σημείο του κύκλου, να αποδείξε-τε ότι το άθροισμα ΜΑ2 + ΜΒ2 + ΜΓ2 + ΜΔ2 είναι σταθερό.

� 8. Σε τρίγωνο ΑΒΓ ισχύει Ρα =iα . Να

δ ίξ , 2 2 2 ε ετε οτι: Ρα = Pp + μ1 • J9. α) Να αποδείξετε ότι υπάρχει τρίγωνο με πλευρές .Jj ' J5' ·.,fiS· •

β) Να βρείτε το είδος του τριγώνου αυτού. Ο συνάδελφος Γ. Ιωαννί?ης απ' τον Χολαργό προτείνι;ι:

� ιΌ Σε τρίγωνο ΑΒΓ είναι Β = 2Γ και ΒΓ = 2ΒΑ . Να υπολογιστούν οι γωνίες του.

\Υπόδειξη: Άσκηση 2 .). 2. Σε ορθογώνιο τρίγωνο ΑΒΓ η t είναι 2. Δίνεται τρίγωνο με πλευρές 2λ, 4λ και Sλ. 15°. Αν ή υπότείvουσα ΒΓ είναι ίση προς Να βρείτε τα μήκη των προβολών των δύο άλ,..

λω� πλευρών στη μικρότερη πλευρά. · την α, να υπολογι�ούν οι δύο άλλες πλευ-3. Επί ευθείας θεωρούμε τα σημεία Α, Β, Γ ρές το� τριγώνο\i zωρίς χρήση τριγωνομε-

τρικών τύπων.

r rPirΩNOMETPIA Β' ΛΥΚΕΙΟΥ Των Χαραλαμποπούλου Λίνα -Ευθυμόγλου Πέτρος

Στο άρθρο αυτό θα ασχοληθούμε με. απλές εφαρμογές που αφορούν στον υπολογισμό των τριγωνομε­τρικών αριθμών του αθροίσματος α+β και της διαφοράς α-β γωνιών α, β.

Α. Στην Α' Λυκείου μάθαμε ότι:

εφχ1= ημχ ,χ ::;:. κπ+ π, κ ε Ζ , συνχ 2

συνχ σφχ = --,χ ::;:. κπ , κ ε Ζ ημχ

ημ2χ+ συν2χ = 1

2 1 π εφ x + l = -. -2-. , χ ::;:. κπ+- , κ ε Ζ συν χ 2

Αι . Υπολογισμός των τριγωνομετρικών

ΕΥΚΛΕΙΔΗΣ Β' λστ' τ.l/57

αριθ-

Page 60: Ευκλειδης Β 45

Μαθηματικά yια την Β' Λυκείου

μών των γωνιών α+β και ά-β αν είναι γνωστοί οι τριγωνομετρικοί αριθμοί των γωνιών α,β.

Για οποιεσδήποτε γωνίες α,β ισχύουν :

συν( α-β) = συνασυνβ +ημαημβ συν( α+β) = συνασυνβ-ημαημβ ηιi( α-β) = ημασυνβ -συνα ημβ ημ( α+β) =ημασυνβ+συναημβ

π Αν συνα :;t: Ο<::> α :;t: κπ+- ,κ Ε Ζ , 2 . π συν β :;t: Ο<=> β :;t: λπ +-,λ Ε Ζ , 2

π συν( α-β) :;t: Ο<=> α-β :;t: νπ+-, ν Ε Ζκαι . 2 συν( α+β) :;t: Ο<=> α+β :;t: μπ+ π ,μ Ε Ζτότε: 2

εφα+ εφβ εφ( α+β) = και ι -εφαεφβ εφ ( α-β) = εφα-εφβ ι + εφαεφβ

Άσκηση lη 4 π 5 3π Αν 1JD=5,o<α<2 και συνβ=-13 , π<β<2

να υπολσyιστούν οι τριγωνομετρικοί αριθμοί της γωνίας α-β.

Λύση Επειδή: η μ( α- β) = η μα συνβ -συνα ημβ και

συν ( α- β) = συνα συνβ +η μα ημβ αρκεί να υπο­λογίσουμε το συνα και το ημβ .

ι6 ' 9 Έχουμε : συν2α= ι-ημ2α= ι--=- , 25 25 π . 3 2a --)Ω 25 ι44 Ο<α<2 αρα σι:Μl=s και ημμ=ι-υυvμ=ι-1w=ιw ' 3π . ι2 π < β <- αρα ημβ = -- . 2 ι 3

Τότε: ημ( α-β) =� · -5 -� · -ι2 =_!i 5 13 5 13 65 3 -5 4 -12 63 και συν( α-β) =- ·-+- ·-=-- . 5 13 5 13 65

16 εφ(α-β) = ημ(α-β) = 65 = -_!i

συν( α- β) _ 63 63

63 σφ(α-β) = -- . ι6

65

Α ν χ+ y = 135° και εφ χ = .!_ να υπολογίσετε 2

την εφy . Λύση

• ( 0 ) εφ135° -εφχ Ειναι: εφy = εφ 135 -χ = 0 ι + εφι35 εφχ Όμως: εφ ι 35° = εφ( ι 80° - 45° ) = -εφ45° = -1

-1-_!_ άρα εφy =---f-=-3 .

ι --2 Άσκηση 3η

Άν εφα =! και εφβ =! με α,β Ε (π, 3π) 3 2 2

1) να βρεθεί η εφ (α+ β) και 2) να βρεθεί η γωνία α+β.

Λύση ι ι - +-

1 ) Είναι εφ( α+β) = εφα+ εφβ = 3 2 = 1 1 -εφαεφβ ι �_!_ . _!_

' 3π 3π 2) Ομως: π < α <- και π < β <-2 2

3 2

άρα 2π < α+β < 3π οπότε α+β = 2π +�= 9π . 4 4 Άσκηση 4η

Αν α,β e (ο,;} ημαημβ=χ και εφαεφβ=y

να βρείτε ως συναρτήσεις των χ, y το συν(α+ β) .

Λύση

'Ε χουμε: εφα εφ β = y ή η μα ημβ = y ή συνa συνβ

� = συνασυνβ . Είναι: συν( α+β) =σιΜlσιJ\.β-ψαημβ Υ χ x ( 1 -y) =--χ = , y>O. Υ Υ

Άσκηση 5η Δίνεται τρίγωνο ΑΒΓ. Αν ισχύει

(l + εφΑ)(l +εφΒ) = 2 (1) τότε: 1) Να βρείτε τη σχέση που συνδέει τις γωνί­

ες Α και Β, 2) να δείξετε ότι το τρίγωνο ΑΒΓ είναι α­

βλυγώνιο και

3) αν εφ Β = � να βρεθεί το συν (Α-Β) . 4

Λύση

ΕΥΚΛΕΙΔΗΣ Β' λστ' τ.l/58

Page 61: Ευκλειδης Β 45

Μαθηματικά ΎU1 την Β' Λυκείου

1)Έχουμε: ( 1 + εφΑ} ( l + εφΒ) = 2ή Έχουμε: β=α-(α-β) άρα 0\λ.β=ω{α-(α-β)]= 1 +εφΑ +εφΒ+εφΑεφΒ = 2 ή εφΑ + εφΒ = 1-εφΑεφΒ ( ). συνασυν( α -β) +η μα η μ( α-β) (1).

Ισχύει: 1 - εφΑεφΒ ::ι:. Ο γιατί αν εφΑεφΒ = 1 Επομένως αρκεί να υπολογίσουμε το η μα και τότε η (2) δίνει εφΑ+ εφΒ = Οπου είναι άτοπο το ημ( α-β) . γιατί δεν υπάρχουν αριθμοί με άθροισμα μηδέν και 2 2 1 15 γινόμενο θετικό.

εφΑ+ εφΒ Άρα η (2) γίνεται: = 1 1 -εφΑεφΒ π π �εφ(Α+Β) = εφ4� Α+Β= κπ+4, κ ε Ζ

' π και επειδή Ο < Α + Β < π εχουμε: Α + Β = 4 . 2) Επειδή: π Α+Β=-4 έχουμε

( ) π 3π ' ' Γ= π - Α+Β = π -- =- , αρα το τριγωνο 4 4 ΑΒΓ είναι αμβλυγώνιο. . 3) Από την (1) έχουμε: (1+εφΑ{1+�)= 2

7 Ι � ( 1 + εφΑ )4 = 2 � εφΑ = 7 . 1 3

εφΑ-εφΒ 7 - 4 17 εφ( Α-Β) = l + εφΑεφΒ = 1 +_!._ ·� = -31και

7 4 εφ2 (Α -Β) + 1 = 2 (

1 ) άρα συν Α-Β

1 289 961 συν2 (Α-Β) = 961 + 1 � συν2 (Α -Β) = 1250 ·

π Το τρίγωνο είναι αμβλυγώνιο με Γ>- άρα 2 π Ο <Α<-2

π Ο < Α < -2 και ή και π π Ο<Β<- --<-Β<Ο 2 2

π π ή -- < Α-Β<-2 2

3 1.fi οπότε συν(Α-Β)>Ο . Άρα συν( Α-Β) = ----sG . Άσκηση 6η

Για κάποιον αριθμό α του (Ο, ; ) και κά-

( π ) ' 1 ποιο μ του -2,ο δεχόμαστε οτι: συνα = "2'

συν( α -β) =.!. . Ν α υπολογιστεί το συν β. 5

Λύση

Είναι : ημ α = l -συν α = l -16 =16, π .J15 Ο < α <- άρα ημα =- και 2 4

2 ( ) 1 24 ημ2 ( α-β) = 1-συν α-β = 1 -- =- . 25 25 {Ο < α < π · {Ο < α <� Όμως: 2 η 2 π π --< β < Ο 0 < -β <-2 2

οπότε

( ) 2J6 Α , Ο < α�β < π , άρα έχουμε: ημ α-β =-5- . πο ι ι .J15 2J6 ι + 6.Ji0 τη σχέση (Ι) συνβ =4 · 5+4·-5- = 20

·

Άσκηση 7η

1 3π '1 ,.,.,{_ Αν συνα = -- και π < α < - να υπο ...... , .. -5 2

σετε την εφ(: + α) .

Λύση

Έχουμε: εφ( �+α) � εφ�:εφα 1- εφ-εφα 4

Αρκεί να υπολογίσουμε την εφα .

l + εφα 1-εφα

Είναι: εφ2α = -1--1 = 25 - 1 = 24 , συν2α π < α < 3π , οπότε εφα = J24 = 2J6 2

π 1 + 2../6 (1 + 2../6)2 25 + 4J6 Άρα εφ(4+α) = 1-2../6 = 1-24 = - 23 Α2.

1. Υπολογισμός των τριγωνομετρικών αριθ­μών της γωνίας 2α

• αν είναι γνωστοί οι τριγωνομετρικοί αριθ­μοί της γωνίας α ,ή

• αν δίνεται μία σχέση μεταξύ των τριγωνο­μετρικών αριθμών της γωνίας α.

2. Υπολογισμός των τριγωνομετρικών αριθ­μών της α από το συνημίτονο της γωνίας 2α.

Τύποι που συνδέουν τα τόξα 2α, α:

ΕΥΚΛΕΙΔΗΣ Β' λστ' τ.Ι/59

Page 62: Ευκλειδης Β 45

Μαθηματικά για την Β' Λυκείου �--------------------------------�

• ημ2α = 2ημασυνα •

συν2α = συν2α-ημ 2α = 2συν2α- 1 = 1 -2ημ2α

2 2εφα εφ α =-�-1-εφ2α

2 1 -συν2α εφ α = 1 +συν2α 2 1 + συν2α συν α =---2 2 1-συν2α

• ημ α =---2 Άσκηση lη �. 4 Sπ λογί n..ν ημα = --,π< α < - να υπο σετε:

5 4 1) Τους τριγωνομετρικούς αριθμούς της

α γωνίας - και 2

2) Τους τριΎωνομετρικούς αριθμούς της Ύωνίας 2α.

Λύση

1) Γ ίζ , 2 α 1-συνα νωρ ουμε οτι ημ "2 = 2 και 2 α 1 +συνα , , λογί συν -= , επομενως αρκει να υπο -2 2

σουμε το συνα . ( 4)2 9 Είναι : συν2α = 1 -ημ2α = 1 - -5 . = 25 και

5π επειδή π < α < -- , θα είναι συνα < Ο , οπότε 4 3 συνα = -- . 5

1 +� 2 α 1 -συνα 5 4 Έχουμε: ημ - = =--=- και 2 2 2 5

2 α 1 +συνα 1 ,0 5π συν - = = - . μως π < α < -- ο-2 2 5 4 , π α 5π , ποτε - <- <-- αρα 2 2 8

α 2.J5 α .J5 ημ-=-- συν-= -- και 2 5 ' 2 5 α

α ημ2 α 1 εφ2=--α =-2 , σφ2 =-2 . συν-2

2) Γνωρίζουμε ότι: ημ2α = 2ημασυνα και

συν2α = συν2α-ημ2α , επομένως αρκεί να υπολο-γίσουμε το συνα .

3 συνα = -- , Στό (1) βρήκαμε οπότε

2α = 2 (-4) (-3) = 24 ημ 5 . 5 25 ' συν2α = ( -�Υ -( -�Υ = ;5

24 εφ2α = 25 = - 24 σφ2α = _ _2._ . 7 7 ' 24

25 Άσκηση 2η

5

και

Αν ; < ω < π και εφω είναι ρίζα της εξίσω-

σης χ2 -χ-6 = Ο να βρεθεί η τιμή της παρά­στασης Α = 10ημ2ω -15συν2ω-3εφ2ω .

Λύση

Οι ρίζες της εξίσωσης χ2 - χ-6 = Ο είναι χ = -2 ή χ = 3 .

Ε δ , π ' 2 πει η: - < ω < π , εχουμε εφω= - . 2 , 2 1 -συν2ω Ειναι εφ ω =------1 +συν2ω

, 1 -συν2ω Άρα: 4 = <=> 5συν2ω = -3 1 + συν2ω 3

<=> συν2ω = -- . 5 Είναι: 2 2 9 16 ημ 2ω = 1-συν 2ω = 1 --=--25 25 '

� < ω < π ή π < 2ω < 2π 2 4 άρα ημ2ω = -- και 5

4 εφ2ω=

-� =j . 5

-4 -3 4 Άρα: Α = ι ο· ( 5) -15( 5)- 3 · 3 = -8+9-4 = -3 . Άσκηση 3η 1) Ν α υπολσyιστούν οι τριΎωνομετρικοί α-

θ , , π ρι μοι της Ύωνιας 12

και

2) να αποδείξετε ότι ημ 2 �-συν4 5π = -1 . 12 12 16 Λύση

1) Είναι � = 2� και 6 12 ΕΥΚΛΕΙΔΗΣ Β' λστΆ τ.l/60

Page 63: Ευκλειδης Β 45

Μαθηματικά ΎUΙ την Β' Λυκεiου

1 +σuν2� 1 +σuν� Οπότε: σuν2 � = 12 = 6 = 12 2 2

J3 1 +2 2+J3 Άρα π fi;J3 σuν-= =-- =--. 2 4 π 1-σuν2-

12 2

και ημ2 � = 12 12 2

π J3 1-σuν6 1 -2 2_J3 = 2 = -2-=-4- , άρα π J2=J3 π J2=J3 ι:; ημ 12 = 2 και εφu = �2+J3

= 2 -ν3

=

5π 6π π π π 2) σuν-= σuν(---) = σuν(---) = 12 12 12 2 12 π , 2 π 4 5π = ημ 12 . Οποτε ημ 12 -σuν U =

= ημ2�-ημ4� = 2-./3 -(2-J3J2 = 12 12 4 4 2-./3 4-4./3 + 3

4 16 = 8-4J3-7+4J3 1 = 16 16

Άσκηση 4η

I= J6 'l .....ί-Αν Ι ημα-συνα -, να υπο....., 1 ι.vετε τους 3

αριθμούς ημ2α, συν2α, εφα όταν α ε (Ο, π) . 2

Λύση

I I ,/6 . ημα-σuνα =- αρα 3 6 (ημα-συνα)2 =- � 9

2 ημ2α+σuν2α-2ημασuνα =- � 3 2 2 1 -2ημασuνα =- � 2ημασuνα = 1 -- � 3 3

1 1 2ημασuνα =- � ημ2α =- . 3 3

2 2 1 8 Άρα: σuν 2α = 1 -ημ 2α = 1 -- =-9 9

0 < 2α < π . Διακρίνουμε τις περιπτώσεις: α) π Ο < α � -4

2Ji άρα π 0 < 2α < -2

σuν2α =--3 και 2 1 -συν2α εφ α = 1 +σuν2α 1 - 2Ji

εφ2α 3 = 17 -12J2 1 + 2Ji

3

π π β) - < α <-4 2 2Ji συν2α = ---3 '

άρα π - < 2α < π 2

2 1 -συν2α 3 + 2J2 17 + 12J2 εφ α = = 1 + σuν2α 3 - 2J2 9-8 Άρα εφα =�17+ 12J2 . Άσκηση 5η

και

και

ή

οπότε

και

Για τη γωνία χ με

Sσυν2χ -18συνχ + ι = ο π , - < χ < π ι.σχuει 2

α) Να δείξετε συνχ = _.!. s

p) Να υπολσyίσετε τους τριγωνομε- τρικούς αριθμούς ημ2χ, συν2χ, εφ2χ .

Λύση

α) 5συν2χ -18συνχ + 1 = 0 � 5συν2χ -9συνχ -2 = 0(1) .

Θέτουμε: συνχ = ω οπότε η (1) γίνεται: 5ω2 -9ω-2= 0 με ρίζες ω = 2 ή ω=-.!

5 , (αδ ' ) ' 1 αρα σuνχ = 2 υνατη η σuνχ = -- . 5

1 2 2 24 β) συνχ = -- και ημ χ = 1 -σuν χ =- . 5 25 Άρα . 2,/6 ( . π ) ημχ =-5- αφου 2 < χ < π .

ημ2χ = 2ημχσuνχ = 2 2,/6 (-.!.) = - 4,/6 5 5 25 '

2 23 ' σuν2χ = 2σuν χ -1 = -- και σuνεπως 25

ΕΥΚΛΕΙΔΗΣ Β' λm' τ.l/61

Page 64: Ευκλειδης Β 45

Μαθηματικά για την Β' Λυκείου

4.J6 -- .J6 εφ2χ = ___1_L = � . 23 23

25 ΕΡΩΤΗΣΕΙΣ ΠΟΛΛΑΠΛΗΣ ΕΠΙΛΟΓΗΣ

ι Α J5 ' 2 ' • ν συνα +η μα =- τοτε το η μ α ισουται 2 1 3 1 με: Α: -- Β: - Γ: 1 Δ: -4 4 4

Ε: κανένα από τα προηγούμενα 2. η τιμή της παράστασης Ε 4 π 4 π ' ' =συν --η μ - ειναι ιση με:

8 8

Α: 2 J2 Β: J2 Γ: J2 Δ: 2 2 Ε: κανένα από τα προηγούμενα

3 Α 1 J3 ' ' . ν ημα = -2 και συνα = -2 τοτε η τιμη . J3 [.; [.; 1 της εφ2α είναι: Α: - Β: -ν3 Γ: -ν3 Δ: -3 2

Ε: κανένα από τα προηγούμενα

4. Αν ημ2α = κ, 0° < α < 90° τότε το η μα+ συνα ισούται με :

Α 1+� Β: 1 -� Γ: 1 + κ Δ: .J1 + κ Ε: κανένα από τα προηγούμενα.

5. Η εφ22,5° είναι ίση με: Α: 3 + 2J2 Β: 2J2 -3 Γ: 3 - 2J2 Δ: 2J2 Ε: κανένα από τα προηγού- μενα. 6. Αν εφχ = κ και εφy = λ , τότε

εφ ( χ + y) είναι ίση με: κλ- 1 Α: --

κ-λ Β: -- Γ: κ +λ Δ: κλ+ 1 κ+ λ 1 + κλ 1 - κλ κ-λ

Ε: κανένα από τα προηγούμενα.

η

7. Αν εφα,εφβ είναι ρίζες της εξίσωσης χ2 +κχ+λ = Ο με λ :;t: 1 τότε η εφ (α+β) είναι ίση

κ κ κ 1 + λ ' με: Α: -- Β: - Γ: - Δ: - Ε: αλλο. 1 + λ λ- 1 1-λ κ ΕΡΩΤΗΣΕΙΣ ΑΝΤΙΣΤΟΙΧΙΣΗΣ

Να aντιστοιχίσετε κάθε στοιχείο της στήλης Α με το ίσο του στη στήλη Β.

·:t� A Ε9Ι'ΒΑΗ Β ι. 2ημ20° συν20° α. εφ30° + εφ20°

2 . 1 + συν40°

β 1 - συν20°

· 1 + συν20° γ. 1

3. 1 - συν25° δ.

2εφ25°

1 - εφ2 25° 4. εφ2 1 0° 1 + συν1 40°

ε.

5. συν50° + 2ημ 2 25 2

2 1 5 στ. σφ -2

6. συν2 75° ο

ζ 2 2 25

. ημ -

7_ 1 + συν1 5: 2

η. ημ40° 1 - συν1 5 θ. 2 συν2 20°

8. εφ50° 2 - J3 ι. --4

ια. συν40°

ΕΡΩΤΗΣΕΙΣ ΑΝΑΠΤΥΞΗΣ 1. Να υπολογίσετε τους τριγωνομετρικούς α-

ριθμούς της γωνίας 165° . 2. Α ν α ε (Ο,�) και β ε (π, 3;) και

4 β 1 λογ' η μα = - , συν = -- να υπο ισετε τους 5 2 τριγωνομετρικούς αριθμούς των γωνιών α+ β , α-β .

3. Να υπολογίσετε την εφ(�+ α)και την

(π ) 2 3π εφ --α αν συνα = -- π< α <- . 3 5 ' 2 4. Αν α+ β = 240° και εφβ = .!.. να υπολογίσετε 3

την εφα . 5. Α ν συνα = 12 , α ε (π , π) να υπολογίσετε 13 2

τους τριγωνομετρικούς αριθμούς του τόξου � . 2 6. Να υπολογίσετε τους τριγωνομετρικούς α-

θ ' ' 3π ρι μους της γωνιας l6 . 7. Α ν εφα = 2 , 0° < α < 90° να υπολογίσετε

, , 3συν2α-2ημ2α την τιμη της παραστασης Α = ____ .:.;.___ 4ημ2α+ 5συν2α

ΕΥΚΛΕΙΔΗΣ Β' λστ' τ.l/62

Page 65: Ευκλειδης Β 45

Μαθη ματικά Για τη Γ ' τάξη του Λυκείου

Γεωμετρικοί τόποι στο Μιyαδικ6 Επίπεδο

Εισαγωγή Σκοπός του παρακάτω άρθρου είναι να βοη­

θήσει τους μαθητές να επεξεργάζονται ασκήσεις, που αφορούν γεωμετρικούς τόπους στο Μιγαδικό Επίπεδο.

Το πρώτο μέρος του άρθρου περιέχει στοιχεία θεωρίας και στη συνέχεια λυμένα παραδείγματα.

Το δεύτερο μέρος περιέχει γενικές ασκήσεις ενώ στο τέλος προτείνονται ασκήσεις προς λύση, παρόμοιες με τις ήδη λυμένες.

Πρόταση Η απόσταση των εικόνων δύο μιγαδικών ι­

σούται με το μέτρο της διαφοράς τους. Υ

χ

Μι Δηλαδή αν Μ1 , Μ2 είναι οι εικόνες των

μιγαδικών z1 , z2 αντίστοιχα, τότε:

(ΜιΜ2) = lzι -z2l · Παράδειγμα 1 Έστω ο μιγαδικός z -:�: Ο •

α) Να αποδείξΕτε ότι οι εικόνες των μιγα­δικών z, -z και iz.J3 είναι κορυφές ισοπλεύρου τριγώνου.

β) Αν το εμβαδόν του ισοπλεύρου είναι .J3 , να υπολογίσετε το μέτρο του z.

Λύση α) Έστω Α, Β, Γ οι εικόνες των z, -z, izJ3

αντίστοιχα. {ΑΒ) = lz - {-z� = 12� = 21� .

· (ΒΓ) = j-z - izJ3j = 1-z (ι + iJ3) = = Ι� Ιι + iν'31 = 1���2 +J32 = 2 1� .

{ΓA) = jizJ3 -� = lz(-l + iJ3) =

του Χρήστου Λαζαρίδη

= lzΙ I-t + iJ31 = Ι� · 2 = 2 1� . ( ΑΒ) = {ΒΓ) = {Γ Α) επομένως το τρίγωνο

ΑΒΓ είναι ισόπλευρο. β) Το εμβαδόν ισοπλεύρου τριγώνου πλευράς ' α2J3 Τ ΑΒΓ ' λευ ' 2 1...1 Ά α ειναι -- . ο εχει π ρα L.f • ρα, 4

4lzl2 J3 r;; 2 __,__,__= ν 3 � 1� = ι � 1� = ι . 4 Παράδειγμα 2 Να υπολογίσετε τον μιγαδικό z, που είναι

τέτοιος ώστε: l2zl = l2z -11 = l2z -η . Λύση

Υ

Β Μ

ο Α χ

Η σχέση γράφεται: 2 1� = 2 1z-� = 2 l'z -� � � lz - {0+ Oi� =

= lz-(�+Oi) = lz -( O+�i) . Έστω Μ η εικόνα του z και 0{0,0) ,

Α(� , ο) . Β( ο.�) οι εικόνες των Ο, � · �i αντί-στοιχα.

Έχουμε: {ΜΟ) = {ΜΑ) = {ΜΒ) , δηλαδή το Μ ισαπέχει από τις κορυφές του ορθογωνίου τριγώ­νου ΟΑΒ. Συμπεραίνουμε ότι το Μ είναι το περί­κεντρο του ΟΑΒ, το οποίο είναι το μέσο της υπο­τείνουσας ΑΒ. [_!+ Ο Ο+_!]

Άρα, Μ 7·---f δηλαδή Μ(� ·�) ·

ΕΥΚΛΕΙΔΗΣ Β' �, τ.l/63

Page 66: Ευκλειδης Β 45

τ λ ' 1 1 . ε ικα z =-+-ι . ' 4 4

Μαθηματικά -yια την Γ' Λυκείου

Παράδειγμα 2

Εφαρμογή ι Έστω a Ε C και ρ Ε JR με ρ > Ο • Ο γεωμε­

τρικός τόπος των εικόνων των μιγαδικών z, που είναι τέτοιοι ώστε: lz -� = ρ , είναι ο κύκλος με κέντρο την εικόνα του a και ακτίνα ρ.

Υ

χ

Μια άλλη διατύπωση είναι η ακόλουθη: Η εξίσωση lz -� = ρ παριστάνει τον κύκλο κέ-ντρου A(a) και ακτίνας ρ.

Παράδειγμα ι Να υπολογίσετε το γεωμετρικό τόπο των

εικόνων των μιγαδικών z, που είναι τέτοιοι ώ-

στε:α) ι� ίι + JS = lz� β) lz+ iz -ψ:�:Ji γ) l2z - ιΙ < 4 .

Λύση α) �2�i� +v'S= Iz������ +v'5=tz1v'5� 4� �� .J5 + .J5 = lzJ../5 � 4lzl + 5 = S lzl � �� = 5 . Ο γ.τ. είναι ο κύκλος κέντρου Ο και ακτίνας 5. β) lz + iz - η �J2 � lz(1 + i) - 1I �J2 �

�Ι(1+i)(z-�)�J2�ιι+η z ( ��ί ") �J2� 1+ι 1+ι 1-ι �J2Ιz- 1;il �J2 � Ιz-(�-�i) � 1 . Ο γ.τ. είναι τα εξωτερικά σημεία του κύκλου

, κ( 1 1) , 1 , κεντρου 2,-2 και ακτινας επισης ο γ.τ. πε-

ριλαμβάνει τα σημεία του κύκλου. γ) l2z - il < 4 � 12z -� < 4 � l2z + � < 4 � 12(z +�i) < 4� ιz- ( o-ii) < 2 . Ο γ.τ. είναι το εσωτερικό του κύκλου κέντρου

κ( 0,-i) και ακτίνας 2.

'Εστω ο μιγαδικός z, ώστε: lz - ι + 2η = J2 . . 2

α) Να υπολογίσετε το γεωμετρικό τόπο των εικόνων των w = 2z + ι + i και των w - ι + 2i .

β) Να βρεθεί ο μιγαδικός w1 , ο οποίος έχει I

το ελάχιστο μέτρο και ο w 2 , που έχει το μέyιστο μέτρο, από τόυς w του α ερωτήματος.

Λύση α) lz - 1 + 2il = J2 � Ι2z - 2 +4il =J2 � 2 �Ι2z+1+ i -3+3� =.fi � lw -(3 - 3i� =J2 . Ο γ.τ. είναι ο κύκλος κέντρου Κ(3,-3) και α-

κτίνας J2 . Ιw - 3 + 3ii =J2 � l w - 3+ 3i i =J2 � � ιw- 3- 3il =J2 � lw-1+2i-2-5il =.fi �l(w-1+2i) -(2+5i� =fi .

Ο γ.τ. είναι ο κύκλος κέντρου Λ(2,5) και α-κτίνας J2 .

β) Το μέτρο ενός μιγαδικού ισούται με την απόσταση της εικόνας του από την αρχή Ο. Οι w 1 , w 2 έχουν αντίστοιχα εικόνες τα Α, Β αντί­στοιχα, αφού (ΟΑ), (ΟΒ) είναι η ελάχιστη και η μέγιστη απόσταση της εικόνας του w από το Ο.

Υ

Η ευθεία ΟΚ έχει εξίσωση y = -χ και ο κύ­κλος έχει εξίσωση (χ - 3)2 + (y + 3)2 = 2 .

Οι συντεταγμένες των Α, Β προκύπτουν από τις λύσεις του συστήματος:

�χ�-�;2 + (y + 3)2 � 2}

� ;:� ή� = =2} Άρα, Α{2,-2) και Β{4,-4) . Τελικά, w1 = 2- 2i και w2 = 4 -4i . Παράδειγμα 3 Έστω z E C . α) Να αποδείξετε ότι η εξίσωση lz -� = ρ ,

όπου a Ε C και ρ > Ο παίρνει τη μορφή

��2 = 2Re(za) + ρ2 - 1�2 .

ΕΥΚΛΕΙΔΗΣ Β' λστ' τ.l/64

Page 67: Ευκλειδης Β 45

Μαθηματικά ΎUI την Γ Λυκείου

β) Αν ισχύει lzl2 = 2 Re(3z - 4iz)- ι6 , να υ­πολογίσετε τον Ύεωμετρικό τόπο των εικόνων του z.

Λύση α) lz - al = ρ� lz - al2 = ρ2 �

� (z-a)(z -a) = ρ2 � zz-aZ-ίiz+aa=ρ2 � � ��z - (az +az) = ρz - lalz � l�z -(za+za) =ρ2 -1�2 ���2 =2Re(za) +ρz -l�z .

β) 1�2 = 2·Re(3z-4iz) - 16� � 1�2 = 2Re(z(3-4i))- 16 � 1�2 = 2Re[z(3 + 4i)] + 32 -52 �

1�2 = 2Re[z(3+ 4i)] + 32 - l3 + 4il2 � lz - (3 +4i� = 9 . Ο Ύ.τ. είναι ο κύκλος κέντρου Κ{3,4) και α­

κτίνας 9. ΠαράδειΎμα 4. Έστω z e C ώστε izi = ι . Να υπολογίσετε

το ελάχιστο και το μέ-yιστο της παράστασης lz - ι + 2η .

Λύση Από τη σχέση I� = 1 , συμπεραίνουμε ότι η ει-κόνα Μ του z, ανήκει στον κύκλο κέντρου Ο και ακτίνας 1 . Έστω A(l , -2) η εικόνα του 1-2i. ·

Η παράσταση lz - 1 + 2� = lz - (1 - 2i� = (ΑΜ) Ύίνεται ελάχιστη, μέyιστη όταν το Μ �μπίπτει α­ντίστοιχα με το Μι , Μ2 (Σ χ 1 ). Η ευθεία ΟΑ έχει εξίσωση y = -2χ .

Τα σημεία Μι , Μ2 προκύπτουν· από τις λύ-

σεις του συστήματος:

x2 + y2 = 1}�5x2 = 1 }� χ � Js /�-Jsj

y = -2x y = -2x _ 2 _ 2 y - -J5 Υ - J5

Mι (Js .-JsJ και Μ2 (-Js. Js} Το ελάχιστο είναι

,---------

( Ί )2 ( 2 )z (ΑΜι ) = 1-J5 + -2 + J5 =J5 - 1.

Το μέγιστο είναι .--------

(AMz) = (ι + Js)2 + (-2 -Js)2 =J5+I .

Υ Mz

A(l ,-2) Σχ. 1

ΕφαρμοΎή 2

Υ

A(a) γ Σχ. 2

B(b)

χ

Έστω a, b e C με a -φ. b , τότε: ο Ύεωμετρι­κός τόπος των μιΎαδικών z, που είναι τέτοιοι ώ­στε: lz -� = lz - bl είναι η μεσοκάθετη του τμή­ματος με άκρα τις εικόνες των a, b (Σχ. 2).

Μια άλλη διατύπωση είναι η ακόλουθη: Η εξίσωση lz -� = lz - bl παριστάνει τη με-

σοκάθετη του τμήματος με άκρα A(a) , B(b) .

ΠαράδειΎμα ι Έστω z e C . α) Να αποδείξετε ότι η εξίσωση

lz -� = lz - bl , όπου a, b e C με a -φ. b παίρνει τη

μορφή 2 Re[(b - a)z] = lbl2 - lal2 • β) Να υπολο-yίσετε το Ύεωμετρικό τόπο των

εικόνων των μιΎαδικών z, που είναι τέτοιοι ώ-στε lz + ι - il = lz - 3 - 3il και στη συνέχεια να βρείτε το ελάχιστο μέτρο �ου z.

z . Ύ) Να υπολογίσετε το Ύ.τ. των εικόνων των

Λ�ση α) lz -al = lz-bl � Ιz- al2 = lz-bl2 � � ( z-a) (z-a) = ( z-b) (z-1>) �

- -zz-az-ίiz + aa = zz-bz-bz + bb � �(b-a)z+(b-a)z = lblz - lalz � �-�z+�-�z =�2 -�2 � � 2Re[(b- a)Z] = Ibl2 - lal2 . β) lz+l-� =lz-3-3� �Ιz-(-l+i� =lz-(3+3i� . Ο γ.τ. των εικόνων του z είναι η μεσοκάθετος

του ΑΒ, όπου Α{-1, 1) και Β(3,3) . Η εξίσωση της μεσοκάθετης, όπως προκύπτει

από το α) ερώτημα, αν θεωρήσουμε a = -1 + i , b = 3 + 3i και z = χ+ yi , γίνεται: 2Re[(3 + 3i + l - i) (x - yi)] = l3 + 3�2 - l- 1+ �2 � 2Re[(4+ 2i) (x -yi)] = 1 8-2 � � 4χ + 2y = 8 � 2χ + y-4 = Ο .

ΕΥΚΛΕΙΔΗΣ Β' λστ' τ.l/65

Page 68: Ευκλειδης Β 45

Μαθηματικά -yια την Γ Λυκείου

Υ

χ

Υ

----��-r--+--+x Ο Α

Σχ.3 Σχ. 4 Οι εικόνες του z ανήκουν στη μεσοκάθετη ε

και το μέτρο τους ισούται με την απόσταση της ει­κόνας τους από την αρχή Ο (Σχ. 3). Η μικρότερη απόσταση, δηλαδή ελάχιστο μέτρο, έχουμε όταν η εικόνα του z είναι η προβολή του Ο στην ε. Στην περίπτωση αυτή, το μέτρο ισούται με την απόστα­ση του Ο, από την ε, δηλαδή είναι:

d = 12 · 0 +0 - � = _!_ = 4J5 �22 + 12 J5 5

γ) Οι εικόνες των z, z είναι σημεία συμμε­τρικά ως προς τον άξονα χ'χ . Συμπεραίνουμε ότι ο γ.τ. των εικόνων των z είναι η ευθεία ε', η ο­ποία είναι συμμετρική της μεσοκάθετης ε ως προς άξονα χ'χ. Η εξίσωση της ε', προκύπτει αν στην εξίσωση της ε, θέσουμε όπου y το -y.

Τελικά, ε' : 2χ - y -4 = Ο .

Παράδειγμα 2 Να βρείτε το σύνολο των εικόνων των μιγα-

δικών z, που είναι τέτοιοι ώστε: ι; -ιι s; ι .

Λύση

ι;-1ι � 1 � ι2:zl � 1 � 121�� � 1 �

� 12- � � ι� � ιz- (2 + 0i� � ιz - (o + oi� .

Ο γ.τ. είναι το αριστερό ημιεπίπεδο που ορίζει η μεσοκάθετη του ΟΑ (Σχ. 4), όπου Ο, Α οι εικό­νες των Ο, 2 αντίστοιχα. Εξαιρείται το σημείο Ο, διότι πρέπει z -::;:. Ο .

Εφαρμογή 3 Έστω a,b E C και a > O , τότε: ο γ.τ. των

εικόνων των μιγαδικών z, που είναι τέτοιοι ώ­στε lz -� + lz -bj = 2α είναι η έλλειψη με κέντρο

Ο, εστίες τις εικόνες των a, b και σταθερό ά­θροισμα 2α.

Υ

χ

Σχ. 5

Μια άλλη δια­τύπωση είναι η α­κόλουθη:

Η εξίσωση lz - ai + lz -� = 2α παριστάνει έλλειψη με κέντρο Ο και εστί­ες E'(a) , E(b) .

Παρατήρηση. Αντίστοιχη εφαρμογή έχουμε και για την υπερβολή.

Παράδειγμα ι Έστω z E C ώστε: jz+ 3l + lz-3j = ιO (ι). α) Να υπολογίσετε το γ.τ. των εικόνων του

z και να βρείτε την εξίσωσή του. β) Να βρείτε τους φανταστικούς που ικα-

νοποιούν τη σχέση (ι). Λύση α) {l) � lz- (-3 + 0i� + lz - (3 + 0i� = 10 Ο γ.τ. είναι έλλειψη με κέντρο Ο, με εστίες

Ε'( -3,0) , Ε (3,0) και σταθερό άθροισμα 10. Έχουμε:

��::ο }� �=� } β2 = α2 -γ2 β2 = 16

2 2 Η έλλειψη έχει εξίσωση: � + L = 1 . 25 16 β) Οι ζητούμενοι φανταστικοί έχουν εικόνες

τα σημεία τομής της έλλειψης με τον άξονα y'y . εξίσωση της έλλειψης για χ = Ο γίνεται: y2 = 1 6 � Υ = -4 ή Υ = 4 .

Τελικά, οι φανταστικοί είναι οι -4ί και ο 4ί. Εφαρμογή 4 Έστω φ Ε [ο, 2π) ,

τότε: ο γ.τ. των εικό­νων των μιγαδικών z, που είναι τέτοιοι ώστε Arg(z) = φ είναι η η-

Υ

χ

μιευθεία Ot, εκτός Σχ. 6 του σημείου Ο, η οποία σχηματίζει με τον άξονα χ'χ γωνία φ.

Παράδειγμα ι Έστω z Ε ιc* ώστε ένα όρισμα του z + zi να

' 3π ειναι το - .

2 α) Να αποδείξετε ότι η εικόνα του z ανήκει

σε μία ημιευθεία, της οποίας να υπολογίσετε την εξίσωση.

ΕΥΚΛΕΙΔΙΙΣ Β' λστ' τ.l/66

Page 69: Ευκλειδης Β 45

Μαθηματικά yuι την Γ Λυκεicn>

i β) Που ανήκουν οι εικόνες των z , ' z z2004 . '

Λύση

α) Έστω Arg(z) = φ , φ ε [Ο,2π) . Έχουμε: z + zi = z(1 + i) .

1 + i = .J2(συν π + iημ�) , άρα, Arg(1 + i) = � . 4 4 4

'Ενα όρισμα του z(l + i) είναι το π + φ , επο-4 ' π 3π 5π μενως, - +φ = - <=> φ = - ε [Ο,2π) . 4 2 4

Υ

χ

Σχ. 7

Η εικόνα του z α­νήκει στην ημιευ-θεία Ot1 , εκτός του Ο. Η ευθεία αυτή έχει κλί­ση

και διέρχεται από το Ο, άρα θα επαληθεύει την ε­ξίσωση: y = x με χ < Ο .

β) Οι εικόνες του z ανήκουν στη συμμετρι­κή της Ot ως προς χ'χ .

Η συμμετρική της Ot είναι η ημιευθεία Ot2 , που έχει εξίσωση y = -χ με χ < Ο .

Ar . π ' ' ' i ' gι = - , αρα, ενα ορισμα του - ειναι το 2 z π 5π 3π 8π 5π . 5π - -- = -- = --+ - = 2(-l)π +- . Οι ει-2 4 4 4 4 4 κόνες του i_ ανήκουν επίσης στην Ot2 εκτός του z Ο.

Ένα όρισμα του z2004 είναι το 2004 · 5π 2505π = 2 · 1 525π + π . Οι εικόνες των 4 z2004 ανήκουν στον ημιάξονα Οχ' εκτός του Ο.

Γενικές Ασκήσεις Άσκηση 1 α) Έστω ρ Ε IR και z Ε C ώστε:

JRe(z)+ ρJ = Jz - ρJ (1). Να αποδείξετε ότι ο γ.τ. των εικόνων του z είναι μία παραβολή.

β) Να υπολογίσετε το γ.τ. των εικόνων των μιγαδικών z, που είναι τέτοιοι ώστε JRe(z)- 2J = Jz + 2J .

Λύση

α) Έστω z = x + yi , x, y ε JR , οπότε M(x,y) είναι τυχαίο σημείο του γ.τ.

Έχουμε: (1) <=> Jx+R = i(x -ρ)+yij <=> <=> Jx + ρJ2 = J( χ _ ρ) + yiJ2 �

χ 2 + 2ρχ + ρ2 = (χ -ρ )2 + y2 <=> χ 2 + 2ρχ + ρ2 = = χ2 -2ρχ + ρ2 + y2 <=> y2 = 4ρχ .

Ο γ.τ. είναι η παραβολή με εξίσωση y2 = 4ρχ .

β) Από το α, αν ρ = -2 , έχουμε ότι ο γ.τ. εί­ναι η παραβολή με εξίσωση y2 = -8χ .

Άσκηση 2 Ν α υπολογίσετε τον μιγαδικό z 'Φ Ο , ο οποί-

ος είναι τέτοιος ώστε ln JzJ = Ο και Arg( z) = π . 4

Λύση · y

Inl� = ο <=> ι� = 1 , άρα η εικόνα Μ του · z, ανήκει στον κύκλο κέ­ντρου Ο και ακτίνας 1 , ο οποίος έχει εξίσωση χ2 + y2 = 1 . Σχ. 8

χ

Arg ( z) = � , άρα η εικόνα Μ του z, ανήκει στην 4 ημιευθεία Ot, που σχηματίζει με τον άξονα χ'χ ,

' π ' ' ξ' ο γωνια "4 , η οποια εχει ε ισωση y = χ , με χ > .

Το σύστημα {χ2 + y2 = 1, y = χ, χ > Ο} έχει τη

λύση (χ,ψ ( � ,�} Τελικά, z =!f-+ i!f- .

Άσκηση 3 Έστω z Ε C με z .φ i , και ο μιγαδικός z+ i Ν μ , , λο , w = --. . α ρειτε το συνο των εικονων Ζ - 1

των μιγαδικών z, που είναι τέτοιοι ώστε: 3π α) Arg(w)= - , 2

β) π Arg(w) = - . 4

Λύση •

Θέτουμε z = x + yi , x, y ε JR με (x, y) :;ιt (O, l) . Έχουμε: x + (y+ l) i [x + (y + 1)i] [x - (y - 1) i]

w - - -- x +(y - 1)i - [x + (y - l)i][x - (y - 1)i] -

χ2 + y2 - 1 2χ . = 2 + 2 1 ' x2 + (y - 1) x2 + (y - 1) ΕΥΚΛΕΙΔΗΣ Β' λστ' τ.l/67

Page 70: Ευκλειδης Β 45

Μαθηματικά yια την Γ Λυκείου

3π w ε Ι } α) Arg(w) = l � hn(w) < O �

� χ2 + y2 - ι = Ο}� χ

2 + y2 = ι} 2χ < Ο χ < 0

Υ

········ ... \

. �

Το σύνολο των εικόνων είναι το τόξο του κύκλου

χ χ2 + y2 = ι με χ < Ο .

Σχ. 9 β) Αrg(w) = � � {εφ π = 2

2χ2 , 2χ > 0} 4 4 χ + y - ι

� {(χ - ι)2 + y2 = 2, χ > Ο} Υ Το σύνολο των ει­

κόνων είναι το τόξο του κύκλου

(χ - ι)2 + y2 = 2 με χ > Ο .

Άσκηση 4

χ

Σχ. ιο

'Εστω z e C τέτοιος ώστε lz - il S ι και lz - 2il = ι . Ν α αποδείξετε ότι: ι S lzl S J3 .

Λύση

χ

Σχ. 1 1

Έχουμε: !z - il � ι � jz - (O + i� � ι . Άρα, η εικόνα Μ του z ανή­κει στον κυκλικό δίσκο κέ­ντρου Κ(Ο, 1) και ακτίνας 1 .

Όμοια παίρνουμε: lz - 2il = ι � lz - (0 + 2i� = 1 .

Άρα η εικόνα Μ του z ανήκει στον κύκλο κέντρο Λ(Ο,2) και ακτίνας 2.

Συμπεραίνουμε έτσι ότι η εικόνα Μ του z, α­νήκει στο τόξο ΑΒ.

Για κάθε σημείο Μ του τόξου ΑΒ, έχουμε: (ΟΚ) � (ΟΜ) � (ΟΑ) , άρα, 1 � I� � J3 .

(ΟΑ2 =ΟΧ -Μ =22 -12 =3 , άρα (ΟΑ) = J3 ). Άσκηση 5 'Εστω οι μιγαδικοί z, w ώστε z2 + w2 = Ο •

α) Αν /z/ = ι , να βρείτε το σύνολο των εικό-νων των w.

β) Αντίστοιχα, αν iz -ι+ 2il = ι . Λύση

z2 + w2 = Ο � w2 - i2z2 = Ο � � (w - iz}(w + iz) = O � w = izή w = -iz . α) Ι� = ι � l±ill� = 1 � l±i� = 1 � Ιwl = ι . Το σύνολο των εικόνων των w είναι ο κύκλος

κέντρου Ο και ακτίνας 1 . β) lz - 1 + 2� = 1 � lillz - 1 + 2il = 1 �

� ji(z - 1 + 2i� = ι � liz- i-� = 1� lw-(2+i� = 1 . Το σύνολο των εικόνων των w είναι ο κύκλος

κέντρου (2, 1 ) και ακτίνας 1 , αν w = iz . Ιz- 1+2� = 1 � 1-iz + i - � = 1 � ιw-(-2-i� = 1 .

Το σύνολο των εικόνων των w είναι ο κύκλος κέντρου ( -2, -1) και ακτίνας ι , αν w = -iz .

Άσκηση 6

'Εστω z e C με lzl = !.. Αν w = z -!. , να α-

2 z ποδείξετε ότι οι εικόνες του w κινούνται σε μια έλλειψη.

Λύση

Έστω z = α+ bi . ι 1 Έχουμε l� = - � α2 +b2 = - (1) και 2 4

1 z z . α-βί w = z-- = z-- = z -- = α+ βι --- = z zz 1�2 _!_ 4

= α+βi - 4α + 4βί = -3α+ 5βί . χ} α = --χ = -3α 3 Έστω w = χ + yί , άρα }� y = 5β β = Υ

5 Αντικαθιστώντας στην (1) προκύπτει:

2 2 χ Υ 1 , , ξ' .c:'l 'I -- + - = , η οποια ειναι ε ισωση Ι:JΙ.ΙΙλ;.ιψης. 9 25 Προτεινόμενες 1) Έστω z, w ε C με w :;t: Ο . Να αποδείξετε

ότι το τρίγωνο με κορυφές τις εικόνες των μιγαδι­κών z + w , z - w και z + iw.J3 είναι ισόπλευρο.

2) Να υπολογίσετε το γ.τ. των εικόνων των μιγαδικών z, που είναι τέτοιοι ώστε: α) l2z - 4il = 3 β) 11 + i + 2i� � l-2iz + η

γ) π �Arg(z) � Ίπ δ) lz + 5l - lz - 5l = 8 . 3 6 3) Έστω z ε C . Να υπολογίσετε το γ. τ. των

εικόνων του z, που είναι τέτοιοι ώστε: α) /z - 2i/2 - /z + �2 = 5 β) lz - 2il2 + lz + �2 = 5 γ) lz ,... 2il = 2 lz + � .

ΕΥΚΛΕΙΔΗΣ Β' λστ' τ.l/68

Page 71: Ευκλειδης Β 45

Μαθηματικά ΎUΙ την Γ Λυκείου

4) Έστω z ε <C τέτοιος ώστε -1 :$; Re{z) :$; 1 , lz - � :$; 2 και -1 :$; hn{z) :$; 1 . Να γραμμοσιαάσετε στο μιγαδικό επίπεδο το σύνολο των εικόνων των μιγαδικών z.

5) Έστω z = {λ - 1) + {λ + 2)ί , όπου λ ε JR . Να βρεθεί ο γ.τ. των εικόνων των μιγαδικών z και να υπολογιστεί εκείνος, ο οποίος έχει ελάχιστο μέ­τρο.

6) Έστω z ε <C ώστε I� = 2 και

Arg(z - 1) =: . α) Να υπολογίσετε το μιγαδικό z. β) Αν w ε <C ώστε liwl = 2 και

Arg{iw - 1) = π , να υπολογίσετε τον w. 4

7) Ο γ.τ. των εικόνων των μιγαδικών z είναι ο κύκλος κέντρου Ο και ακτίνας 1 . Να υπολογίσε­τε το γ.τ. των εικόνων των μιγαδικών w, όταν:

α) z + izJ3 . w = -iz β) w = γ) w = z + 2ι . 2 8) Έστω z ε <C ώστε lz - 1 - � = 1 . α) Να βρείτε τους μιγαδικούς z1 , z2 που έ­

χουν αντίστοιχα ελάχιστο και μέγιστο μέτρο. β) Να εξετάσετε αν υπάρχει z με πρωτεύον

όρισμα αμβλεία γωνία. 9) Έστω ο z ε <C , που είναι τέτοιος ώστε ένα

, z , 35π Ν δ ίξ ορισμα του ι::; να ειναι -- . α απο ε ετε 1 + ivJ 3

ότι η εικόνα του z ανήκει σε μία ημιευθεία. ι Ο) Έστω z ε <C τέτοιος ώστε lz - η :$; 1 και

lz - � = 1 . Να αποδείξετε ότι: 1 :$; I� :$; J3 . ll)Έστω w,z ε <C με w � JR και α ε JR , έτσι

, w - wz Ν β , , λο , ωστε: = α . α ρειτε το συνο των εικο-1 - z νων του z.

• Γ' ΛΥΚΕΙΟΥ: ΜΑθΙΙΜΑΤΙΚΑ ΓΕΝΙΚΙΙΣ ΠΑΙΔΕΙΑΣ θΕΜΑΤΑ ΤΟΥ ι ου ΚΕΦΑΛΑΙΟΥ

Στόχος του άρθρου αυτού είναι να βοηθηθεί ο μαθητής της Γ Λυκείου στα ζητήματα του ι ου

κεφαλαίου των Μαθηματικών Γενικής Παιδεί­ας (ιδίως της θεωρητικής κατεύθυνσης ... για ευνόητους λόγους).

Στη συνέχεια γίνεται ταξινόμηση των θεμά­των του κεφαλαίου ανά παράγραφο. τίθενται οι κυριότερες ερωπ]σεις θεωρίας και δίνεται η μεθοδολογία yιa την επίλυση των ασκήσεων διαφόρων τύπων. Τέλος μέσα από κατάλληλες

ασκήσεις που λύνονται επαληθεύονται οι μεθο­δολογίες καθώς και οι βασικοί· άξονες δουλειάς

του ι ου κεφαλαίου.

Στο τέλος του άρθρου παραθέτουμε τα θέ­

ματα των πανελληνίων εξετάσεων των τελευ­ταίων ετών που αντιστοιχούν στο ι ο κεφάλαιο.

Α. Συναρτήσεις . - Όριο και συνέχεια συ­νάρτησης

Ερωτήσεις θεωρίας ι . Ποιο είναι το πεδίο ορισμού μιας συνάρ-

τησης: α) πολυωνυμικής β) ρητής γ) με τύπο ριζικό δ) τριγωνομετρικής ε) εκθετικής στ) λογαριθμικής 2. Να δοθούν οι ορισμοί για την γνησίως αύ­

ξουσα και γνησίως φθίνουσα συνάρτηση f σε ένα διάστημα Δ του πεδίου ορισμού της.

του Βασίλη Καρκάνη 3. Να δοθούν οι ορισμοί του τοmκού μέγι­

στου και τοπικού ελάχιστου μιας συνάρτησης f σε ένα διάστημα Δ του πεδίου ορισμού της.

4. Να διευκρινίσετε λεκτικά την ισότητα: lim f {χ) = .e με χ0 , .e ε JR . Χ�Χο 5. Να γράψετε τις ιδιότητες των ορίων. 6. Να δοθούν οι ορισμοί: α) της συνέχειας μιας συνάρτησης στο ση­

μείο χ 11 που ανήκει στο πεδίο ορισμού της β) της συνέχειας μιας συνάρτησης στο πεδίο

ορισμού της. Λυμένες ασκήσεις Άσκηση ι Δίνεται η συνάρτηση f(x)=.J9-x1 + lnx

ex-1 χ - 1

.rx - 1 .

α) Να βρεθεί το πεδίο ορισμού της f. β) Να υπολογιστεί το lim f (χ) . :ι-+1 γ) Έχει νόημα να εξετάσουμε τη συνέχειQ

της f στο χ0 = 5 ; δ) Να εξεταστεί η συνέχεια της f στο διά­

στημα (Ο,ι).

ΕΥΚΛΕΙΔΗΣ Β' λστ' τ.l/69

Page 72: Ευκλειδης Β 45

Μαθηματικά yια την Γ Λυκείου

Λύση α) Πρέπει: 9-χ2 � ο -3 � χ � 3 χ > Ο χ > Ο e

x-1 -::f:. O � Χ Ε �

-Γχ-1 -::f:. Ο x -::f:. l χ � Ο χ � Ο οπότε Α = (0, 1)u (1, 3] . β) Είναι:

O < x < l � ή

1 < χ � 3

limf(x) = ιim[)9-x2 + ln�- �1 ] = Χ--+1

. Χ--+1 e

X VX - 1 ι. r:;--:;9 2 ι· Ιη χ ι· χ -1 ιmν�- χ- + ιm--1 - ιm----r- = χ--+1 Χ--+1 eX- Χ--+1 V Χ -: 1

γ) Εξ' ορισμού ό'Ί), εφόσον το 5 ιιι: Α . δ) Στο διάστημα (0, 1) η συνάρτηση f ορίζεται

οπότε θα είναι συνεχής ως αποτέλεσμα πράξεων συνεχών συναρτήσεων.

Παρατήρηση 1 α) Για το πεδίο ορισμού μιας συνάρτησης

υπενθυμίζουμε τα εξής: ί. Το συμβολίζουμε συνήθως με Α ιι. Οι πολυωνυμικές συναρτήσεις ορίζονται

στο. � Για παράδειγμα η συνάρτηση f(x) = x5 -3x3 + x2 -10 έχει Α =� .

ίίί. Οι ρητές συναρτήσεις ορίζονται στο � ε­κτός από τις τιμές της μεταβλητής που μη­δενίζουν τον παρονομαστή. Για παράδειγ­

χ3 μα η συνάρτηση g (χ) = 2 έχει Α =�- { 2,3} χ2 -5x +6 -::f:. O .

χ - 5χ +6 εφόσον πρέπει

ίν. Όταν ο τύπος της συνάρτησης έχει ρι-ζικό πρέπει το υπόρριζο να είναι θετική ποσότητα ή μηδέν. Για παράδειγμα η συ-νάρτηση h(x) =V25 -x2 έχει Α = [-5,5] εφόσον πρέπει: 25 - χ 2 � Ο .

ν. Οι εκθετικές συναρτήσεις ορίζονται στο � . Για παράδειγμα η συνάρτηση φ (χ) = e2χ+Ι έχει Α = � .

νι. Οι λογαριθμικές συναρτήσεις ορίζονται για θετικές ποσότητες. Για παράδειγμα η

συνάρτηση σ (χ) = Ιη (3χ - 1) έχει Α= (k,+oo) εφόσον πρέπει: 3χ - 1 > Ο .

νίί. Οι τριγωνομετρικές συναρτήσεις .f1 (χ) = ημχ , f2 (χ) = συνχ ορίζονται στο � . Ενώ οι συναρτήσεις f3 (χ) = εφ χ και f4 (χ) = σφχ ορίζονται στο � εκτός από τα χ εκείνα για τα οποία ισχύει συνχ * Ο και ημχ * Ο αντίστοιχα.

β) Για το όριο μιας συνάρτησης f(x) στο χ0 διακρίνουμε τις περιπτώσείς�

i) Αν το χ0 Ε Α τότε το lim f( χ) το υπολο-χ---tχ0 γίζουμε με αντικατάσταση, θέτοντας όπου χ στον τύπο της συνάρτησης το χ0 • Δηλαδή ισχύει: lim f (χ) = f ( Χ0 ) . Χ4Χο

ii) Αν το χ0 ιιι: Α (στα Μαθηματικά γενικής παιδείας συνήθως έχουμε είτε ρητή συνάρτηση με το χ0 ρίζα του αριθμητή και παρονομαστή, είτε συνάρτηση κλάσμα με ριζικά στον αριθμητή ή πα­ρονομαστή) τότε κάνουμε παραγοντοποίηση σε α­ριθμητή και παρονομαστή ή πολλαπλασιάζουμε ό­ταν έχουμε ριζικά με την συζυγή παράσταση · του αριθμητή ή παρονομαστή για να κάνουμε στη συ­νέχεια aπλοποιήσεις ώστε να αρθεί η aπροσδιορι­στία και να υπολογίσουμε στη συνέχεια το όριο με τον τρόπο που εκθέσαμε στο (ί).

γ) Για τη συνέχεια μιας συνάρτησης στο χ0 έχουμε τα εξής:

ί) Αν το χ0 ιιι: Α τότε εξ' ορισμού η εξέταση της συνέχειας της συνάρτησης στο χ0 δεν έχει νόημα.

ίί) Αν το χ0 ε Α και lim f (x) -::f:. f (x0 ) τότε Χ---+Χο

πάλι η f δεν είναι.συνεχής στο χ0 • ίίί) Αν το χ0 ε Α και lim f (x) = f (x0 ) τότε η

Χ�Χο

f είναι συνεχής στο χ0 •

Επιπλέον υπενθυμίζουμε ότι αν μια συνάρτη­ση είναι συνεχής σε κάθε σημείο χ0 του πεδίου ο­ρισμού Α τότε λέμε ότι είναι συνεχής στο Α.

Τέλος μια συνάρτηση που προκύπτει ως ά­θροισμα ή γινόμενο ή πηλίκο ή σύνθεση κλπ. συ­νεχών συναρτήσεων σ' ένα διάστημα Δ είναι συ­νεχής στο Δ.

Για παράδειγμα η συνάρτηση f ( ) ln x , , , λ χ = --2 -+ ημχ ειναι συνεχης στο συνο ο χ - 1 Α = (Ο, 1) υ ( 1, +οο) όπου ορίζεται.

ΕΥΚΛΕΙΔΗΣ Β' λστ' τ.Ι/70

Page 73: Ευκλειδης Β 45

Μαθηματικά για την Γ Λυκείου Οι γνωστές μας συναρτήσεις, πολυωνυμικές,

τριγωνομετρικές, εκθετικές, λογαριθμικές αλλά και όσες προκύπτουν από πράξεις μεταξύ αυτών είναι συνεχείς συναρτήσεις.

Άσκηση 2 Δίνονται οι συναρτήσε�

f (Χ) = {ΚΧ2 + λχ - 2, Χ ::# 2' 2 , χ = 2

g(χ) = {ΚΧ+ λ, Χ :# 0 4 , χ = Ο

και h(x)= {�=� , χ ε [Ο,3)u(3,-+<ο) με μ , χ = 3

κ, λ, μ ε JR . α) Να βρεθεί η τιμή του πραγματικού α­

ριθμού μ ώστε η συνάρτηση h να είναι συνεχής στο χ0 = 3 .

β) Για την τιμή του μ που βρέθηκε στο (α) ερώτημα είναι η συνάρτηση h συνεχής στο πε­δίο ορισμού της;

γ) Να βρεθούν οι πραγματικοί αριθμοί κ, λ ώστε η συνάρτηση f να είναι συνεχής στο χ1 = 2 και η συνάρτηση g να είναι συνεχής στο χ2 = Ο .

δ) Για τις τιμές των κ, λ που βρέθηκαν στο ερώτημα (γ) να υπολογιστεί το όριο:

. g(O)+f(x)- x · g(x)+ x + x2 - 2f(2) lιm . Ι� Χ-1

Λύση α) Η συνάρτηση h θα είναι συνεχής

χ0 = 3 όταν ισχύει ιimh(x) = h(3) (1).

Κ--+3

Ε, ι· h ( ) ι· Fx -J3 ιναι: ιm χ = ιm = Χ--+3 Κ--+3 Χ -3

= lim (Fx-J3){Fx + J3)

= <--+3 (x - 3)(Fx+J3)

lim y3 = Κ--+3 (y])( νΓχ + J3) ι· 1 1 1 χι� Fx +J3 = J3 +J3 =

2J3 ·

στο

Ακόμη h (3) = μ . Έτσι λόγω της (1) πρέπει: 1 μ =

2./3.

β) Γ ι ' ια μ = ι;; ειναι 2ν3 {Fx -J3 , χ

.

ε [0,3)u(3,-+<o) h (x) = f -3

2J3 , χ = 3

Προφανώς για κάθε χ0 ε [Ο, 3) u ( 3, +οο) είναι

ιim h (χ) = h ( χ0 ) και λόγω του (α) ερωτήματος το Χ----+Χο

ίδιο ισχύει και για χ0

= 3 . Άρα η συνάρτηση h εί­ναι συνεχής στο πεδίο ορισμού της Α = [Ο, +οο) .

γ) Η συνάρτηση f θα είναι συνεχής στο χ1 = 2 όταν:

ιimf ( χ) = f (2) <:::} ιϊm(κχ2 + λχ - 2)= 2 <:::} κ--+2 χ--+2 <::> 4κ + 2λ-2 = 2 <:::} 4κ + 2λ = 4 <:::} 2κ + λ = 2 (2).

Επίσης η συνάρτηση g θα είναι συνεχής στο χ2 = 0 όταν:

limg(x) = g(O) <::> ιϊm(κχ + λ) = 4 <::> λ = 4 κ--+0 κ--+0

και από την (2) είναι: κ = -ι . δ) Για κ = -1

f (x) = {-χ2 + 4χ - 2, χ * 2 2 , χ = 2

( ) {-x + 4, x :;t: O , g χ = 4 , χ = 0 οποτε:

και λ = 4 και

ιim g (Ο) + f (χ) -xg (χ) + χ + χ 2 - 2f ( 2) =

χ--+1 χ - 1 . 4 + (-χ2 + 4χ - 2)-χ (-χ + 4) + χ + χ2 -4

ιιm = κ--+1 χ - 1

η η

ι. -χ2 + 4χ -2 + χ2 -4χ + χ + χ2 ι· χ2 + χ - 2 ιm = ιm = Χ--+1 Χ � 1 Χ--+1 Χ -1

(χ - 1)(χ + 2) ιϊm = ιϊm(χ + 2) = 3 . χ--+1 χ - 1 χ--+1

Β. Η έννοια της παραγώγου Παραγώγιση βασικών συναρτήσεων

. Κανόνες παραγώγισης Εφαπτομένη συνάρτησης

Ερωτήσε� θεωρίας 1. Να δοθεί ο ορισμός της παραγώγου μιας

συνάρτησης στο σημείο χ0 του πεδίου ορισμού της.

2. Τις ερωτήσεις που ακολουθούν να χαρα­κτηρίσετε ως σωστές (Σ) ή λάθος (Λ).

Η παράγωγος μιας παραγωγίσιμης συνάρτη­σης f σ' ένα σημείο χ0 του πεδίου ορισμού της εκφράζει:

ΕΥΚΛΕΙΔΗΣ Β' λστ' τ.l/71

Page 74: Ευκλειδης Β 45

Μαθηματικά ΎUΙ την Γ' Λυκεiοu

α) το ρυθμό μεταβολής της y = f (χ) ως προς χ όταν χ = Χ0 •

β) το συντελεστή διεύθυνσης της εφαπτομέ­νης της γραφικής Παράστασης της f στο σημείο ( Χο, f( χο )) ·

3. Να σχολιάσετε τη συνέχεια και παραγωγι­σιμότητα της συνάρτησης f (χ) = I xj στο σημείο χ0 = 0 .

4. Πώς ορίζεται η εξίσωση της εφαπτομένης της γραφικής παράστασης της συνάρτησης f στο σημείο (x0, f(x0 }) ;

5. Πώς ορίζεται η συνάρτηση της πρώτης και δεύτερης παραγώγου μιας συνάρτησης f;

6. Να δείξετε όn: α) (c)' = 0 , c E IR β) (χ)' = 1 γ) (x2 J = 2χ

7. Να συμπληρώσετε τις ισότητες: α) (�)' = · · · β) (:2 )'

= · · · γ) (.Γχ) = · · ·

δ) (xP J = · · · , ρ ρητός ε) (ημχ)' = · · · στ) ( συνχ )' = · · · ζ) ( e' J = · · · η) (Ιη χ)' = · · · θ) (εφχ)' = · · · ι) (σφχ)' = · · ·

8. Ν α δείξετε ότι: α) [ c · f (χ)]' = c · f' (χ) , c Ε lR

β) [ f (χ) + g (χ)]' = f' (χ) + g' (χ) 9. Να συμπληρώσετε nς ισότητες: α) [f (x) ·g (x)J' = · · ·

β) [:i:iJ � .

γ) [r (g(x))J' = · · · 10. Να συμπληρώσετε τις ισότητες: α) (U (χ) τ) = . . . ' ν Ε Ν *

β) (4r(x) J = · .. , ν Ε Ν �

γ) [ημf(χ)J' = · · · δ) [ συνf (χ)]' = · · ·

ε) [ εφf(χ)J' = · · · στ) [ ef(x) ]' = . . .

ζ) [Iηf(x)J' = · · · Παρατήρηση 2 Στις ερωτήσεις που προηγήθηκαν και ζητείται

η παράγωγος διαφόρων συναρτήσεων, δεν αναφέ­ρουμε διαστήματα, υπονοώντας όn οι απαντήσεις θα δίνονται για τα διαστήματα που αυτές έχουν νόημα.

Λυμένες ασκήσε-ις λσκηση 3 Δίνεται η συνάρτηση ι παραΎοηίσιμη στο

(0,+«>) ώστε να ισχύει: - ι(χ2 +5χ)= 2χ7 +71nx . Να βρεθεί η εφαπτομένη της Ύραφικής παρά­στασης της συνάρτησης ι στο σημείο Μ(6,ι(6)).

Λύση Έστω ε: y = λχ +β η ζητούμενη εφαπτομένη. Για τη συνάρτηση f έχουμε: f(x2 + 5x)= 2x7 + 7lnx (1). Το 1° μέλος της (1) είναι σύνθεση της συνάρτησης f με τη συνάρτηση χ2 + 5χ άρα παραγωγίσιμη ·συνάρτηση. · Επίσης και το 2° μέλος της (Ι) είναι παραγωγίσψη συνάρ­τηση οπότε είναι:

[r(x2 + 5x)J' = (2χ7 + 71nxJ ή f'(x2 + 5x)· (x2 + 5xJ = 14χ6 + 7 ·_!_ ή χ (2x + 5) · f'(x2 + 5χ )= ι4χ6 +2 . χ

Από την τελευταία ισότητα για χ = ι ( εφόσον αυτή ισχύει για κάθε χ > Ο) παίρνουμε: 7 . f'(6) = ι4+ 7 <=> 7f'(6) =2 ι <=> f'(6) = 3 .

Άρα λ = 3 οπότε ε: y = 3χ + β . Ακόμη από

την (ι) για χ = ι παίρνουμε: f( 6) = 2 οπότε Μ( 6,f( 6))= ( 6,2) .

Όμως Μ Ε ε άρα 2 = 3 · 6+ β<:>β = -ι6 . Έτσι ε: y = 3χ -ι6 είναι η ζητούμενη εφα­

πτομένη. Παρατήρηση 3 Σύμφωνα και με την προηγούμενη άσκηση για

να ορίσουμε την εφαπτομένη της γραφικής παρά­στασης της συνάρτησης f στο σημείο Μ ( χ0 , f ( χ0 )) εργαζόμαστε ως εξής:

α) Θέτουμε ως εφαπτομένη την ευθεία ε: y =λχ+β .

β) Το λ = f' (χ0 ) . ΕΥΚΛΕΙΔΗΣ Β' λcrr' τ.ΙΠ2

Page 75: Ευκλειδης Β 45

Μαθηματικά pα την Γ Λυκείου

γ) Βρίσκουμε τις συντεταγμένες του σημείου Μ που είναι και σημείο της ε άρα την επαληθεύει και έτσι υπολογίζουμε το β.

Άσκηση 4 Δίνεται η συνάρτηση f(x)= ιη(2χ - χ1)+ κχ :+- λ με κ, λ ε JR . α) Να pρεθεί το πεδίο ορισμού της. p) Να pρεθεί η f'(x) . γ) Να pρεθούν οι τιμές των κ, λ ώστε η ευ­

θεία ε: y = 2χ -1 να εφάπτεται της γραφικής παράστασης της f στο σημείο Α(ι,r(ι)).

δ) Για την τιμή του κ που pρέθηκε στο ε­ρώτημα (γ) να pρεθούν:

3 i) Ο ρυθμός μεταpολής της f στο Χ0 = 2 . ii) Ο συντελεστής διεύθυνσης της εφαπτο­

μένης της γραφικής παράστασης της f στο ση-

μείο s(i,r(i)} iii) Η τετμημένη · χ1 του σημείου

Γ(χ1 ,f(χ1 )) στο οποίο η εφαπτομένη της γρα­φικής παράστασης της f είναι παράλληλη στην ευθεία ζ: y = χ+ 2004 .

Λύση α) Πρέπει: 2χ - χ2 > Ο <=> Ο < χ < 2 άρα

Α = (0, 2) . β) Η f είναι παραγωγίσιμη στο Α με: f' (χ) = (Ιη (2χ -χ2 )+ κχ + λ) =

ι , 2 - 2χ = 2 ( 2χ - χ2 ) + κ = 2 + κ . 2χ - χ 2χ -χ γ) Εφόσον η ευθεία ε: y = 2χ - ι εφάπτεται

της Cr στο σημείο Α (ι, f (ι)) θα είναι: (β)

f'(ι) = 2<=>κ = 2 . Ακόμη f(ι) = κ+λ=2+λ οπό-τε Α(1,2+λ) και Α ε ε άρα: 2+λ= 2-ι <::> λ=-ι . Δηλαδή κ = 2 και λ = -ι οι ζητούμενοι αριθμοί.

τε:

(β) 2 - 2χ δ) Για κ = 2 είναι: f' (χ) = 2 + 2 οπό-2χ -χ

3 ( 3 ) 2- 2- 2 - 3 i) f' 2 = 2 � -(l)2 + 2 =

3 -2 + 2 =

2 2 4

-ι 4 2 = - + 2 = --+ 2 = - ο ζητούμενος ρυθμός 3 3 3 4

μεταβολής.

ii) rH)� 2��(1) ' + 2 � :�l + 2 � 2 2 4

ι 4 ιο ζ , λε ' = - + 2 = -+ 2 = - ο ητουμενος συντε στης 3 3 3 4

διεύθυνσης. iii) Αν ζ' η εφαπτομένη ευθεία στο σημείο Γ

τότε ζΊΙ ζ <=> �· = � <=> f'(χι ) = ι <::> 2-2Χι2 + 2 =

2χι -Χι 2-2χι 2 = ι <=> 2 = -ι <=> 2 -2Χι = -2Χι + Χι <=> 2χι - Χι

2 2 ' Χι = -J2 � Α}

<=> Χι = <=> η Χι = J2 ε Α

Άρα Χι = J2 η ζητούμενη τιμή.

Παρατήρηση 4 Για τις ευθείες υπενθυμίζουμε τα εξής: α) Αν ει 11 ε2 <=> λε, = λε2 . β) Αν ει l_ ε2 <=> �ε, · λε2 = -1 οπότε

ι ι λε, = -λ και λε2 = -λ . ε2 ει

γ) Αν η ευθεία ε σχηματίζει με τον άξονα χ'χ γωνία ω τότε: λε = εφω .

δ) Αν η ευθεία ε είναι παράλληλη στον άξονα χ'χ τότε λε = Ο .

λσκηση 5 α) Δίνεται η συνάρτηση f{x) = eα'x' με

χ ε JR . Να pρεθούν οι τιμές του πραγματικού α­ριθμού α όταν ισχύει:

f"( x)- 2α1x · f'(x)-8f(x) = Ο .

p) Για τη μεγαλύτερη από τις τιμές του α που pρέθηκαν στο (α) ερώτημα να λυθεί η εξί-σωση:

f'( x)-8f( χ)+ f"( χ)= Ο .

Λύση α) Έχουμε τη συνάρτηση f (χ) = eα2χ2 που εί­

ναι δύο φορές παραγωγίσιμη στο JR με

ΕΥΚΛΕΙΔΗΣ Β' λστ' τ.l/73

Page 76: Ευκλειδης Β 45

\1αθηματικά yια την Γ' Λυκείου

f' (x) = (eα2x2 j = eα2χ2 · (α2χ2 ) = 2α2χ · eα'•' και

f"(x) = [ 2α2x · ea2x2 J' = 2α2 (χ · e0:'1 J =

= 2α2 { eα2χ2 + χ · eα2χ2 · (α2χ2) ] =

2α2 (ea,x, + 2α2χ2 · ea,x, )= 2α2ea,x, +4α4 · Χ2eα,χ, . Έτσι: f"(x)-2α2x · f' (x)-8f(x) = O �

� 2α2ea,x, +4α4x2ea2x2 - 'lιix·'lιix·erix, -8eιl-x2 =0� � 2α2eα2χ2 + 4α4x2ea2x2 -4α4x2ea2x2 -8ea2x2 =0 �

η

2(α2 -4)ea2x2 = 0 � α2 -4 = 0 � α�-2} α = 2

οι ζητούμενες τιμές. β) Για α = 2 είναι: f (x) = e4x, '

εξίσωση: f' ( x)-8f(x) +f"(x) = Ο �

8x(1 - 8x)e4x2 = 0 � ή � ή χ = Ο .1 χ = Ο

οι ζη-1 -8χ = 0 1 χ = -

8 τούμενες λύσεις της εξίσωσης.

Παρατήρηση 5 Σύμφωνα με την προηγούμενη άσκηση, όταν

θέλουμε να δείξουμε σχέση ή να υπολογίσουμε κάποια παράμετρο ή να λύσουμε εξίσωση κλπ. που περιέχει τις εκφράσεις f, f' , f" κλπ. αφού πρώτα τις υπολογίσουμε, στη συνέχεια τις aντικαθιστού­με και εργαζόμαστε αλγεβρικά μέχρι να οδηγη­θούμε στο ζητούμενο.

Γ. Εφαρμογές των παρα'Υόηων Μονοτονία - Ακρότατα συνάρτησης Επίλυση προβλημάτων μεyίστου - ελαχί-

στου

Ερωτήσεις θεωρίας 1 . Να διατυπωθούν οι προτάσεις με τις οποί­

ες διαπιστώνουμε τη μονοτονία μιας συνάρτησης. Πότε αυτή είναι γνησίως αύξουσα και πότε γνησί­ως φθίνουσα;

2. Να διατυπωθούν οι προτάσεις με τις οποί­ες διαπιστώνουμε ότι μια συνάρτηση f στο χ0 ε (α, β) παρουσιάζει μέγιστο ή ελάχιστο.

Λυμένες ασκήσεις

Άσκηση 6 Δίνεται η συνάρτηση ( ) l s 1 4 2 3 r χ = sx -4χ -3χ - 7 , χ ε � .

α) Να βρεθούν τα διαστήματα μονοτονίας της συνάρτησης.

β) Ν α βρεθούν οι θέσεις ακροτάτων της συνάρτησης.

Ύ) Να βρεθούν οι τιμές των ακροτάτων. δ) Να βρεθούν τα ακρότατα. Λύση Η συνάρτηση f είναι παραγω-yίσιμη στο � με

' ( ) ( 1 s 1 4 2 J )' 4 J 2 f χ = 5 χ -4 χ -3 χ - 7 = χ - χ - 2χ .

χ =Ο ή

Έτσι: f'(x) =O�x2 (x2 -χ-2)=0�χ =-1 ή

χ = 2 Επίσης: f' (x) > O � χ2 (χ2 - χ -2) > 0 �

� χ2 -χ -2 > 0 � χ �-1} χ > 2

Έτσι για την f σχηματίζουμε τον παρακάτω πίνακα μεταβολών.

χ r f

Οπότε:

-οο -1 ο 2 +οο + � - ό - ό +

�!ι.._.....,. �!� ' ' ' ' ' ' Τ.Μ. ΤΕ.

α) Η f είναι γνησίως αύξουσα στα διαστήμα­τα: ( -οο, -1] , [ 2, +οο) . Επίσης η f είναι γνησίως φθίνουσα στο διάστημα [- 1 ,2].

β) Στο σημείο χ1 = -1 η f εμφανίζει τοπικό μέγιστο ενώ στο σημείο χ2 = 2 η f εμφανίζει τοπι­κό ελάχιστο.

Ύ) Είναι f ( -1) = - 407 η τιμή του τοπικού 60 ' f (2) 149 ' ' μεyιστου και = -15 η τιμη του τοπικου ε-

λαχίστου. δ) Τέλος το σημείο A(-1,f{-1))=( -1,-:J

είναι το τοπικό μέγιστο και Β ( 2, f ( 2)} = ( 2,- 1::) είναι το τοπικό ελάχιστο.

ΕΥΚΛΕΙΔΗΣ Β' λστ' τ.l/74

Page 77: Ευκλειδης Β 45

Μαθηματικά yια την Γ Λυκείου

Παρατήρηση 6 Όπως φαίνεται και από την προηγούμενη ά­

σκηση η μελέτη των ριζών και του πρόσημου της f' μας πληροφορούν για τη μονοτονία και τα α­κρότατα της συνάρτησης f. Πρέπει να σημειωθεί ότι άλλο είναι η θέση τοπικού ακροτάτου και άλλο το τοπικό ακρότατο. Συγκεκριμένα το χ0 Ε Α εί-ναι η θέση του τοπικού ακροτάτου, το f ( χ0 ) η τι­μή του ακροτάτου και το σημείο Μ ( χ0 , f ( χ0 )) εί­ναι το τοπικό ακρότατο.

Άσκηση 7 Δίνεται η συνάρτηση f (χ) = eια. + λχ με

χ Ε JR και κ, λ Ε JR •

α) Να βρεθεί η f' και η f" . β) Να βρεθεί η τιμή του κ ώστε να ισχύει: f"(x)+ 2f'(x)+ e"x - 2λ = Ο .

γ) Για την τιμή του κ που βρέθηκε στο ε­ρώτημα (β) να βρεθεί η τιμή του λ ώστε η εφα­πτομένη της γραφικής παράστασης της f στο σημείο M(o,r(o)) να Ι:ίναι παράλληλη στον ά­ξονα χ'χ .

δ) Για τις τιμές ταιν κ, λ που βρέθηκαν στα ερωτήματα (β) και (γ) να μελετηθεί η συνάρτη­ση f ως προς τη μονοτονία και τα ακρότατα.

Λύση α) rια χ Ε JR είναι: f( χ) =( eισι +λχJ =κeισι +λ, f" (χ) = ( κe"" + λ) = κ2e"" .

'. ι (α) β) Επίσης: f"(x) +2f' (x)+ e"" -2λ = Ο� κ2e"" + 2 (κe"" + λ )+ e"" -2λ = Ο � - ,

� κ2e"" + 2κe"" + 2λ + e"" -2λ = Ο<=> ( κ2 + 2κ + 1 )e"" = Ο� (κ + 1 )2 = Ο <=> κ = -1 η ζητούμενη τιμή.

γ) Για κ = -1 είναι f {x) = e-x +λχ και f' (χ) = -e-x + λ . Αν ε είναι η εφαπτομένη της Cr στο σημείο Μ (Ο, f (Ο)) και η ε 11 χ'χ τότε πρέπει: ·

λ. = 0� f'(O) = O � -l + λ = Ο � λ :::;: l η ζητού­μενη τιμή.

δ) Λόγω των (β), (γ) είναι: f (x) = e-x + χ και f' (x) = -e-x + 1 .

Έτσι: f'(x) =O�-e-x + l=O�e-x =l�x=O και

f'(x) >0�-e-x +1 > O�e-x < l�e-x <e0 �.

� -χ < Ο � χ > Ο . Με τα παραπάνω δεδομένα για την f έχουμε

τον παρακάτω πίνακα μεταβολών. χ -οο ο +οο r - 6 +

' f � i � Τ.Ε.

Άρα η f είναι γνησίως αύξουσα στο ( -οο, Ο] και η f είναι γνησίως φθίνουσα στο [Ο, +οο) .

Επίσης στο χ0 = Ο εμφανίζει ελάχιστο με nμή f (O) = l . Δηλαδή το σημείο A (O,f (O)) = (O, l) εί­ναι το ελάχιστο της f.

Άσκηση 8 Το πλήθος σε δεκάδες χιλιάδες κομμάτια

των πωλήσεων μιας εταιρείας που παράγει ηλε­κτρονικούς υπολογιστές, δίνεται από τη συνάρ-

Ρ( ) 400t , άζ , τηση t = 2 , οπου το t εκφρ ει σε μη-. t + 25 νες το χρόνο κυκλοφορίας ενός νέου μοντέλου.

α) Να βρεθεί ο ρυθμός μεταβολής των πω­λήό'εrον της εταιρείας μετά από ένα μήνα, από την κυκλοφορία στην αγορά ενός νέου μοντέλου.

β) Να βρεθεί η χρονική στιγμή (μήνας) κα­τά την οποία οι πωλήσεις παίρνουν τη μέγιστη τιμή.

γ) Να βρεθεί η μέγι<tτη ποσότητα σε εκα­τοντάδες χιλιάδες κομμάτια που πουλά η εται­ρεία το μήνα που βρέθηκε στο ερώτημα (β).

Λύση Για t � Ο έχουμε τη συνάρτηση των πωλήσε-. 400t ων P (t) = 2 . t + 25 α) Η συνάρτηση Ρ είναι παραγωγίσιμη με

I ( 400t )I 400(t2 + 25)-400t ; 2t P (t) - - -. - t2 + 25 - (t2 + 25)2 -

400t2 + 10000 -800t2 10000-400t2 = = οπότε ο ( t2 + 25 )2 ( t2 + 25 )2 ζητούμενος ρυθμός μεταβολής είναι: Ρ' (Ι) 10000 -400 = 9600 :: 14 2 ή 142.000 262 676 , κομμάτια το μήνα.

β) Για t > Ο είναι: P'(t) = O � 10000-400t2 = Ο � (t2 + 25)2

10000 -400t2 = 0 � t2 = 25 οπότε t = 5 .

ΕΥΚΛΕΙΔΗΣ Β' λστ' τ.l/75

Page 78: Ευκλειδης Β 45

Μαθηματικά yια την Γ Λυκείου

Επίσης: Ρ' ( t) > Ο <=> 10000-400t2 > Ο <=> t2 < 25 οπότε 0 < t < 5 .

Έτσι για τη συνάρτηση Ρ έχουμε τον παρακά­τω πίνακα μεταβολών.

001 / 5 +οο I Τ.Μ.

Απ' όπου συμπεραίνουμε ότι η συνάρτηση Ρ παίρνει τη μέγιστη τιμή της για t = 5 . Δηλαδή ύ­στερα από 5 μήνες η εταιρεία θα πετύχει το μέγι­στο των πωλήσεων.

γ) Λόγω του ερωτήματος (β) είναι 400 · 5 2000 Ρ ( 5) = 52 2 = -- = 40 δεκάδες χιλιάδες κομ-

+ 5 50 μάτια ή 4 εκατοντάδες χιλιάδες κομμάτια.

Παρατήρηση 7 Οι μέθοδοι του Διαφορικού Λογισμού για τον

προσδιορισμό των ακροτάτων τιμών ενός μετα­βαλλόμενου μεγέθους έχουν πρακτική εφαρμογή σε πολλές περιοχές των επιστημών αλλά και της καθημερινής ζωής. Για την επίλυση τέτοιων προ­βλημάτων αυτό που κυρίως προέχει είναι η μετα­τροπή του προβλήματος που είναι διατυπωμένο στην καθημερινή γλώσσα σε πρόβλημα μεγίστου ή ελαχίστου με τον ορισμό μιας συνάρτησης της ο­ποίας πρέπει να βρεθούν τα ακρότατα.

Επισημαίνεται ότι η διαδικασία επίλυσης προ­βλήματος δεν είναι τίποτα άλλο παρά μια συλλογή στρατηγικών τις οποίες κάθε λογικά σκέπτόμενος άνθρωπος πρέπει να χρησιμοποιήσει προκειμένου να αντιμετωπίσει ένα πρόβλημα.

Σχετικά με την επίλυση προβλημάτων με τη βοήθεια του Διαφορικού Λογισμού πρέπει να ανα­φερθεί ότι πολλά προβλήματα μεγίστου ή ελαχί­στου περιέχουν διακριτές μεταβλητές. Για παρά­δειγμα ο αριθμός των παραγόμενων μονάδων ενός προϊόντος καθώς και ο αριθμός των εργαζομένων σε ένα εργοστάσιο πρέπει να είναι μη αρνητικοί α­κέραιοι αριθμοί.

Ο Διαφορικός Λογισμός όμως δεν εφαρμόζε­ται απευθείας σε προβλήματα που περιέχουν δια­κριτές μεταβλητές. Ωστόσο, μπορούμε μερικές φορές να οδηγηθούμε στη λύση ενός τέτοιου προ­βλήματος υποθέτοντας ότι κάθε μεταβλητή παίρνει τιμές σε όλο το σύνολο των πραγματικών αριθμών ή σε κάποιο διάστημά του, ακόμα και αν η φυσική ερμηνεία της μεταβλητής έχει νόημα μόνο για δια­κριτές τιμές. Έτσι χρησιμοποιώντας το Διαφορικό Λογισμό βρίσκουμε μια λύση για το μαθηματικό

μοντέλο, η οποία ελπίζουμε ότι προσεγγίζει τη λύ­ση του πραγματικού προβλήματος.

Σημείωση. Η παρατήρηση 7 μεταφέρει αυ­τούσια ό,τι γράφεται στη σελίδα 147 των οδηγιών του Παιδαγωγικού Ινστιτούτου για τη διδασκαλία των μαθηματικών στο Λύκειο για το έτος 2001-2002.

Δ. Θέματα πανελληνίων εξετάσεων που α­ντιστοιχούν στο 1 ο κεφάλαιο.

Θέμα 1° Δίνεται η συνάρτηση : · f(x) =

ημχ χ- 3

α) Να βρείτε το πεδίο ορισμού της. (Μονάδες 7) β) Να βρείτε την παράγωγο f της συνάρτησης f.

(Μονάδες 9) γ) Να υπολογίσετε την τιμή f(Ο).(Μονάδες 9)

(Πανελλήνιες εσπερινού λυκείου 2000) Θέμα 2ο Δίνεται η συνάρτηση: f(x) =2χ3 + 5χ + 3 α) Να βρείτε την πρώτη παράγωγο της f.

(Μονάδες 12) β) Να δείξετε ότι η συνάρτηση f είναι γνησί

ως αύξουσα στο πεδίο ορισμού της. (Μονάδες 13)

(Πανελλήνιες εσπερινού λυκείου 2000) Θέμα 3ο · Α. α) Δίνεται η συνάρτηση F(x) = f(x) + g(x)

Α ν οι συναρτήσεις f, g είναι παραγωγίσιμες, να α ποδείξετε ότι: F'(x) = f(x) + g'(x) . (Μονάδες 8)

β) Να γράψετε στο τετράδιο σας τις παρα γώγους των παρακάτω συναρτήσεων:

cf(x), f(x)g(x), f(x) με g(x):;eO, όπου c πραγ g(x) ματική σταθερά. (Μονάδες 4,5)

Β. α) Να γράψετε στο τετράδιο σας τα γράμ­ματα της στήλης Α και δίπλα τον αριθμό της στή­λης Β που αντιστοιχεί στη σωστή απάντηση.

�TJJAR Α Σ')'ΙL\11 Β .σ'bνάρm.σu πιιφτη παράγωγ� α. χ2+3 1 . 1 -ημχ

β. χ+συνχ 2. 3χ2 -8χ

γ. χημχ 3 . 2χ+3 δ. χ3 -4χ2 4. ημχ-χσυνχ

5 . 2χ 6. 3χ2-4χ 7. ημχ+χσυνχ

(Μονάδες Β) β) Να γράψετε στο τετράδιο σας το γράμμα

που αντιστοιχεί στη σωστή απάντηση. Η πρώτη παράγωγος της συνάρτησης

ΕΥΚΛΕΙΔΗΣ Β' λστ' τ.l/76

Page 79: Ευκλειδης Β 45

Μαθηματικά -yια την Γ Λυκείου

e' f(x) = - , x:;t:O είναι:

χ e' - xe' e'x + e' Α: ex, Β: 2 , Γ: 2 , χ χ

e'x - e' xe' - e' Δ: 2 , Ε: ---χ χ

(Μονάδες 4,5) (Πανελλήνιες γενικού λυκείου 2000) Θέμα 4ο Δίνεται η συνάρτηση f(x)= συνχ + ημχ. Α. Να αποδείξετε ότι f(x)+ f '(χ)=Ο.

(Μονάδες 8) Β. Να βρείτε την εξίσωση της εφαπτομένης

της γραφικής παράστασης της f στο σημείο Α(Ο, 1 ) (Μονάδες 8)

Γ. Να βρείτε την τιμή του λεR για την οποία ισχύει η σχέση λf ( �) -2f( �) =2. (Μονάδες 9)

(Πανελλήνιες γενικού λυκείου 2001 ) Θέμα 5ο Α. Να αποδείξετε ότι η παράγωγος της ταυτο­

τικής συνάρτησης f(x)=x είναι f(x)=1 . Μονάδες 9

Β. Για καθεμιά από τα παρακάτω προτάσεις να γράψετε στο τετράδιό σας τον αριθμό της και, ακριβώς δίπλα, την ένδειξη (Σ),, αν η πρόταση εί­ναι σωστή, ή (Λ) αν αυτή είναι λανθασμένη θεω­ρώντας ότι υπάpχουν οι f(x) και g'(x).

1 . [f(x)+g(x)] ' = f(x)+g'(x) 2. (ημχ)' =. συνχ 3. [f(x) · g (χ)] ' = f(x) · g'(x)

4. [��] ' f(x) -g(x)+f{x) ·g(x) g( ) 0 ,z ' χ :;t: [g(x)J

5. (J;.)' = 1Γ ' Χ > 0

2νχ 6. [c·f(x)] ' = c·f(x) 7. (συνχ)' =ημχ 8. (χΡ) '=χΡ·', ρ ρητός, χ>Ο Μονάδες 16

(Πανελλήνιες εσπερινού λυκείου 2002) Θέμα 6ο

Δ' ' fiY ) Χ2 + 3x-1Q ινεται η συναρτηση \χ =

χ-2 α. Ν α βρείτε το πεδίο ορισμού της συνάρτη-

σης f(x) Μονάδες 5 μ. να βρείτε τα: limf(x) , lim f(x)

Χ-+1 Χ-+2 Μονάδες 12

γ. Να δείξετε ότι η συνάρτηση f(x) είναι γνη-σίως αύξουσα στο (2 , + οο) Μονάδες 8

(Πανελλήνιες εσπερινού λυκείου 2002) Θέμα 7ο

Δ' ' f( ) 2χ ινεται η συναρτηση χ = -- .

χ + 1 α. Να βρείτε το πεδίο ορισμού της συνάρτη-

σης f. Μονάδες 4 μ. Να υπολογίσετε το όρω lim f(x) χ-+3

Μονάδες 4 γ. Να βρεθεί η πρώτη παράγωγος της f.

Μονάδες 7 δ. Να βρεθούν οι εφαπτόμενες της καμπύλης

της συνάρτησης f που η παράλληλες στην ευθεία ψ =2χ+5 . Μονάδες 10

(Πανελλήνιες γενικού λυκείου 2002

Jι ι • : ι:

ΚΥΚΛΟ.ΟΡΟΥΝ ΑΛ ΓΕ Β ΡΑ

a· Λυκείου

ΑΛrΕΒΡΑ •· Λυκείου

ΑΝΑΣfΑΣΙΟΣ Χ. ΜΠΑΡΛΑΣ ΔΙΑΜΑΝΤΗΣ Α. ΜΑΝΕΣΙΩΤΗΣ

ΠΑΝΑΠΩΤΗΣ Α. ΚΥΡΙΑΖΗΣ

ΕΚΔΟΣΕΙΣ ΕΛΛΗΝΟΕΚΔΟ1ΙΚΗ 1ΉΛ.: 010-3613676

Page 80: Ευκλειδης Β 45

• Ο συνάδελφος ι. Μαντάς είχε προτείνει στο προηγούμενο tεύχος τα παρακάτω προβλήμα­τα. 1. Κάθε πόση ώρα συναντιέται ο λεπτοδεί­

κτης με τον ωροδείκτη του ωρολογίου

Λύση -Έστω ότι η ώρα είναι 12. Τότε οι δύο δείκτες βρίσκονται στην ίδια θέση.

Μετά από 1 ώρα ο λεπτοδείκτης θα βρίσκεται 30° πίσω από τον ωροδείκτη.

Αν t είναι ο χρόνος σε λεπτά, τότε ο λεπτο­δείκτης σε χρόνο t διανύει 6 t μοίρες ενώ ο ωρο-δ ' δ ' 1 ' εικτης ιανυει 2" t μοιρες.

Αν σε t λεπτά ο λεπτοδείκτης και ο ωροδεί­κτης συναντιούνται ξανά πρέπει να ισχύει:

1 1 1 60 - . 6t = -t + 30 <::::> -t = 30 <::::> t = - = 5,45mιn ::::: 5 2 2 1 1

λεπτά, 27 δευτερόλεπτα και 27 εκατοστά. Άρα ο λεπτοδείκτης και ο ωροδείκτης συνα­

ντιούνται κάθε 1 ώρα, 5 λεπτά, 27 δεύτερα και 27 εκατοστά του δευτερολέπτου

2) Μια διαφημιστική πινακίδα αποτελείται από ένα πανί προσαρμοσμένο σ' ένα μηχανισμό, όπως στο σχήμα:

μηχανισμός

πανί

Επάνω στο πανί είναι τυπωμένες 4 διαφη­μίσεις, στη σειρά η μία κάτω απ' την άλλη, ό-

πως στα φιλμ. Ο μηχανισμός τυλίγει και ξετυλί­γει το πανί, σαν πάπυρο, κατά τακτά χρονικά διαστήματα, ώστε να εμφανίζεται κάθε φορά και μια διαφορετική διαφήμιση. Πρώτα εμφα­νίζεται η πρώτη διαφήμιση, ύστερα η δεύτερη κ.ο.κ. Όταν εμφανιστεί η τέταρτη διαφήμιση ο μηχανισμός τυλίγει αντίστροφα το πανί ώστε να εμφανιστεί στη συνέχεια η τρίτη, η δεύτερη κ.ο.κ. Όταν πάλι εμφανιστεί η πρώτη ο μηχανι­σμός στρέφει το πανί αντίστροφα ώστε να εμ­φανιστεί η δεύτερη κ.ο.κ. Ας υποQέσουμε ότι πρώτη, δεύτερή, τρίτη και τέταρτη είναι η δια­φήμιση της εταιρείας Α, Β, Γ, Δ αντίστοιχα. Η εταιρεία Β πληρώνει 20.000 δρχ. ημερησίως -(ια να διαφημίζεται στην παραπάνω διαφημιστική πινακίδα.

Λύση Η σει ά των διαφημίσεων είναι:

Β Γ Δ Γ Β , IΑΒ Γ Δ Γ Β I . . . . . . . . .

Άρα για κάθε 2 εμφανίσεις των Β, Γ τα Α, Δ εμφανίζονται από 1 φορά. Επομένως καθεμία ε­ταιρεία από τις Α, Δ πρέπει να πληρώνουν τα μισά απ' όσα πληρώνει καθεμία από τις Β, Γ, δηλαδή οι Β, Γ πρέπει να πληρώνουν από 20.000 δρχ. ημε­ρησίως και οι Α, Δ από 10.000 δρχ. ημερησίως • Ο συνάδελφος Χ. Πατίλας απ' τα Τρίκαλα

προτείνει: Πτυσσόμενη κεραία μήκους 25cm στέκεται

κατακόρυφα με το ένα άκρο της (σταθερό) στο έδαφος. Σκοινί μήκους lm είναι στερεωμένο με τις δύο άκρες του Κ, Λ στο έδαφος και το μέσο του είναι περασμένο από μικρή οπή που υπάρχει όσο άνω άκρο (μεταβλητό) της κεραίας έτσι ώ­στε να είναι τεντωμένο.

Η κεραία αρχίζει να κινείται οριζόντια έτσι ώστε να είναι πάντα κάθετη στο έδαφος και κά­θε χρονική στιγμή t απ' τη στιγμή που άρχισε

, , dh r:--:-ι1 4 1 dx 0 να κινειται ισχυει: dt · ν ι - 4χ- + χ · dt =

, ι , , , οπου χ

< - η αποσταση της κεραιας απ την αρ-2 χική της θέση και h το ύψος της κεραίας σε μέ­τρα.

Να δείξετε ότι: «Το σκοινί κατά τη παρα­πάνω κίνηση είναι τεντωμένο».

ΕΥΚΛΕΙΔΗΣ Β' λστ' τ.l/78

Page 81: Ευκλειδης Β 45

Η στήλη της Αλληλοyραφίας Από τον συνάδελφο Ηλία Ντζιώρα πήραμε

την παρακάτω επιστολή: Αγαπητέ ΕΥΚΛΕΙΔΗ Β', Με αφορμή το τρίτο (3ο) θέμα της Θετικής

και Τεχνολογικής Κατεύθυνσης των Γενικών Εξε­τάσεων της Γ' Λυκείου, σας στέλνω μια πιο γενική διατύπωση και καινούργια συμπεράσματα, ώστε να αποτελέσει θέμα για μελέτη των μαθητών της Γ' Λυκείου:

Δύο συναρτήσεις f και g είναι ορισμένες στο R Η συνάρτηση h = fog είναι « 1-1 )), α) Να δείξετε ότι η g είναι « 1-1 )). β) Να δικαιολογήσετε ότι η f δεν είναι απα­

ραίτητα « 1-1 )) . γ) Είναι γνωστό ότι κάθε πολυώνυμο Ρ(χ),

βαθμού ν � 2 , έχει το πολύ ν διαφορετικές ρίζες. Αν Ρ(χ) = αχ3 +βχ2 +γχ + δ με α, β, γ, δ Ε R και α * Ο , τότε να δείξετε ότι ισχύουν οι προτά­σεις:

γΙ) αν α > Ο , δ > Ο και α+ β + γ + δ < Ο , τότε το Ρ(χ) έχει ακριβώς δύο θετικές και μία αρ­νητική ρίζα.

γ2) αν α > Ο , δ < Ο και α - β + γ- δ < Ο , τό-τε το Ρ(χ) έχει ακριβώς δύο αρνητικές και μία θε­τική ρίζα.

γ3) αν Q(x) = χ3 + αχ2 + β με β > Ο και α + β < -1 , τότε το Q(x) έχει ακριβώς δύο θετι­κές και μία αρνητική ρίζα.

δ) Να δείξετε ότι η εξίσωση: g (f(x) + χ3 + 2χ2 ) = g (r (x) - 3x2 - 2χ + ι) έχει ακριβώς δύο αρνητικές και μία θετική ρίζα.

ε) Να δείξετε ότι η εξίσωση: g (r (x) + χ3 - χ) = g (f (x) + 2χ - 1) έχει ακριβώς δύο θετικές και μία αρνητική ρί-

ζ α. ΛΥΣΗ

f ( g (χι)) = f (g (χ2)) , δηλαδή h (χι ) = h (χ2) και επειδή η h = fog είναι 1-1 θα έχουμε χι = χ2 . Άρα η g είναι 1-1 .

Θα μπορούσαμε να απαντήσουμε και ως εξής: Έστω ότι η g δεν είναι 1-1 , τότε θα υπήρχαν δύο πραγματικοί αριθμοί χι * χ2 με g (χι ) = g (χ2) , οπότε θα είχαμε και f (g (χι )) = f (g (χ2)) ή (fog) (xι ) = (fog) (x2) , δηλαδή h (χι ) = h (χ2) , πράγμα άτοπο, αφού η h, από υπόθεση, είναι 1-1 .

β) Η συνάρτηση f δεν είναι απαραίτητα « 1-1 )) και αυτό φαίνεται με το παρακάτω παράδειγμα (βλ. και Ευκλείδη Β', λ.α. τ. 4/53). Θεωρούμε τις συναρτήσεις f και g με τύπους αντίστοιχα: f ( ) {χ, αν χ Ε (- 1 , 1) χ = και Ο, αν χ Ε R / (- 1 , 1)

χ g (χ) = I I '

χ Ε R 1 + χ Οι συναρτήσεις f και g είναι ορισμένες στο R

και η μεν g, όπως εύκολα μπορεί να διαπιστωθεί είναι « 1-1 )) , ενώ η f δεν είναι « 1-1 )) , γιατί π. χ. αν και 2 * Ο , όμως f (2) = f (Ο) = Ο .

Η συνάρτηση h = fog είναι ορισμένη στο R και είναι « 1-1 )), γιατί αν πάρουμε Χι * χ2 , τότε θα είναι g (χι) * g (χ2) · Αλλά

lg (x)l = lχΊ I < 1 για κάθε χ Ε R , 1 + χ οπότε

f (g (xι )) = g (χι) και f ( g (x2)) f (g (χι)) * f (g (χ2)) , (fog) (χι) * (fog) (χ2 ) , h (χι) * h (χ2 ) .

που

= g (x2) , άρα δηλαδή

σημαίνει

γΙ) Η συνάρτηση Ρ(χ) είναι συνεχής στο R ως πολυωνυμική.

Ισχύει: Ρ (Ο) = δ > Ο και α) Έστω τότε Ρ (1) = α + β + γ + δ < Ο .

ΕΥΚΛΕΙΔΗΣ Β' λστ' τ.l/79

Page 82: Ευκλειδης Β 45

Η στήλη της Αλληλσyραφίας

Επιπλέον έχουμε ότι: lim Ρ (χ) = lim (αχ3 + βχ2 + γ + δ) =

Χ -+·· . Χ--++οο

l ιm ι αχ . I = + οο, άρα υπάρχει χ1 > ι ώστε

lim Ρ (χ) = lim (αχ3 + βχ2 + γ + δ) = χ --+-00 χ --+-00

ε) Αφού η g είναι 1-1 διαδοχικά έχουμε: f(x) + χ3 - χ = f (x) + 2χ - 1 <=> χ3 - 3χ + 1 = ο (3) Παρατηρούμε ότι για το πολυώνυμο Ρ (χ) = χ 3 - 3χ + 1 ισχύουν οι προϋποθέσεις για την πρόταση (γι), καθόσον τό Ρ(χ) είναι 3ου βαθ-

lim (αχ3) = - οο, άρα υπάρχει χ2 > ο ώστε μού με α = 1 > Ο και α+β+γ+δ=1+0- 3+1= χ --+-00

Ρ (χ2) < Ο Παρατηρούμε τώρα ότι η συνάρτηση Ρ(χ) ι­

κανοποιεί τις προϋποθέσεις του θεωρήματος Bol­zano σε καθένα από τα διαστήματα [χ2 , Ο] , [0, ι ] και [ ι , χι ] , άρα υπάρχουν ένα (τουλάχιστον) ρι ε (χ2 , Ο) με Ρ (ρι ) = Ο , ένα (τουλάχιστον) ρ2 ε (0, ι) με Ρ (ρ2) = Ο και ένα (τουλάχιστον) ρ3 ε ( ι , χι) με Ρ (ρ3) = Ο και επειδή η συνάρ­τηση είναι 3ου βαθμού έχει το πολύ 3 ρίζες, άρα το πολυώνυμο Ρ(χ) έχει ακριβώς 3 ρίζες, δύο θετικές τις ρz, ρ3 και μία αρνητική, την ρ ι .

γz) Με ανάλογους συλλογισμούς δείχνουμε την πρόταση ( γz), καθόσον ισχύουν:

= - 3 + 2 = - ι < ο ' άρα, σύμφωνα με την πρόταση (γι), το Ρ(χ) έχει ακριβώς δύο θετικές και μία αρ­νητική ρίζα. • Ο συνάδελφος Ελευθέριος Πρωτοπαπάς από

το Περιστέρι μας έστειλε ένα γράμμα με τις παρακάτω επισημάνσεις για το τεύχος 43: - Για το διαγωνισμό ΕΥΚΛΕΙΔΗ (σελ. 20):

«Νομίζω ότι είναι πιο ε6κ:ολο για κάποιον να σκε­φθεί ως εξής: Έστω ότι κανένας μαθητής δε δήλω­σε και τα 4 μαθήματα. Αυτό σημαίνει ότι κάθε έ­νας από τους 20 μαθητές θα έχει δηλώσει το πολύ 3 μαθήματα, οπότε οι συμμετοχές θα είναι συνολι­κά το πολύ 20 · 3 = 60 .

Όμως, κατά τα δεδομένα οι συμμετοχές είναι: ι5 + ι3 + ι4 + ι9 = 6ι Ρ (Ο) = δ < Ο και Συνεπώς, υπάρχει ένας τουλάχιστον μαθητής Ρ (- 1) = - α + β - γ + δ = - (α - β + γ - δ) > Ο που δήλωσε όλα τα μαθήματα».

και lim Ρ (χ) = lim (αχ3) = + οο και χ -++σο χ -++σο

lim Ρ (χ) = lim (αχ3) = - οο χ --+-οο χ --+-οο

γ3) Για το Q (χ) = χ3 + αχ2 + Οχ + β

πολυώνυμο έχουμε

α' = ι > ο και ι + α + ο + β < ο (γιατί α + β < - ι ). Επομένως, σύμφωνα με την πρότα­ση (γι) το Q(x) έχει δύο θετικές και μία αρνητική ρίζα.

δ) Αφού η g είναι · ι-ι έχουμε διαδοχικά: f (x) + χ3 + 2χ2 = f (x) - 3χ2 - 2χ + ι <=> χ3 + 5χ2 + 2χ - ι = ο (2) Παρατηρούμε τώρα ότι το πολυώνυμο Q (χ) = χ3 + 5χ2 + 2χ - ι είναι 3°υ βαθμού και ισχύει α = 1 > Ο και α - β + γ - δ = ι - 5 + 2 - (- ι) = - 5 + 4 = - ι < Ο

' , άρ!i, σύμφωνα με την πρόταση (γ2) το Q(x) έχει ακριβώς δύο αρνητικές και μία θετική ρίζα.

- Για τον ορισμό 5 (σελ. 34) για το τετράγωνο όπου καθώς σημειώνει υπάρχει τρεις φορές η λέξη «παραλληλόγραμμο».

Η Σ.Ε. απαντά: «Δίκιο έχει ο συνάδελφος. Ε­μείς απλά επαναλάβαμε τον ορισμό του σχολικού βιβλίου».

Επίσης, δίνει απαντήσεις στα εξής θέματα: •:• Θέμα ι που πρότεινε ο ακαδημαϊκός κ:. Ν.

Αρτεμιάδης (σελ. ι4). •:• Θέμα 2 που πρότεινε ο ακαδημαϊκός κ:. Ν.

Αρτεμιάδης (2 τρόποι). •:• Θέμα 1 που πρότεινε ο συνάδελφος Ν.

Βαδιβούλης. •:• Θέμα 2 που πρότεινε ο συνάδελφος Ν.

Βαδιβούλης.

λόγω πληθώρας

ΕΥΚΛΕΙΔΗΣ Β' λστ' τ.l/80

Page 83: Ευκλειδης Β 45

Εκδόσει� τη� ΕΛΛΗΝΙΚΗΣ ΜΑθΗΜΑΤΙΚΗΣ ΕΤΑΙΡΕΙΑΣ \F.\_τ((t

\ llt'Ι'-tn. \HiiH\1\flll.fl�{ H.IFH.\.� BU.JfΠ'f"i ΜΑΘΗΜΑτΙΚΗ 55

ΕΠΙΘΕΩΡΗΣΗ ·�..;.;

Ε'ριuvηηι:η διοστοσn

,.,.� ... . · �

EUCL IDES Υ . -·--EIIU:I8

--rιι.�..: .... -·

LOU/5 BRAND

ΜΑθΗΜΑ11ΚΗ ΑΝΑΛΥΣΗ

-znι

- - -----

.._ _

__ .,. 1

1,-νι-·οR..ιι-ιτιιs 'fl

Σl'ΟιχF.ιΟ.4ΗΣ rt:nMετrι,\ .,πο ΛΝΩΤ.ΕΡΗ ΣκQrιι;ι ��ΩΡJΑ

ΛΡΙΘΜΩι�

ΕυκλείδηςΑ": Τεύχος 2 ευρώ Διεθνής ΜαθημαΊικές 11οο ΓΊαvελληνkχι Συνεδρίου 17 ευρώ Διαλέξεις: Ο τόμος Συνδρομή (4 τεύχη) 10 ευρώ Ολυμπιάδες 1959 - 1999: 20 ευρώ 14ou ΓΊαvελληνkχι Σιινεδρίσυ 17 ευρώ Μαθημαnκή Ανάλυση

(4 τεύχη + 2 ευρώ ταχυδρομικά) Βαλκανικές Μαθημαnκές 15ou ΓΊαvελληνkχι Συνεδρίου 17 ευρώ Σχολεία: 8 ευρώ δρχ. Ολυμπιάδες 1984-2001: 15 ευρώ 16ou ΓΊαvελληνkχι Συνεδρίου 17 ευρώ

(Louis Brand) Ευκλείδης&:: Τεύχος 2,5 ευρώ Θέματα εξετάσεων στα A.E.I 17οο ΓΊαvελληνkχι Συνεδρίου 25 εuρώ Διαφορικές Εξισώσεις

5 ευρώ

25 ευρώ

Συνδρομή (4 τεύχη) 12 ευρώ 1976 - 1989: 6 ευρώ 1βου ΓΊαvελληνkχι Συνεδρίου 25ευρώ (Stephenson) 10 ευρώ (4 τεύχη + 2 ευρώ ταχυδρομικά) Πρακτικά: Συνέδριο Hermis "92 (Αyyλικά)

Σχολεία: 1 Ο ευρώ 1 οο Πανελληνίου Σιινεδρίσυ 6 ευρώ 25 ευρώ ΕυκλείδηςΓ": Τεύχος 5 ευρώ 2οο Πανελληνίου Σwεδρίου 8 ευρώ Συνέδριο Hennis "94 (Αyyλικά)

Συνδρομή (2 τεύχη) 1 Ο ευρώ 3ou Πανελληνίου Σιινεδρίσu 8 ευρώ 2 τόμοι ο τόμος 15 ευρώ Μαθημ. Επιθεώρηση: 4oιJ..5ou ΓΊανελληνbυ ΣιινεδρΚJυ 8 ευρώ Γραμμική Άλγεβρα

Τεύχος 5 ευρώ 6ou Πανελληνίου Σιινεδρίσu 8 ευρώ (Gr. Muncres) 6 ευρώ Συνδρομή (2 τεύχη) 1 Ο ξυρώ 7οο Πανελληνίου Σιινεδρίσu 8 εuρώ ΗΡΩΝΟΣ ΑΛΕΞΑΝΔΡΕΩΣ

Ιστορία Μαθημαnκών Loήa (4 Ίόμοι) Α, Β, Γ Αι Γ Β ο Ίόμος 8 ευρώ 70 Χρόνια Ε.Μ.Ε. 4 ευρώ Ελληνική Μαθημαnκή Βιβλιογραφία 4 ευρώ Στοιχειώδης Γεωμετρία από Αστρολάβος: Τεύχος 5 ευρώ βου Πανελληνίου Συνεδρίου 8ευj:κ;ι (Ονόματα Γεωμετρικών όρων

Συνδρομή (2 τεύχη) 10 ευρώ 9ou Πανελληνίου Συνεδρίου 8 ευρώ ΓΕΩΜΕΤΡΙΚΑ) 20 ευρώ Ανώτερη Σκοπιά 8 ευρώ Δελtίο (Bυlletin): Τεύχος 7,30 ευρώ 10ou ΓΙανελληνίου Σιινεδρίσu 17 ευρώ (Μετρικά - Διόmρα) 20 ευρώ Θεωρία Αριθμών 17 ευρώ

Τα παλαιότερα τεύχη όλων των εκδόcχων ποaλούντοι μι 11ς τρίχοu� 11μίς τοu aooa

Page 84: Ευκλειδης Β 45

r ΡΑΜΜ Ι Κ Η I ΑΛΓΕΒΡΑ

Σημείο αναφοράς στο εκπαι δευτικό βιβλίο

� 'I r /} r ! !

ΖQΟΔ. ΠΗΓΗΣ 1 8 & ΣΟΛQΝΟΣ, ΑθΗΝΑ 1 0 6 81 ΤΗΛ. : 210 33 0 1 25 1 FAX: 210 3 3 06 9 ' 8